ĐỀ CHÍNH THỨC VÀ ĐỀ XUẤT KÌ THI HSG KHU VỰC DUYÊN HẢI VÀ ĐỒNG BẰNG BẮC BỘ MÔN TIẾNG ANH KHỐI 11

Page 1

ĐỀ THI HSG TIẾNG ANH CHUYÊN DUYÊN HẢI

vectorstock.com/10212099

Ths Nguyễn Thanh Tú eBook Collection DẠY KÈM QUY NHƠN OLYMPIAD PHÁT TRIỂN NỘI DUNG

TỔNG HỢP ĐỀ CHÍNH THỨC VÀ ĐỀ XUẤT KÌ THI HSG KHU VỰC DUYÊN HẢI VÀ ĐỒNG BẰNG BẮC BỘ MÔN TIẾNG ANH KHỐI 11 NĂM 2019 CÓ FILE NGHE VÀ ĐẦY ĐỦ ĐÁP ÁN WORD VERSION | 2020 EDITION ORDER NOW / CHUYỂN GIAO QUA EMAIL TAILIEUCHUANTHAMKHAO@GMAIL.COM Tài liệu chuẩn tham khảo Phát triển kênh bởi Ths Nguyễn Thanh Tú Đơn vị tài trợ / phát hành / chia sẻ học thuật : Nguyen Thanh Tu Group Hỗ trợ trực tuyến Fb www.facebook.com/DayKemQuyNhon Mobi/Zalo 0905779594


TRƯỜNG THPT CHUYÊN NGUYỄN TẤT THÀNH YÊN BÁI KỲ THI HỌC SINH GIỎI CÁC TRƯỜNG THPT CHUYÊN KHU VỰC DUYÊN HẢI VÀ ĐỒNG BẰNG BẮC BỘ ĐỀ THI ĐỀ XUẤT MÔN: TIẾNG ANH – KHỐI 11 I. LISTENING Part 1. For questions 1-5, listen to a podcast on Camber’s Theme Park and choose the correct answer A, B, or C for each question. 1. According to the speaker, in what way is Camber’s different from other theme parks? A. It’s suitable for different age groups. B. It offers lots to do in wet weather. C. It has a focus on education. 2. The Park first opened in…………………… A. 1980 B. 1997 C. 2004 3. What’s included in the entrance fee? A. most rides and parking B. all rides and some exhibits C. parking and all rides 4. Becoming a member of the Adventurers Club means…………………… A. You can avoid queuing so much B. You can enter a Park free for a year C. You can visit certain zones closed to other people 5. The Future Farm zone encourages visitors to …………………… A. buy animals as pets B. learn about the care of animals C. get close to the animals Part 2: For questions 1-5, listen to a radio program about the perils of the online world. Listen and answer the true/false questions below. 1. Jenny's guest works for an e-mail company. 2. David says it's better not to mix work and personal e-mail accounts. 3. There are little software programs that travel the internet looking for e-mail addresses. 4. David advises people not to write e-mails in blue. 5. Fishing is a recent phenomenon. Your answer: 1. 2. 3. 4. 5. Part 3. Listen and answer the following questions using NO MORE THAN THREE WORDS AND/OR A NUMBER for each answer. 1. How did the students do their practical sessions? 2. In the second semester how often did Kira work in a hospital?

1


3. How much full-time work did Kira do during the year? 4. Having completed the year, how does Kira feel? 5. In addition to the language, what do overseas students need to become familiar with?

Part 4: Listen to a piece of news about social media and complete the note with NO MORE THAN THREE WORDS for each gap.

Social Media 2018 The world lived out some of the biggest moments of 2018 like the Winter Olympics on social media. According to the 1. ________________________________ from Twitter, Instagram, and Google, here's what took the Internet by storm in 2018. Over on Twitter, Korean pop group BTS is who have 7.4 million followers for the most tweeted about people in 2018, well as 2. ________________________________ about celebrities or musicians. LeBron James was the second most tweeted about celebrity and the most tweeted about athlete; meanwhile most tweeted about movie of the year was 3. ________________________________ Moving over to Instagram, the #METOO hashtag was used 1.5 million times during 2018, followed by 4. ________________________________ and #MarchforOurLives. The app also revealed that ‘5. ________________________________’ challenge was the top dance movement of the year. So what were people searching. According to Google, Rosanne was the most searched show of the year. The most searched terms of the year were dominated by 6. ________________________________ deaths like the World Cup and Hurricane Florence as well as 7. ________________________________ including Mac Miller, Kate Spade, Anthony Bourdain and Stan Lee. And Demi Lovato who entered 8. ________________________________ after overdosing earlier this year was the most searched person of 2018 followed by newly appointed Duchess Meghan Markle, 9. ________________________________ Brett Kavanaugh, 10. ________________________________ Logan Paul and Khloe Kardashian. LEXICO-GRAMMAR Part 1: Choose the correct answer A, B, C or D to each of the following questions and write your answers in the corresponding numbered boxes provided. 1. In these times of high unemployment everyone thought my giving up my job was________ madness. A. sheer B. steep C. high D. deep 2. With your qualifications, there will be no__________ of firms willing to employ you. A. want B. inadequacy C. deficiency D. shortage 3. The project will be kept__________ until the new manager comes. A. in order B. off and on C. on ice D. off the peck 4. It would set a bad_________ if we changed the rules just for one student. A. precaution B. infringement C. precedent D. manifestation 5. To make a profit, we’ll have to add a few thousand new customers to our list, which is __________. A. a piece of cake B. out of order C. out of our depth D. a tall order 6. Peace talks between the two countries__________, with neither side able to agree on terms. A. collapsed B. collaborated C. collared D. compromise 7. The problem with losing weight is that, if you succeed, all your clothes need to be_________. A. cut down B. taken in C. made down D. let down 2


8. Even though it had ___________ a great deal of support, the political party suffered a resounding defeat. A. flinched B. endowed C. downplayed D. rallied 9. Unconditional acceptance was the principle _________ his core philosophy. A. snagging B. underpinning C. conspiring D. limping 10. As a _________, politicians and other public figures should avoid making direct statements that could be used against them. A. show of hands B. word of mouth C. rule of thumb D. change of heart Your answers: 1. 2. 3. 4. 5. 6. 7. 8. 9. 10. 11. 12. 13. 14. 15. Part 2. Underline five mistakes in the paragraph and provide the correction for each of them. What is digital currency? Bitcoin ison the upswing at the moment, though. A year ago, one unit would have 1………………… cost you a little under $650. Yesterday, that value had surged to more than $5,200 for one bitcoin. Itset a record. Even financial analysts have had a hard time explaining bitcoin`s rises and falls. But they think a couple of things may be 2………………… factoring in here. One, it is possible that countries and companies that donot currently trade in bitcoinwill start doing it. And two, a cheaper form of the currency 3………………… was created earlier this year and that might have made investors more confident in it. Some analysts do not expect bitcoin to stay this valuable though. The head of 4………………… JPMorgan, an investment company, has called it a fraud. And experts believe that governments will be hesitant to allow large payments to be made anonymouslywith 5…………………. bitcoin, because it would be harder to stop crimes and collect taxes. Part 3. Fill in each gap with one preposition. 1. Shrimp can trigger ________ an allergic reaction. 2. Help yourself to any drink you like. All are ________ the house. 3. I wish you wouldn’t do that. It really ticks me _________. 4. Recent research bears_________ the idea that women are safer drivers than men. 5. Tom’s parents were highly critical _________ his friends. Your answer: 1. 2. 3. 4. 5. Part 4: Write the correct form of each bracketed word in the numbered space provided in the column in the right.

3

Your answers:


British workers are among the (1) ________ (HAPPY)in Europe and the time so many of them spend commuting has been identified as a significant (2) _______ (CONTRIBUTE)factor. People who spend more than two hours a day travelling to and from work report increased(3) ________ (IRRITABLE)with colleagues, less efficiency in their work, and more problems in their personal lives. Some more (4) _______ (LIGHT) employers are beginning to realize that traditional work patterns will have to change if they want to retain a contented work force. (5) ________ (FLEX) working hours have been introduced by a number of companies who claim that this has led to greater job satisfaction and a noticeable increase in(6) ________ (PRODUCT). Another, more(7) ________ (REVOLUTE), approach has become feasible as a result of developments in information technology and a small growing number of people are now working from home. For most of us, however, the daily (8) ________ (MORALE) tedium of commuting remains unavoidable. As we sit or stand on a crowded, and (9) ________ (VARY)late, bus or train, we can only dream of a more leisurely lifestyle devoid of the misery commuting (10) ________ (INEVITABLE)brings.

1........................... 2........................... 3........................... 4........................... 5. ……................. 6........................... 7........................... 8. ……................. 9........................... 10.........................

READING Part 1. Read the texts below and decide which answer best fits each space. SOUND ADVICE FOR LANGUAGE LEARNERS A recent issue of a language learning magazine has consulted a number of experts in the (1) ______ of second language acquisition. Their advice may prove invaluable for those (2) ______ a language course. One suggestion is that you (3) ______ whether you are likely to be successful at learning a language. Did you enjoy studying languages at school, for example? Do you have enough time to learn a language? The major (4) ______ will be your own time and effort. If proof of your level of proficiency is important you must make sure that the course on offer leads to a (5) ______ qualification. Also, be realistic in your (6) ______. If you don't set achievable aims you are more likely to give up. Do not be deceived (7) ______ thinking that the most expensive courses are the best. (8) ______ around to get the best possible value for money. You should also bear in mind that the quicker you learn a language the more quickly you forget it. Sandra Miller, a French teacher, tried to teach herself German by enrolling on a (9) ______ course. Already fluent in four languages and with a sound knowledge of teaching methodology her chances of (10) ______ progress were high. Three years on she remembers very little. She feels her biggest mistake was not to follow up her first experience. "I should have consolidated what I'd learn by continuing to study, even if it were by myself." 1. A. domain B. branch C. field D. area 2. A. wondering B. thinking C. looking D. considering 3. A. assess B. review C. balance D. survey 4. A. change B. cost C. price D. evaluation 5. A. recognized B. understood C. valued D. regarded 6. A. sights B. ends C. objects D. goals 7. A by B. about C. into D. in 8. A. Nose B. Push C. Run D. Shop 9. A. rapid B. crash C. quick D. fast 10. A. achieving B. doing C. gaining D. making 4


Your answer: 1. 2. 3. 4. 5. 6. 7. 8. 9. 10. Part 2. Read the text and think of the word which best fits each space. Use only ONE word in each space. Smart toys If your kids easily become bored while watching the television or listening to music, a smart toy may help to maintain their interest. The toy, which is controlled by signals hidden in the sound, will respond to the TV or dance around (1) ______ the music. Ian Hosking, (2) ______ work at Scientific Generics on adapting spread spectrum technology has (3) ______ to the development of the technique of hiding control signals in sound, claims that the toy is actually (4) ______simple. “It needs little more than the (5) ______ to decode signals and to respond to them� The idea of controlling devices with sound is (6) ______ new. Some early television remote controls emitted ultrasonic bleeps, but they were unreliable. Traffic noise could (7) ______ off the television, and the ultrasound (8) ______ to upset pets. In the new system, coded control signals are spread over a wide range of frequencies, but they are (9) ______ faint to be audible on a normal domestic sound system, (10) ______ avoiding problems of interference. Your answer: 1. 6.

2. 7.

3. 8.

4. 9.

5. 10.

Part 3. Read the extract taken from Darwin's book The Voyage of the Beagle then choose the best answer A, B, C or D to complete each statement. That large animals require a luxuriant vegetation, has been a general assumption which has passed from one work to another; but I do not hesitate to say that it is completely false, and that it has vitiated the reasoning of geologists on some points of great interest in the ancient history of the world. The prejudice has probably been derived from India, and the Indian islands, where troops of elephants, noble forests, and impenetrable jungles, are associated together in every one's mind. If, however, we refer to any work of travels through the southern parts of Africa, we shall find allusions in almost every page either to the desert character of the country, or to the numbers of large animals inhabiting it. The same thing is rendered evident by the many engravings which have been published of various parts of the interior. Dr. Andrew Smith, who has lately succeeded in passing the Tropic of Capricorn, informs me that, taking into consideration the whole of the southern part of Africa, there can be no doubt of its being a sterile country. On the southern coasts there are some fine forests, but with these exceptions, the traveller may pass for days together through open plains, covered by a poor and scanty vegetation. Now, if we look to the animals inhabiting these wide plains, we shall find their numbers extraordinarily great, and their bulk immense. We must enumerate the elephant, three species of rhinoceros, the hippopotamus, the giraffe, the boscaffer, two zebras, two gnus, and several antelopes even larger than these latter animals. It may be supposed that although the species are numerous, the individuals of each kind are few. By the kindness of Dr. Smith, I am enabled to show that the case is very different. He informs me, that in lat. 24', in one day's march with the bullock-wagons, he saw, without wandering to any great distance on either side, between one hundred and one hundred and fifty rhinoceroses - the same day he saw several herds of giraffes, amounting together to nearly a hundred. At the distance of a little more than one hour's march from their place of encampment on the previous night, his party actually killed at one spot eight hippopotamuses, and saw many more. In this same river there were likewise crocodiles. Of course it was a case quite extraordinary, to see so many great animals crowded together, but it evidently proves that they must exist in great numbers. Dr.Smith describes the country passed through that day, as 'being thinly covered 5


with grass, and bushes about four feet high, and still more thinly with mimosa-trees.' Besides these large animals, every one the least acquainted with the natural history of the Cape, has read of the herds of antelopes, which can be compared only with the flocks of migratory birds. The numbers indeed of the lion, panther, and hyena, and the multitude of birds of prey, plainly speak of the abundance of the smaller quadrupeds: one evening seven lions were counted at the same time prowling round Dr. Smith's encampment. As this able naturalist remarked to me, the carnage each day in Southern Africa must indeed be terrific! I confess it is truly surprising how such a number of animals can find support in a country producing so little food. The larger quadrupeds no doubt roam over wide tracts in search of it; and their food chiefly consists of underwood, which probably contains much nutriment in a small bulk. Dr.Smith also informs me that the vegetation has a rapid growth; no sooner is a part consumed, than its place is supplied by a fresh stock. There can be no doubt, however, that our ideas respecting the apparent amount of food necessary for the support of large quadrupeds are much exaggerated. The belief that where large quadrupeds exist, the vegetation must necessarily be luxuriant, is the more remarkable, because the converse is far from true. Mr.Burchell observed to me that when entering Brazil, nothing struck him more forcibly than the splendour of the South American vegetation contrasted with that of South Africa, together with the absence of all large quadrupeds. In his Travels, he has suggested that the comparison of the respective weights (if there were sufficient datA. of an equal number of the largest herbivorous quadrupeds of each country would be extremely curious. If we take on the one side, the elephants hippopotamus, giraffe, boscaffer, elan, five species of rhinoceros; and on the American side, two tapirs, the guanaco, three deer, the vicuna, peccari, capybara (after which we must choose from the monkeys to complete the number), and then place these two groups alongside each other it is not easy to conceive ranks more disproportionate in size. After the above facts, we are compelled to conclude, against anterior probability, that among the mammalia there exists no close relation between the bulk of the species, and the quantity of the vegetation, in the countries which they inhabit. (Adapted from: Voyage of the Beagle, Charles Darwin) 1. The author is primarily concerned with __________. A. discussing the relationship between the size of mammals and the nature of vegetation in their habitats B. contrasting ecological conditions in India and Africa C. proving the large animals do not require much food D. describing the size of animals in various parts of the world 2. According to the author, the ‘prejudice’ has lead to __________ . A. errors in the reasoning of biologists B. false ideas about animals in Africa C. doubt in the mind of the author D. incorrect assumptions on the part of geologists 3. The author uses information provided by Dr. Smith to __________. A. supply information on quality and quantity of plant life in South Africa B. indicate the presence of large numbers of animals C. give evidence of numbers of carnivorous animals D. A, B and C are correct 4. The flocks of migratory birds are mentioned to __________. A. describe an aspect of the fauna of South Africa B. indicate the abundance of wildlife C. contrast with the habits of the antelope D. suggest the size of antelope herds 5. The ‘carnage’ refers to the __________. A. number of animals killed by hunters B. number of prey animals killed by predators C. number of people killed by lions D. amount of food eaten by all species 6. To account for the ‘surprising’ number of animals in a ‘country producing so little food’, Darwin suggests all of the following as partial explanations except __________. A. food requirements have been overestimated B. rapid regrowth of plant material C. large area for animals to forage in D. mainly carnivorous animals 6


7.The author makes his point by reference to all of the following except ___________. A. historical documents B. published illustrations C. private communications D. recorded observations 8. Darwin quotes Burchell’s observations in order to ___________. A. describe a region of great splendor B. counter a popular misconception C. account for a curious situation D. illustrate a well-known phenomenon 9. Darwin apparently regards Dr. Smith as __________. A. reliable and imaginative B. observant and excitable C. intrepid and competent D. foolhardy and tiresome 10. ‘Anterior probability’ refers to ___________. A. what might have been expected B. ideas of earlier explorers C. likelihood based on data from India D. hypotheses of other scientists Your answers 1. 6.

2. 7.

3. 8.

4. 9.

5. 10.

Part 4. Dirty river but clean water Floods can occur in rivers when the flow rate exceeds the capacity of the river channel, particularly at bends or meanders in the waterway. Floods often cause damage to homes and businesses if they are in the natural flood plains of rivers. While riverine flood damage can be eliminated by moving away from rivers and other bodies of water, people have traditionally lived and worked by rivers because the land is usually flat and fertile and because rivers provide easy travel and access to commerce and industry. A FIRE and flood are two of humanity’s worst nightmares. People have, therefore, always sought to control them. Forest fires are snuffed out quickly. The flow of rivers is regulated by weirs and dams. At least, that is how it used to be. But foresters have learned that forests need fires to clear out the brush and even to get seeds to germinate. And a similar revelation is now dawning on hydrologists. Rivers 一 and the ecosystems they support — need floods. That is why a man-made torrent has been surging down the Grand Canyon. By Thursday March 6th it was running at full throttle, which was expected to be sustained for 60 hours. B Floods once raged through the canyon every year. Spring Snow from as far away as Wyoming would melt and swell the Colorado river to a flow that averaged around 1,500 cubic meters (50,000 cubic feet) a second. Every eight years or so, that figure rose to almost 3,000 cubic meters. These floods infused the river with sediment, carved its beaches and built its sandbars. C However, in the four decades since the building of the Glen Canyon dam, just upstream of the Grand Canyon, the only sediment that it has collected has come from tiny, undammed tributaries. Even that has not been much use as those tributaries are not powerful enough to distribute the sediment in an ecologically valuable way. D This lack of flooding has harmed local wildlife. The humpback chub, for example, thrived in the rustred waters of the Colorado. Recently, though, its population has crashed. At first sight, it looked as if the reason was that the chub were being eaten by trout introduced for sport fishing in the mid-20th century. But trout and chub co-existed until the Glen Canyon dam was built, so something else is going on. Steve Gloss, of the United States’ Geological Survey (USGS), reckons that the chub’s decline is the result of their losing their most valuable natural defense, the Colorado’s rusty sediment. The chub were well adapted to the poor visibility created by the thick, red water which gave the river its name, and depended on it to hide from predators. Without the cloudy water the chub became vulnerable. E And the chub are not alone. In the years since the Glen Canyon dam was built, several species have vanished altogether. These include the Colorado pike-minnow, the razorback sucker and the roundtail chub. 7


Meanwhile, aliens including fathead minnows, channel catfish and common carp, which would have been hard, put to survive in the savage waters of the undammed canyon, have moved in. F So flooding is the obvious answer. Unfortunately, it is easier said than done. Floods were sent down the Grand Canyon in 1996 and 2004 and the results were mixed. In 1996 the flood was allowed to go on too long. To start with, all seemed well. The floodwaters built up sandbanks and infused the river with sediment. Eventually, however, the continued flow washed most of the sediment out of the canyon. This problem was avoided in 2004 ,but unfortunately, on that occasion, the volume of sand available behind the dam was too low to rebuild the sandbanks. This time, the USGS is convinced that things will be better. The amount of sediment available is three times greater than it was in 2004. So if a flood is going to do some good, this is the time to unleash one. G Even so, it may turn out to be an empty gesture. At less than 1,200 cubic metres a second, this flood is smaller than even an average spring flood, let alone one of the mightier deluges of the past. Those glorious inundations moved massive quantities of sediment through the Grand Canyon, wiping the slate dirty, and making a muddy mess of silt and muck that would make modem river rafters cringe. Questions 1-6 Do the following statements agree with the information given in Reading Passage ? In boxes 1-6 on your answer sheet, write TRUE if the statement is true FALSE if the statement is false NOT GIVEN if the information is not given in the passage 1. Damage caused by fire is worse than that caused by flood. 2. The flood peaks at almost 1500 cubic meters every eight years. 3. Contribution of sediments delivered by tributaries has little impact. 4. Decreasing number of chubs is always caused by introducing of trout since mid-20th. 5. It seemed that the artificial flood in 1996 had achieved success partly at the very beginning. 6. In fact, the yield of artificial flood water is smaller than an average natural flood at present. Questions 7-10 Complete the summary below. Choose NO MORE THAN TWO WORDS from the passage for each answer. Write your answers in boxes 8-13 on your answer sheet. The Eco- Impact of the Canyon Dam Floods are peopled nightmare. In the past, canyon was raged by flood every year. The snow from far Wyoming would melt in the season of spring and caused a flood flow peak in Colorado river. In the four decades after people built the Glen Canyon dam, it only could gather7………………………. together from tiny, undammed tributaries. humpback chub population reduced, why? Then, several species disappeared including Colorado pike-minnow, 8 …………… and the round-tail chub. Meanwhile, some moved in such as fathead minnows, channel catfish and 9…………………The non-stopped flow leaded to the washing away of the sediment out of the canyon, which poses great threat to the chubs because it has poor 10………………… away from predators. In addition, the volume of sand available behind the dam was too tow to rebuild the bars and flooding became more serious. Your answer: 1. 2. 3. 4. 5. 6. 7. 8. 9. 10. 11. 12. 13.

Part 5. You are going to read a magazine article. Seven paragraphs have been removed from the extract. Choose from the paragraphs A-H the one which fits each gap (1-7). There is one extra paragraph which you do not need to use.

8


The Do-gooders The people who changed the morals of English society. In the last decades of the 18th century, the losers seriously outnumbered the winners. Those who were fortunate enough to occupy the upper levels of society, celebrated their good fortune by living a hedonistic life of gambling, parties and alcohol. It was their moral right, they felt, to exploit the weak and the poor. Few of them thought their lives should change, even fewer believed it could. 1 But the decisive turning point for moral reform was the French revolution. John Bowlder, a popular moralist of the time, blamed the destruction of French society on a moral crisis. Edmund Burke, a Whig statesman agreed. 'When your fountain is choked up and polluted,' he wrote, 'the stream will not run long or clear.' If the English society did not reform, ruin would surely follow 2 Englishmen were deeply afraid that the immorality of France would invade England. Taking advantage of this, Burke was able to gain considerable support by insisting that the French did not have the moral qualifications to be a civilised nation. He pronounced 'Better this island should be sunk to the bottom of the sea that than... it should not be a country of religion and morals.' 3 Sobering though these messages were, the aristocracy of the time was open to such reforms, not least due to fear. France's attempt to destroy their nobility did much to encourage the upper classes to examine and re-evaluate their own behaviour. Added to this was the arrival of French noble ĂŠmigrĂŠs to British shores. As these people were dependent on the charity of the British aristocracy, it became paramount to amend morals and suppress all vices in order to uphold the state. 4 Whether the vices of the rich and titled stopped or were merely cloaked is open to question. But it is clear that by the turn of the century, a more circumspect society had emerged. Styles of dress became more moderate, and the former adornments of swords, buckles and powdered hair were no longer seen. There was a profusion of moral didactic literature available. Public hangings ceased and riots became much rarer. 5 One such person was Thomas Wackley who in 1823 founded a medical journal called 'the Lancet'. At this time, Medicine was still a profession reserved for the rich, and access to knowledge was impossible for the common man. The Lancet shone a bright light on the questionable practices undertaken in medicine and particularly in surgery, and finally led to improved standards of care. 6 How though did changes at the top affect the people at the bottom of the societal hierarchy? Not all reformers concerned themselves which changes at the authoritative and governmental levels. Others concentrated on improving the lives and morals of the poor. In the midst of the industrial revolution, the poorest in society were in dire straits. Many lived in slums and sanitation was poor. No-one wanted the responsibility of improvement. 7 Could local authorities impose such measures today? Probably not. Even so, the legacy of the moral reform of the late 1800s and 1900s lives on today. Because of it, the British have come to expect a system which is competent, fair to all and free from corruption. Nowadays everyone has a right to a home, access to education, and protection at work and in hospital. This is all down to the men and women who did not just observe society's ills from a distance, but who dared to take steps to change it. Paragraphs 9


A But a moral makeover was on the horizon, and one of the first people to promote it was William Wilberforce, better known for his efforts in abolishing the slave trade. Writing to a friend, Lord Muncaster, he stated that 'the universal corruption and profligacy of the times...taking its rise amongst the rich and luxurious has now ... spread its destructive poison through the whole body of the people.' B But one woman, Octavia Hill, was willing to step up to the mark. Hill, despite serious opposition by the men who still dominated English society, succeeded in opening a number of housing facilities for the poor. But, recognising the weaknesses of a charity-dependent culture, Hill enforced high moral standards, strict measures in hygiene and cleanliness upon her tenants, and, in order to promote a culture of industry, made them work for any financial handouts. C At first, moralists did not look for some tangible end to moral behaviour. They concerned themselves with the spiritual salvation of the rich and titled members of society, believing that the moral tone set by the higher ranks would influence the lower orders. For example, Samuel Parr, preaching at London's St Paul's Cathedral, said 'If the rich man...abandons himself to sloth and all the vices which sloth generates, he corrupts by his example. He permits...his immediate attendants to be, like him, idle and profligate.' D In time, the favour for improved morals strayed beyond personal behaviour and towards a new governance. People called for a tightening of existing laws which had formerly been enforced only laxly. Gambling, duelling, swearing, prostitution, pornography and adultery laws were more strictly upheld to the extent that several fashionable ladies were fined fifty pounds each for gambling in a private residence. E So far, however, circumspection in the upper classes had done little to improve the lives of those in the lower classes. But that was to change. Against a backdrop of the moral high ground, faults in the system started to stand out. One by one, people started to question the morality of those in authority. F The attitudes of the upper classes became increasingly critical during the latter part of the eighteenth century. In 1768, the Lord of the Treasury was perfectly at ease to introduce his mistress to the Queen, but a generation later, such behaviour would have been unacceptable. Such attitudes are also seen in the diaries of Samuel Pepys, who, in 1793 rambles without criticism about his peer's many mistresses. A few years later, his tone had become infinitely more critical. G Similar developments occurred in the Civil Service. Civil servants were generally employed as a result of nepotism or acquaintance, and more often than not took advantage of their power to provide for themselves at the expense of the public. Charles Trevelyan, an official at the London Treasury, realised the weaknesses in the system and proposed that all civil servants were employed as a result of entrance examinations, thus creating a system which was politically independent and consisted of people who were genuinely able to do the job. H These prophecies roused a little agitation when first published in 1790. But it was the events in 179293 which shocked England into action. Over in France, insurrection had led to war and massacre. The King and Queen had been tried and executed. France was now regarded as completely immoral and uncivilized, a country where vice and irreligion reigned. Your answer: 1.

2.

3.

4.

5.

WRITING 10

6.

7.


Part 1.Read the following passage and use your own words to summarize it. Your summary should be about 80words long. Influenza, or 'the flu' is an infection of the lungs and the surrounding areas. This infection of the respiratory tract is caused by the influenza virus. The virus usually spreads during the winter in temperate climates. When many people catch the flu at the same time, the situation is called a flu epidemic. The proteins that coat the flu virus change constantly. As a result, new strains of the flu virus circulate every few years. In some countries, people at high risk are encouraged to get a flu vaccination every year. Some of the people at risk are those over 65 years, children with heart or lung conditions and health care workers. People with flu infections feel as if they have a cold, but the signs and symptoms are usually more severe. Body areas other than the respiratory tract may be infected. Signs and symptoms include weakness, chills, fatigue, muscle aches, headache, fever, running nose and cough. The signs and symptoms could last for a week to ten days. The influenza virus is spread largely through the air. A typical situation is where one person infected with the flu coughs or sneezes when in close proximity with another person. Droplets of the virus, suspended in the air, are breathed in by the other person. Once the virus lands on the lining of the nose, throat or other body areas related to breathing, it reproduces rapidly. Usually the flu goes away with a rest, drinking plenty of fluids and taking mild pain medication. Health care providers may prescribe certain medication for people who are at high risk. When symptoms do not go away after seven to ten days, there is difficulty in breathing or persistent high temperatures, a health care provider should be consulted. …………………………………………………………………………………………………….………… ………………………………………………………………………………………….…………………… ……………………………………………………………………………….……………………………… …………………………………………………………………….………………………………………… ………………………………………………………….…………………………………………………… ……………………………………………….……………………………………………………………… …………………………………….………………………………………………………………………… ………………………….…………………………………………………………………………………… ……………….……………………………………………………………………………………………… …….…………………………………………………………………………………………………….…… Part 2. The diagrams below show the students’ family economic background, the total number of students at university in the UK, the government spending on each student. Write a report for university lecturer describing the information below.

11


Percentage of students by family economic background in 1991 0% High income [PERCENTAGE]

Low income [PERCENTAGE]

Middle income [PERCENTAGE]

Total number of students at university in the UK 10000 3000000 2000000

Government spending on each student (pound)

1000000 0

0 1991 1993 1995 1997 1999 2001

1991 1992 1993 1994 1995 1996 1997 1998 1999 2000 2001

………………………………………………………………………………………….…………………… ……………………………………………………………………………….……………………………… …………………………………………………………………….………………………………………… ………………………………………………………….…………………………………………………… ……………………………………………….……………………………………………………………… …………………………………….………………………………………………………………………… ………………………….…………………………………………………………………………………… ……………….……………………………………………………………………………………………… …….…………………………………………………………………………………………………….…… ……………………………………………………………………………………………….……………… …………………………………………………………………………………….………………………… ………………………………………………………………………….…………………………………… ……………………………………………………………….……………………………………………… …………………………………………………….………………………………………………………… ………………………………………….…………………………………………………………………… 12


……………………………….……………………………………………………………………………… …………………….………………………………………………………………………………………… ………….……………………………………………………………………………………………………. …………………………………………………………………………………………………….………… ………………………………………………………………………………………….……………………

Part 3. Education is a lifelong task. To what extent do you agree or disagree? …………………………………………………………………………………………………….………… ………………………………………………………………………………………….…………………… ……………………………………………………………………………….……………………………… …………………………………………………………………….………………………………………… ………………………………………………………….…………………………………………………… ……………………………………………….……………………………………………………………… …………………………………….………………………………………………………………………… ………………………….…………………………………………………………………………………… ……………….……………………………………………………………………………………………… …….…………………………………………………………………………………………………….…… ……………………………………………………………………………………………….……………… …………………………………………………………………………………….………………………… ………………………………………………………………………….…………………………………… ……………………………………………………………….……………………………………………… …………………………………………………….………………………………………………………… ………………………………………….…………………………………………………………………… ……………………………….……………………………………………………………………………… …………………….………………………………………………………………………………………… ………….……………………………………………………………………………………………………. …………………………………………………………………………………………………….………… ………………………………………………………………………………………….…………………… ……………………………………………………………………………….……………………………… …………………………………………………………………….………………………………………… ………………………………………………………….…………………………………………………… ……………………………………………….……………………………………………………………… 13


…………………………………….………………………………………………………………………… ………………………….…………………………………………………………………………………… ……………….……………………………………………………………………………………………… …….…………………………………………………………………………………………………….…… ……………………………………………………………………………………………….……………… …………………………………………………………………………………….………………………… ………………………………………………………………………….…………………………………… ……………………………………………………………….……………………………………………… …………………………………………………….………………………………………………………… ………………………………………….…………………………………………………………………… …………………………………………………………………………………………………….………… ………………………………………………………………………………………….…………………… ……………………………………………………………………………….……………………………… …………………………………………………………………….………………………………………… ………………………………………………………….…………………………………………………… ……………………………………………….……………………………………………………………… …………………………………….………………………………………………………………………… ………………………….…………………………………………………………………………………… ……………….……………………………………………………………………………………………… …….…………………………………………………………………………………………………….…… ……………………………………………………………………………………………….……………… …………………………………………………………………………………….………………………… ………………………………………………………………………….…………………………………… ……………………………………………………………….……………………………………………… …………………………………………………….………………………………………………………… ………………………………………….…………………………………………………………………… ……………………………….……………………………………………………………………………… …………………….………………………………………………………………………………………… ………….……………………………………………………………………………………………………. …………………………………………………………………………………………………….………… ………………………………………………………………………………………….…………………… ……………………………………………………………………………….……………………………… …………………………………………………………………….………………………………………… 14


………………………………………………………….…………………………………………………… ……………………………………………….……………………………………………………………… …………………………………….………………………………………………………………………… ………………………….…………………………………………………………………………………… ……………….……………………………………………………………………………………………… …….…………………………………………………………………………………………………….…… ……………………………………………………………………………………………….………………

15


TRƯỜNG THPT CHUYÊN NGUYỄN TẤT THÀNH YÊN BÁI KỲ THI HỌC SINH GIỎI CÁC TRƯỜNG THPT CHUYÊN KHU VỰC DUYÊN HẢI VÀ ĐỒNG BẰNG BẮC BỘ HƯỚNG DẪN CHẤM ĐỀ XUẤT MÔN: TIẾNG ANH – KHỐI 11 LISTENING Part 1 1. C Part2 1. F Part 3 1. (in) groups Part 4 1. year roundups 6. global events

2. B

3. A

2. T

4.A

3. T

2. every 2 days 2. The most tweeted 7. High profiled celebrity

5.C 4. F

3. 2 weeks

5. F

4. confident

3. Black Panther 8. Rehab

5. education system

4. #time’sup

5. In my feeling

9. Newly appointed Duchess

10. Supreme Court

LEXICO-GRAMMAR Part 1 1.A Part 2 1. in on Part 3.

2.D

3.C

4.C

2. kept set

1. off Part 4 1. unhappiest 6. productivity READING Part 1 1. C 6. D Part 2 1.To 6. Not / nothing Part 3. 1. A 6. D Part 4 1. NOT GIVEN 6. TRUE

5.D

6.A

3. started will start

7.B

8.D

9. B

10. C

4. value valuable

5. by with

2. on

3. off

4. out

5. of

2. contributory 7. revolutionary

3. irritability 8. demoralizing/ demoralising

4. enlightened 9. invariably

5. Flexible 10. inevitably

2. D 7. C

3. A 8. D

4. B 9. B

5. A 10. D

2. Whose 7. Switch / turn

3. Led / contributed 8. Used / tended

4. Quite / very / perfectly / 9. Too

5. Ability / capability 10. Thereby / thus

2. D 7. A

3. D 8. B

4. D 9. C

5. B 10. A

4. FALSE 9. Common carp

5. TRUE 10. Visibility

2. FALSE 7. Sediment

3. NOT GIVEN 8. Razorback sucker


Part 5. 1. F

2. H

3. C

4. D

5. E

6. G

7. B

WRITING Part 1. Main points Paragraph 1: What is influenza? A virus infection of the respiratory tract. Paragraph 2: Different strains of influenza are caused by protein coating changes. Paragraph 3: Signs and symptoms of influenza: same as for a cold but could be more serious. Paragraph 4: How influenza is spread: breathing in droplets in the air when an infected person coughs or sneezes Paragraph 5: How influenza is treated: rest, plenty of fluids, mild medication, consult doctor if symptoms persist or worsen Answer Influenza is an infection of the respiratory system. It is caused by a virus. When the protein coating of the virus changes new strains appear. The signs and symptoms are similar to that of a cold, but often more severe. When an infected person cough is or sneezes, droplets of the virus float in the air. Another person gets infected by breathing iii these droplets. The virus multiplies quickly once it settles on body areas related to breathing. Rest, drinking plenty of fluids and mild medication usually get rid of influenza. However, people with prolonged or more serious symptoms have to be treated by doctors. (104 words) Part 2. The charts illustrate the number of undergraduates, government spending on each student from 1991 to 2001, and compare the percentages of students of different family economic backgrounds in 1991. Figures were taken in the UK. Overall, in 1991 the majority of undergraduates came from middle-income families. Also, in contrast to the increased number of university students, government spending became more and more limited toward the end of the period. The proportion of university students whose family financial background fell into the middle category was just over 60%, approximately twice as much as that of students from high-income families. Meanwhile, only 8% of UK’s undergraduates were from families with low income in 1991. There was a moderate rise in the number of undergraduates from over 1000000 in 1991 to more than 2000000 in 1999. This figure then slightly declined to precisely 2000000 after 2 years. Despite the overall increase in students’ number, government budget allocated for supporting those students fell exponentially from 6500 pounds to around 4700 pounds, a decrease of 1200 pounds in only 5 years. After that, the figure experienced almost no change until the end of the period.


Part 1. You will hear a podcast on Camber’s Theme Park. Presenter: Welcome to Camber’s Park podcast. In the next few minutes, I’ll tell you alittle about the park and the amazing things we have to offer. We like to think that Camber’s offers more than other theme parks. Like them, we have avariety of exciting rides for people of all ages, but Camber’s also places strongemphasis on the educational experience for its visitors ... not boring facts but lots ofinteractive exhibits. Although it’s mainly an outdoor experience, we do have some indooractivities if the weather gets too dreadful. The park’s got a lovely well established feel, set in eighty acres of beautiful countrysideabout three miles south of the tourist resort of Dulchester. The park was set up innineteen ninety seven by the Camber family but then taken over by new owners intwo thousand and four, who have maintained the original vision of the Cambers. It has lotsof old trees, hundreds of flower beds and a gorgeous lake. Camber’s has over forty- five different rides, exhibits and arcades. All but one ofthese is free once you have paid your entrance fee (we charge a small fee for our newestride, to reduce the length of the queues). You don’t pay anything for parking. A familyticket - for a family of four - works out at about eight pounds per person, which is amazingvalue. Full details of current prices are shown on our website, along with full details ofrides, etc., and directions for getting to us. We also have a number of special offers. For example, if you live locally, why not join ourAdventurers’ Club, which entitles you fifty percent off ticket prices all year round, and aspecial ‘lane’ for all rides and exhibits which means you don’t have to wait to get intoany part of the park. See the Offers tab on the website. We’ve recently added a number of new exhibits to the park, and we’re particularly proud ofour Future Farm Zone, which houses over twenty different species of animals, fromchipmunks to dairy cows. The emphasis is on getting near to the animals- all ofthem can be petted and you can buy food for feeding the animals. Many of our youngervisitors say that this is the high point of their visit! And speaking of food, don’t let the animals have all the fun. We have a total of sevendifferent catering outlets on the site. We’re open ten to five thirty all year round and colddrinks and snacks can be bought at any time during opening hours. And hot food isavailable most of the day in the Hungry Horse cafe - from eleven until five - just halfan hour before closing time. Presenter: Now we want all our visitors to have an exciting time when they come to thepark but our first priority must be safety. Parents and guardians know their children’sbehaviour and capabilities. But here at the park we have set certain conditions for each ofthe rides to ensure that all visitors get the maximum enjoyment out of the experience andfeel secure at all times. There are four major rides at the park. Our newest ride is the RiverAdventure which is designed to reproduce the experience of white-water rafting. Noamount of protective clothing would make any difference so only go on this ride if you’reprepared to get wet! Children under eight can go on this ride, but all under sixteensmust have an adult with them. Not all of our rides are designed for thrills and spills. Our Jungle Jim rollercoaster is agentler version of the classic loop the loop, specially created for whole family enjoyment -from the smallest children to elderly grandparents, suitable for all levels of disability andhealth conditions. Carriages have comfortable seating for up to eight people, withsafety belts for each passenger which must be worn at all times. Sit back and enjoythe scenery! One of the best established and most popular of Camber’s rides is the massive SwoopSlide. Whizz down the polished vertical slide nine metres in height and scream to yourheart’s content. There are no age or height restrictions. Be careful though - you musthave on long trousers so you won’t get any speed burns!


And then there’s the famous Zip Go-kart stadium with sixteen carts: eight for single driversand eight for kids preferring to ride along with mum, dad or carer. Take part in high-speedraces in our specially designed Formula One-style karts - but no bumping other karts,please. All riders must be above one point two metres because they have to be ableto reach the pedals ... even in the shared karts. Full details of all safety features are available on our website atwww.Camberspark.com.So come and make a day of it at Camber’s Theme Park! Part 2. Jenny: Good evening. My name is Jenny Fowler and I would like to welcome all our listeners to Net News, your weekly program dealing with all matters online. Tonight we are fortunate to be joined by David Akris, head of the Internet security company WebLock. Now, this evening, we are going to be talking specifically about matters related to e-mail such as spam and the latest fear to grip the online community, the strange phenomenon which goes by the name of phishing. David, welcome to Net News. David: Good evening Jenny. Jenny: Right, what I would really like to know first from you David is this: how on earth can I reduce the amount of spam mail I find every morning in my inbox? I think things are getting absolutely ridiculous. I mean, this morning there must have been about 30 to 40 e-mails covering everything from having won the lottery to purchasing things I would rather not mention on this show. What can a normal e-mail user such as myself do to combat this ever-growing problem? David: Now there are a couple of strategies that we can employ to reduce this admittedly annoying problem. First of all, you have to be really guarded about where you leave your e-mail address. Of course, you give it to all your friends and you give it to contacts you have through your workplace, although I would also add as a strong recommendation that you have separate personal and work e-mail accounts. But what you must not do is to navigate around the Internet leaving your e-mail address for everyone and his brother to find. Because, believe me, there is no shortage of unscrupulous individuals out there who will harvest your e-mail address among the thousands of others either for their own personal gain or to sell to third parties. Jenny: Wow, I didn't realize it was so simple. David: It's actually worse than that. There are little software programs called "spiders" that trawl around the Internet on their own looking for e-mail addresses, so your e-mail address, left innocently on that cookery message board, doesn't even need to be seen by human eyes to be gathered. If any of your listeners are using what we call throwaway e-mail addresses, you know, provided by companies such as Yahoo or Hotmail, then I would recommend creating a new e-mail account and starting over. Jenny: That sounds like worthwhile advice, David. You have spoken to us about how we can prevent our e-mail address falling into the wrong hands. What can we do to alleviate the spam problem if this has already occurred, beyond creating a new account, which may not be practical for all of our listeners? Is there anything we can do to fight back against the spammers? David: Sure. There are a variety of handy antispam tools out there on the market. Your ISP, that is, your Internet service provider, will most likely have one of the main antispam filters active on all their users' accounts. Jenny: Hmmm, tell me about these filters. I think I need to get one.


David: Without going into too much technical detail, spam filter software will analyze all incoming messages and flag, divert or delete e-mails that it considers to be spam. This may be because the message contains certain words - I think we all know what types of words I'm talking about here. Or the e-mail may have other qualities which marks it out to be spam in all likelihood, such as large blue or red font, lack of return address, too much capitalization and a whole list of other considerations. Jenny: But doesn't that mean we have to ask our friends never to write e-mails to us in blue? I would be very concerned about legitimate e-mails being deleted ... David: Oh, absolutely! An antispam filter will only flag an e-mail message as spam if SEVERAL of these factors are found to be present in a particular message. If your aunt Cathy chooses to write to you in blue, but the message is otherwise perfectly normal, it will not be considered spam. These filters are great because users are often given control over the sensitivity level, but that's not true if you use one of the huge national ISPs such as AOL. Jenny: We've had a call from Margaret in Boston wanting to know if these antispam filters can also help in preventing her computer from becoming infected by a computer virus. Now, David, these are two different issues I believe, aren't they? David: Yes, yes. Many computer users do in fact manage to infect their PCs by opening attachments in e-mails but these could easily come from people they know who have unwittingly sent them infected files. The best advice I can give in the area of viruses is not to open any attachments that arrive unexpectedly ... and of course to always have an up to date antivirus program installed on your computer. Jenny: We now move on to an issue we mentioned at the beginning of the program. Phishing. What is it and how can we best protect ourselves from it? David: First of all, we should explain that even though the pronunciation is the same, this variety of phishing is spelt with a "PH" and not an "F". Right, phishing is quite a recent phenomenon but can be very damaging if users are not very aware of the dangers that exist. If you get an e-mail that claims to be from, let's say, your bank and asks you to click on a link, visit a site and enter information such as your account number, password, or username, you can be certain that you have been a victim of phishing. Jenny: Does this only affect bank customers, David? David: Not at all. Any service which requires online access and a password can be targeted so online stores or auction sites have also fallen foul of this. The good news is that avoiding phishing is as simple as you could imagine - don't ever, and I mean never, reply to any e-mail that asks for personal information of this type. The authentic sites would never request this information from you via e-mail so you can be certain that any e-mail such as this is a fraudulent attempt to gain sensitive information from you. Jenny: David, it's been very informative to have you here speaking to us today. A little understanding of these issues can help us to see the world of e-mail and the internet isn't so sinister after all. Please come back and see us soon, won't you? David: I'd be delighted. Thanks very much for inviting me to speak to your listeners. Part 3 Part 3. For questions 11 – 15, answer the questions below.


Write NO MORE THAN THREE WORDS AND/OR A NUMBER for each answer. Paul:

And how was your timetable? Was it a very busy year?

Kira: Very, very busy. They make you work very hard. Apart from lectures, we had practical sessions in a lot of subjects. We did these in small groups. I had to go and work four hours every week in a community pharmacy. Actually, I enjoyed this very much – meeting new people all the time. Then in second semester, we had to get experience in hospital dispensaries, so every second day we went to one of the big hospitals and worked there. And on top of all that we had our assignments, which took me a lot of time. Oh, I nealy forgot, between first and second semesters, we had to work full-time for two weeks in a hospital. Paul: That does sound a very heavy year. So are you pleased now that you did it? Do you feel some sense of achievement? Kira:

Yeah, I do feel much more confident, which I suppose is the most important thing.

Paul:

And have you got any recommendations for people who are studying from overseas?

Kira: Well, I suppose they need very good English. It would be much better if they spent more time learning English before they enter the university, because you can be in a big trouble if you don’t understand what people are saying and you haven’t got time to translate. Paul:

Anything else?

Kira:

Well, as I said before, the biggest problem for me was a lack of familiarity with the education system here.

Paul:

It sounds as if it was a real challenge. Congratulations, Kira.

Kira:

Thanks, Paul.



KỲ THI CHỌN HỌC SINH GIỎI ĐỀ ĐỀ XUẤT

KHU VỰC DUYÊN HẢI & ĐỒNG BẰNG BẮC BỘ NĂM HỌC 2018 - 2019

THPT Chuyên Thái Bình (Đề thi gồm 20 trang)

ĐỀ THI MÔN: ANH VĂN - KHỐI 11 Thời gian làm bài: 180 phút

I. LISTENING: HƯỚNG DẪN PHẦN THI NGHE HIỂU Bài nghe gồm 3 phần, mỗi phần được nghe 2 lần, mỗi lần cách nhau 10 giây, mở đầu và kết thúc mỗi phần nghe có tín hiệu. Thí sinh có 3 phút để hoàn chỉnh bài nghe. Mọi hướng dẫn cho thí sinh (bằng tiếng Anh) đã có trong bài nghe. Part 1: You will hear an interview with an author called Rachel White. For questions 1 – 5, choose the answer A, B, C or D which fits best according to what you hear. 1. What does Rachel particularly recall about her school days? A She was only really interested in the subject of literature. B She was aware that she was brighter than her classmates C She was given preferential treatment by a certain teacher. D She disliked being considered hard working by her peers. 2. Which of the following added to Rachel's negative feelings about her essay? A It was not her own work. B Her classmates were critical of it. C Some of the content was misleading. D There was an insufficient amount of material. 3. According to Rachel, what attitude do many people have towards Jane Austen 's books? A They lose interest in them at an early age. B They read them because they feel they should. C They believe they suit a certain type of personality. 1


D They feel they should be read in certain situations. 4. Why did Rachel write the essay “Literature and the Young Mind”? A. in order to express a commonly-held view B. in order to encourage young people to read literature C. because she wanted to express her gratitude to writers like Austen D. because she wanted to recommend certain writers to young readers 5. When discussing her own writing, Rachel highlights its A. contemporary relevance.

B. emotional content.

C. standard development.

D. essential complexity.

Part 2: Listen to the conversation and decide whether these statements are true T or false F. 6. The film speaker A likes is a romance based on a book. 7. In the film that speaker B likes, Javier Bardem is really an anti-hero. 8. In Pride and Prejudice, Elizabeth thinks Mr Darcy is obstinate. 9. Speaker D thinks that in The Matrix, Laurence Fishburne acts very badly. 10. “Let the right one in” is a soppy film. Part 3: Listen to the following recording and answer the following questions. WRITE NO MORE THAN THREE WORDS for each answer. 11. What part of the earth will the company deliver Internet access? ________________________________________________ 12. How did the company CEO feel about the project? ______________________________________________ 13. What will the company connect besides boats and planes? _____________________________________________ 14. What is China currently testing? _________________________________________ 15. What is Russia worried the network might be used for? _______________________________________________ Part 4: For questions 16 - 25, listen to a report on the Natural phenomena that science 2


cannot explain and supply the blanks with the missing information. Write NO MORE THAN THREE WORDS taken from the recording for each answer in the space provided. There exist a great many _____________(16) things in the world such as animals raining from the sky. Mammatus clouds – which means "mammary cloud", is a cellular ___________(17) hanging underneath the base of a cloud. Scientists have been studying the Great Pyramid of Giza and have found strange ___________ (18) that have no explanation. Doctors found something ___________(19): the Oakville Blobs contained human ___________________(20). Australia is the home of unusual ____________(21). Lake Hillier is one of them located on the Middle Island which still surprises and ___________(22) scientists. Socotra Island has been called the most (23) ______________ place on Earth, and a “lost world” because the heat and drought conditions of the island have produced around 800 rare species of (24) ________________ . Mount Kelimutu has three (25) ______________ that vary colors from each other at different times, making them surreal and exciting to be discovered. II: Grammar& Vocabulary Part 1: Choose the best option to complete each of the following questions. 26. I’m sure there’s a definite __________ of envy in her nasty comments about you. A. factor

B. ingredient

C. component

D. element

27. In the ___________ of just two days, her whole life changed. A. interval

B. space

C. spell

D. duration

28. The thieves took ___________ when they heard a police car approaching. A. retreat

B. flight

C. escape

D. getaway

29. Afterwards, when I ___________ on the events of that day, I could hardly believe what had happened. A. contemplated B. reviewed

C. reflected

D. weighed

30. There was a huge ___________ of applause when the star of the show appeared. A. bout

B. stint

C. round 3

D. spate


31. As he accepted the award, his voice ___________ with emotion. A. quivered

B. flinched

C. cringed

D. winced

32. Rose has always had a ___________ interest in matters to do with the environment. A. fierce

B. sharp

C. grave

D. keen

33. By the ___________ of it, the economy will improve over the next few months. A. face

B. impression

C. evidence

D. look

34. Tim and Alan have never got on well and there is a lot of

___________ feeling

between them. A. cross

B. adverse

C. ill

D. vile

35. Peter has now arrived late for work three days in a ___________ . A. line

B. sequence

C. series

D. row

Part 2: The passage below contains 5 mistakes. Underline the mistakes and write their correct forms in the space provided in the column on the right. Our little boy Sam goes to play at the next-door farm, and sometimes we collect him at the end of the meal. 36. Whichever the weather, and however far from the farmhouse ___________________ they are working, our neighbours come home in midday to 37. their dinner and sit down at the family table. The meal is hot, ___________________ exhibited the staple diet of the English – roast meat, root 38. vegetables, potatoes in gravy, and afterwards a fruit pie. If the ___________________ children are home, they too participate, as do grandparents, 39. ___________________

siblings, nieces and nephews.

These meals are remarkable for their silence, which is a 40. peculiar contented, sociable silence, quite distinct from the ___________________ silence of the lone commuter eating pizza on the tram. It is lain like a cloth across the table, and provides a soft, clean background to the gentle sound of eating. Above this silence, the members of the family communicate in wordless ways helping the children for food, passing the ketchup, grunting and nodding when the pudding appears. This speechless 4


conversation includes the newborn and the senile, and binds the whole family in a web of mutual dependence. Isolation is overcome, and anxiety stifled in the small, including gestures of the table. Part 3: Fill in the gaps with suitables particles. 41. I’m afraid that your party doesn’t quite tie _____ ______ our arrangements. 42. I know you are unhappy, but don’t take it _____ ______ me. 43. The president warned that terrorists will be hunted _____ . 44. Have you settled _____ a name for the baby yet? 45. We were set _____ by a a gang of hooligans. Part 4: Write the correct form of each bracketed word in the numbdered space provided in the column on the right. Perhaps the most vivid illustration of our gift for recognition is the 46. ........................... magic of caricature—the fact that the sparest cartoon of a familiar

47.............................

face, even a single line dashed off in two seconds, can be identified by our brains in an instant. It’s often said that a good caricature looks 48. .......................... more like a person than the person himself. As it happens, this 49. .......................... notion, 46. ______ (INTUITION) though it may sound, is actually 50. ........................... supported by research. In the field of vision science, there’s even a term for this seeming paradox—the caricature effect—a phrase that 51. ........................... hints at how our brains 47. ______ (PERCEPTION) faces as much as perceive them. Human faces are all built pretty much the same: two eyes above a nose that’s above a mouth, the features 48. ______ (VARIETY) from person to person generally by mere millimeters. So what our brains

52. ........................... 53. ........................... 54.

look for, according to vision scientists, are the 49. ______ (LIE) ........................... features—those 50. ______ (CHARACTER) that deviate most from 5


the ideal face we carry around in our heads, the running average of 55. ........................... every visage we’ve ever seen. We code each new face we encounter not in absolute terms but in the several ways it differs 51. ______ (MARK)

from the mean. In other words, to beat what vision

scientists call the 52. ______ (HOMOGENEOUS) problem, we 53. ______ (ACCENT) what’s most important for recognition and largely ignore what isn’t. Our perception fixates on the 54. ______ (TURN) nose, rendering it more porcine, the sunken eyes or the 55. ______ (FLESH)

cheeks, making them loom larger. To better

identify and remember people, we turn them into caricatures. . III. READING. Part 1: Choose the best option to fill in each blank. No matter how serious the sporting event, once an animal becomes (56) ____ , it will almost certainly steal the (57) ____. In November 1985, a football match between Newcastle Town and Chell Heath (58) ____ an unexpected turn after a terrier dog called Susie decided to make a contribution to the game. Newcastle Town were leading 1-0. A Chell Heath player was (59) ____ a great chance of (60) ____ , with only the goalkeeper standing between him and the goal, but the shot (61) ____ wide. At that moment, Susie the dog came sprinting up the pitch, leaped up and headed the ball past the goalkeeper and neatly into the net. To the amazement of the crowd and (62) ____ disbelief of the Newcastle players, the referee (63) ____ a goal. Fortunately, Newcastle Town went on to win by three goals to two, as (64) ____ speaking the goal shouldn't have been allowed. Sadly. before local football scouts had a chance to (65) ____ her up and turn her into a star, Susie had disappeared back into the crowd, never to be seen at the stadium again. 56. A. absorbed

B. involved

C. obsessed

D. averted

57. A highlight

B limelight

C footlight

D spotlight

58 A took

B made

C got

D gave

59 A out for

B up against

C away on

D in with

6


60. A. equalising

B scoring

C. dedeating

D. forwarding

61. A. came

B. went

C. got

D. became

62 A entire

B whole

C utter

D full

63. A. realised

B. disallowed

C. awarded

D. conceded

64 A truly

B rightly

C precisely

D strictly

65 A sign

B enroll

C join

D enlist

Part 2: Fill in each blank in the following passage with ONE suitable word. It is nearly impossible in our post-industrial society to conceive of a world without wheels. From clocks to huge machinery and from cars to computer disks, (1) _______ employs cogs, wheels or other types of cylindrical components that spin on an axis. (2) _______ the wheel took a relatively long time to be invented and several civilizations reached a relatively high level of technological sophistication (3) _______ it. The most likely explanation is that neither terrain nor climate suited the wheel. Until 10,000 BC, much of the world was (4) _______ the grip of the last vestiges of the Ice Age. (5) _______ was not under ice sheet was covered by desert, jungle or bog - conditions obviously unsuited for something like the wheel. Most experts agree that the wheel evolved from the fact that Neolithic man was familiar with moving heavy objects by putting a roller, such as a tree trunk, under the load. (6) _______ techniques were used to move the huge stone blocks to build the pyramids around 2980 BC and probably Stonehenge, which dates (7) _______ to around 2000 BC. (8) _______ technique for moving large, heavy objects was to place them on sledges and to put the sledges on rollers. In time, it is likely that the sledge wore grooves into the rollers (9) _______ the result that ancient man had a ratio – a small turn of the inner edge of the worn groove generated a larger turn of the outer edge of the roller. The next (10) _______ final step in the invention of the wheel was to reduce the weight of the roller by cutting away the wood between the grooves, in thhis way creating an axle with a wheel at each end. At last man could better indulge his passions for travel, speed and movement. Part 3: Read the following passage and choose the best answer to each question. Smart Energy 7


The next few decades will see great changes in the way energy is supplied and used. In some major oil producing nations, 'peak oil' has already been reached, and there are increasing fears of global warming. Consequently, many countries are focusing on the switch to a low carbon economy. This transition will lead to major changes in the supply and use of electricity. [A] Firstly, there will be an increase in overall demand, as consumers switch from oil and gas to electricity to power their homes and vehicles. [B] Secondly, there will be an increase in power generation, not only in terms of how much is generated, but also how it is generated, as there is growing electricity generation from renewable sources. [C] To meet these challenges, countries are investing in Smart Grid technology. [D] This system aims to provide the electricity industry with a better understanding of power generation and demand, and to use this information to create a more efficient power network. Smart Grid technology basically involves the application of a computer system to the electricity network. The computer system can be used to collect information about supply and demand and improve engineer's ability to manage the system. With better information about electricity demand, the network will be able to increase the amount of electricity delivered per unit generated, leading to potential reductions in fuel needs and carbon emissions. Moreover, the computer system will assist in reducing operational and maintenance costs. Smart Grid technology offers benefits to the consumer too. They will be able to collect real-time information on their energy use for each appliance. Varying tariffs throughout the day will give customers the incentive to use appliances at times when supply greatly exceeds demand, leading to great reductions in bills. For example, they may use their washing machines at night. Smart meters can also be connected to the internet or telephone

system,

allowing

customers

to

switch

appliances

on

or

off

remotely. Furthermore, if houses are fitted with the apparatus to generate their own power, appliances can be set to run directly from on-site power source, and any excess can be sold to the grid. With these changes comes a range of challenges. The first involves managing the supply and demand. Sources of renewable energy, such as wind, wave and solar, are 8


notoriously unpredictable, and nuclear power, which is also set to increase as nations switch to alternative energy sources, is inflexible. With oil and gas, it is relatively simple to

increase the supply of energy to match the increasing demand during peak times of the day or year. With alternative sources, this is far more difficult, and may lead to blackouts or system collapse. Potential solutions include investigating new and efficient ways to store

energy and encouraging consumers to use electricity at off-peak times. A second problem is the fact that many renewable power generation sources are located in remote areas, such as windy uplands and coastal regions, where there is currently

a lack of electrical infrastructure. New infrastructures therefore must be built. Thankfully, with improved smart technology, this can be done more efficiently by reducing the reinforcement or construction costs.

Although Smart Technology is still in its infancy, pilot schemes to promote and test it are already underway. Consumers are currently testing the new smart meters which can be

used in their homes to manage electricity use. There are also a number of demonstrations being planned to show how the smart technology could practically work, and trials are in place to test the new electrical infrastructure. It is likely that technology will be added in 'layers', starting with 'quick win' methods which will provide initial carbon savings, to be

followed by more advanced systems at a later date. Cities are prime candidates for investment into smart energy, due to the high population density and high energy use. It is here where Smart Technology is likely to be promoted first, utilising a range of sustainable

power sources, transport solutions and an infrastructure for charging electrically powered vehicles. The infrastructure is already changing fast. By the year 2050, changes in the

energy supply will have transformed our homes, our roads and our behaviour. 1. According to paragraph 1, what has happened in some oil producing countries?

A. They are unwilling to sell their oil any more. B. They are not producing as much oil as they used to.

C. The supply of oil is unpredictable. D. Global warming is more sever here than in other countries. 2. Where in paragraph 1 can the following sentence be placed?

9


There is also likely more electricity generation centres, as households and communities take up the opportunity to install photovoltaic cells and small scale wind turbines. A

B

C

D

technology to consumers? 3. Which of the following is NOT a benefit of Smart Grid technology A. It can reduce their electricity bills. B.

It

can

tell

them

how

much

energy

each

appliance

is

using.

C. It can allow them to turn appliances on and off when they are not at home. D.

It

can

reduce

the

amount

of

energy

needed

to

power

appliances.

4. According to paragraph 4, what is the problem with using renewable sources of power?

A. They do not provide much energy. B. They often cause system failure and blackouts.

C. They do not supply a continuous flow of energy. D .They can't be used at off-peak times. 5. In paragraph 5, what can be inferred about cities in the future? A.

More

people

will

be

living

in

cities

in

the

future

than

nowadays.

B. People in cities will be using cars and buses powered by electricity. C. All buildings will generate their own electricity. D.

Smart

Grid

technology

will

only

be

available

in

cities.

6. The word 'remote' in paragraph 5 could be best replace by:

B. crowded

A. isolated

C. attractive

D.

alone

7. The word 'underway' in paragraph 6 is closest in meaning to:

B. complete

A. permanent

C. beneficial

D. in progress

8. What is the main idea of the final paragraph? (paragraph 6).

A. B. C.

To To To

describe

who

outline summarise

will the the

benefit

from

aadvantages dvantages main

ideas

Smart

of

Grid

Smart in

the

technology Grid

previous

first.

technology. paragraphs.

D. To describe how, where and when Smart Technology will be introduced. 9. In paragraph 6, what can be inferred about the introduction of Smart Grid Technology?

10


A. The technologies which produce most benefits will be introduced first. B.

The

cheapest

technologies

will

be

introduced

first.

C. The technologies which are most difficult to put into place will be introduced first. D. Technologically advanced systems will be introduced first. 10. The word “infancy” in paragraph 6 is closest in meaning to: A. completion

B. beginning

C. old age

D. malnutrition

Part 4: Read the following passage and answer the questions. ALTERNATIVE FARMING METHODS IN OREGON

Onion growers in eastern Oregon are adopting a system that saves water and keeps topsoil in place, while producing the highest quality “super colossal” onions. Pear growers in southern Oregon have reduced their use of some of the most toxic pesticides by up to two-thirds, and are still producing top-quality pears. Range managers throughout the state have controlled the poisonous weed tansy ragwort with insect predators and saved the Oregon livestock industry up to $4.8 million a year. These are some of the results Oregon growers have achieved in collaboration with Oregon State University (OSU) researchers as they test new farming methods including integrated pest management (IPM). Nationwide, however, IPM has not delivered results comparable to those in Oregon. A recent U.S General Accounting Office (GAO) report indicates that while integrated pest management can result in dramatically reduced pesticide use, the federal government has been lacking in effectively promoting that goal and implementing IPM. Farmers also blame the government for not making the new options of pest management attractive. “Wholesale changes in the way that farmers control the pests on their farms is an expensive business.” Tony Brown, of the National Farmers Association says. “If the farmers are given tax breaks to offset the expenditure, then they would willingly accept the new practices.” The report goes on to note that even though the use of the riskiest pesticides has declined nationwide, they still make up more than 40 percent of all pesticides used today; and national pesticide use has risen by 40 million kilograms since 1992. “Our food supply remains the safest and highest quality on Earth but we continue to overdose our farmland with powerful and toxic pesticides and to under-use the safe and 11


effective alternatives,” charged Patrick Leahy, who commissioned the report. Green action groups disagree about the safety issue. “There is no way that habitual consumption of foodstuffs grown using toxic chemicals of the nature found on today’s farms can be healthy for consumers,” noted Bill Bowler, spokesman for Green Action, one of many lobbyists interested in this issue. The GAO report singles out Oregon’s apple and pear producers who have used the new IPM techniques with growing success. Although Oregon is clearly ahead of the nation, scientists at OSU are taking the Government Accounting Office criticisms seriously. “We must continue to develop effective alternative practices that will reduce environmental hazards and produce high quality products,” said Paul Jepson, a professor of entomology at OSU and new director of OSU’s Integrated Plant Protection Centre (IPPC). The IPPC brings together scientists from OSU’s Agricultural Experiment Station, OSU Extension service, the U.S. Department of Agriculture and Oregon farmers to help develop agricultural systems that will save water and soil, and reduce pesticides. In response to the GAO report, the Centre is putting even more emphasis on integrating research and farming practices to improve Oregon agriculture environmentally and economically. “The GAO report criticizes agencies for not clearly communicating the goals of IPM,” said Jepson. “Our challenge is to greatly improve the communication to and from growers, to learn what works and what doesn’t. The work coming from OSU researchers must be adopted in the field and not simply languish in scientific journals.” In Oregon, growers and scientists are working together to instigate new practices. For example, a few years ago scientists at OSU’s Malheur Experiment Station began testing a new drip irrigation system to replace old ditches that wasted water and washed soil and fertilizer into streams. The new system cut water and fertilizer use by half, kept topsoil in place and protected water quality. In addition, the new system produced crops of very large onions, rated “super colossal” and highly valued by the restaurant industry and food processors. Art Pimms, one of the researchers at Malheur comments: “Growers are finding that when they adopt more environmentally benign practices, they can have excellent results. The new practices benefit the environment and give the growers their success.” 12


OSU researchers in Malheur next tested straw mulch and found that it successfully held soil in place and kept the ground moist with less irrigation. In addition, and unexpectedly, the scientists found that the mulched soil created a home for beneficial beetles and spiders that prey on onion thrips – a notorious pest in commercial onion fields – a discovery that could reduce the need for pesticides. “I would never have believed that we could replace the artificial pest controls that we had before and still keep our good results,” commented Steve Black, a commercial onion farmer in Oregon, “but instead we have actually surpassed expectations.” OSU researchers throughout the state have been working to reduce dependence on broad-spectrum chemical sprays that are toxic to many kind of organisms, including humans. “Consumers are rightly putting more and more pressure on the industry to change its reliance on chemical pesticides, but they still want a picture-perfect product,” said Rick Hilton, entomologist at OSU’s Southern Oregon Research and Extension Centre, where researchers help pear growers reduce the need for highly toxic pesticides. Picture perfect pears are an important product in Oregon and traditionally they have required lots of chemicals. In recent years, the industry has faced stiff competition from overseas producers, so any new methods that growers adopt must make sense economically as well as environmentally. Hilton is testing a growth regulator that interferes with the molting of codling moth larvae. Another study used pheromone dispensers to disrupt codling moth mating. These and other methods of integrated pest management have allowed pear growers to reduce their use of organophosphates by two-thirds and reduce all other synthetic pesticides by even more and still produce top-quality pears. These and other studies around the state are part of the effort of the IPPC to find alternative farming practices that benefit both the economy and the environment. Questions 78 – 85: Match the views (78 – 85) with the people listed below.

78. There is a double advantage to the new techniques. 79. Expectations of end users of agricultural products affect the products. 80. The work on developing these alternative techniques is not finished. 81. Eating food that has had chemicals used in its production is dangerous to our health. 13


82. Changing current farming methods is not a cheap process. 83. Results have exceeded anticipations. 84. The research done should be translated into practical projects. 85. The U.S. produces the best food in the world. TB

Tony Brown

PL

Patrick Leahy

BB

Bill Bowler

PJ

Paul Jepson

AP

Art Pimms

SB

Steve Black

RH

Rick Hilton

Questions 86 - 90 Read the passage about alternative farming methods in Oregon again and look at the statements below.

In boxes 86 - 90 write: TRUE FALSE NOT GIVEN

if the statement is true if the statement is false if the information is not given in the advertisement

86. Integrated Pest Management has generally been regarded as a success in the US. 87. Oregon farmers of apples and pears have been promoted as successful examples of Integrated Pest Management. 88. The IPPC uses scientists from different organisations. 89. Straw mulch experiments produced unplanned benefits. 90. The apple industry is now facing a lot of competition from abroad. Part 5: Read throught the following text and choose from the list A- the best sentence to fit each blank.

What do we mean by being ‘talented’ or ‘gifted’? The most obvious way is to look at the work someone does and if they are capable of significant success, label them as talented. The purely quantitative route - ‘percentage definition’ - looks not at individuals, but at 14


simple percentages, such as the top five per cent of the population, and labels them - by definition - as gifted. (1) ………………….. The IQ test has been eclipsed in turn. Most people studying intelligence and creativity in the new millennium now prefer a broader definition, using a multifaceted approach where talents in many areas are recognised rather than purely concentrating on academic achievement. (2)………………….. Mental dysfunction - such as schizophrenia - can, so is an efficient mental capacity passed on from parent to child? Animal experiments throw some light on this question, and on the whole area of whether it is genetics, the environment or a combination of the two that allows for intelligence and creative ability. (3) ……………..If these are brought up in normal conditions and then mn through a maze to reach a food goal, the ‘bright’ strain make far fewer wrong turns that the ‘dull’ ones. But if the environment is made dull and boring the number of errors becomes equal. Return the rats to an exciting maze and the discrepancy returns as before - but is much smaller. In other words, a dull rat in a stimulating environment will almost do as well as a bright rat who is bored in a normal one. (4) …………… Evidence now exists that most young children, if given enough opportunities and encouragement, are able to achieve significant and sustainable levels of academic or sporting prowess. (5) ………….. They may also talk earlier, and this, in turn, breeds parental interest. This can sometimes cause problems with other siblings who may feel jealous even though they themselves may be bright. Their creative talents may be undervalued and so never come to fruition. Two themes seem to run through famously creative families as a result. (6) …………..Individual differences were encouraged, and friendly sibling rivalry was not seen as a particular problem. If the father is, say, a famous actor, there is no undue pressure for his children to follow him onto the boards, but instead, their chosen interests are encouraged. (7) …………………. Martin Sheen was the seventh of ten children born to a Spanish immigrant father and an Irish mother. (8) ……………… His acting successes in films such as Badlands and Apocalypse Now made him one of the most highly-regarded actors of the 1970s. Three sons

15


- Emilio Estevez, Ramon Estevez and Charlie Sheen - have followed him into the profession as a consequence of being inspired by his motivation and enthusiasm. A stream seems to run through creative families. Such children are not necessarily smothered with love by their parents. (9) ……………..They may see from their parents that it takes time and dedication to be master of a craft, and so are in less of a hurry to achieve for themselves once they start to work. (10) ……………………………This last point - luck - is often not mentioned where talent is concerned but plays an undoubted part. Mozart, considered by many to be the finest composer of all time, was lucky to be living in an age that encouraged the writing of music. He was brought up surrounded by it, his father was a musician who encouraged him to the point of giving up his job to promote his child genius, and he learnt musical composition with frightening speed - the speed of a genius. Mozart himself simply wanted to create the finest music ever written but did not necessarily view himself as a genius - he could write sublime music at will, and so often preferred to lead a hedonistic lifestyle that he found more exciting than writing music to order.

A. The first is that the parents were able to identify the talents of each child, and nurture and encourage these accordingly but in an even-handed manner. B. Despite intense parental disapproval he turned his back on entrance exams to university and borrowed cash from a local priest to start a fledgling acting career. C. If we are therefore assuming that talented, creative or gifted individuals may need to be assessed across a range of abilities, does this mean intelligence can run in families as a genetic or inherited tendency? D. This principle applies to humans too - someone may be born with innate intelligence, but their environment probably has the final say over whether they become creative or even a genius. E. They feel loved and wanted, and are secure in their home, but are often more surrounded by an atmosphere of work and where following a calling appears to be important.

16


F. This definition has fallen from favour, eclipsed by the advent of IQ tests, favoured by luminaries such as Professor Hans Eysenck, where a series of written or verbal tests of general intelligence leads to a score of intelligence. G. Bright or creative children are often physically very active at the same time, and so may receive more parental attention as a result - almost by default - in order to ensure their safety. They may also talk earlier, and this, in turn, breeds parental interest. H. There need not even by any obvious talent in such a family since there always needs to be someone who sets the family career in motion, as in the case of the Sheen acting dynasty. I. The generation of creativity is complex: it is a mixture of genetics, the environment, parental teaching and luck that determines how successful or talented family members are. J. Different strains of rats show great differences in intelligence or ‘rat reasoning’. IV. WRITING: Part 1: Read the following extract and use your own words to summarise it. Your summary must be between 100 and 120 words long. Psychology and personality assessment

Our daily lives are largely made up of contacts with other people, during which we are constantly making judgments of their personalities and accommodating our behaviour to them in accordance with these judgments. A casual meeting of neighbours on the street, an employer giving instructions to an employee, a mother telling her children how to behave, a journey in a train where strangers eye one another without exchanging a word - all these involve mutual interpretations of personal qualities. Success in many vocations largely depends on skill in sizing up people. It is important not only to such professionals as the clinical psychologist, the psychiatrist or the social worker, but also to the doctor or lawyer in dealing with their clients, the businessman trying to outwit his rivals, the salesman with potential customers, the teacher with his pupils, not to speak of the pupils judging their teacher. Social life, indeed, would be impossible if we did not, to some extent, understand, and react to the motives and qualities of those we meet; and clearly we are sufficiently accurate for most practical purposes, although we also recognize that misinterpretations easily arise particularly on the part of others who judge us! 17


Errors can often be corrected as we go along. But whenever we are pinned down to a definite decision about a person, which cannot easily be revised through his “feed-back”, the inadequacies of our judgments become apparent. The hostess who wrongly thinks that the Smiths and the Joneses will get on well together can do little to retrieve the success of her party. A school or a business may be saddled for years with an undesirable member of staff, because the selection committee which interviewed him for a quarter of an hour misjudged his personality. ………………………………………………………………………………………………… ………………………………………………………………………………………………… ………………………………………………………………………………………………… ………………………………………………………………………………………………… ………………………………………………………………………………………………… ………………………………………………………………………………………………… ………………………………………………………………………………………………… ………………………………………………………………………………………………… ………………………………………………………………………………………………… ………………………………………………………………………………………………… ………………………………………………………………………………………………… ………………………………………………………………………………………………… ………………………………………………………………………………………………… ………………………………………………………………………………………………… ………………………………………………………………………………………………… ………………………………………………………………………………………………… ………………………………………………………………………………………………… ………………………………………………………………………………………………… Part 2: The graph below shows the different modes of transport used to travel to and from work in one European city in 1960, 1980 and 2000. Summarise the information by selecting and reporting the main features, and make comparisons where relevant. You should write about 150 words.

18


………………………………………………………………………………………………… ………………………………………………………………………………………………… ………………………………………………………………………………………………… ………………………………………………………………………………………………… ………………………………………………………………………………………………… ………………………………………………………………………………………………… ………………………………………………………………………… ………………………………………………………………………………………………… ……………………… ………………………………………………………………………………………………… ………………………………………………………………………………………………… ………………………………………………………………………………………………… ………………………………………………………………………………………………… ………………………………………………………………………………………………… ………………………………………………………………………………………………… ……………………………………………… ………………………………………………………………………………………………… ………………………………………………… ………………………………………………………………………………………………… ………………………………………………………………………………………………… ………………………………………………………………………………………………… ………………………………………………………………………………………………… Part 3:

Gaming has become a popular e-sport and has been included in some regional and international sports events. Some people advocate this, claiming that there is almost no risk of injury with e-sports; others are worried that the increasing popularity of 19


competitive gaming tournaments may exacerbate video game addiction among young people. Write an essay of about 350 words to present your opinion on this issue. Give reasons and specific examples to support your position. ………………………………………………………………………………………………… ………………………………………………………………………………………………… ………………………………………………………………………………………………… ………………………………………………………………………………………………… ………………………………………………………………………………………………… ………………………………………………………………………………………………… ………………………………………………………………………………………………… ………………………………………………………………………………………………… ………………………………………………………………………………………………… ………………………………………………………………………………………………… ………………………………………………………………………………………………… ………………………………………………………………………………………………… ………………………………………………………………………………………………… ………………………………………………………………………………………………… ………………………………………………………………………………………………… ………………………………………………………………………………………………… ………………………………………………………………………………………………… ………………………………………………………………………………………………… ………………………………………………………………………………………………… ………………………………………………………………………………………………… ………………………………………………………………………………………………… ………………………………………………………………………………………………… ………………………………………………………………………………………………… ………………………………………………………………………………………………… ………………………………………………………………………………………………… ………………………………………………………………………………………………… 20


21


I. LISTENING: Part 1: 0,2 điểm/1 câu 1. D

2. A

3. B

4. A

5. C

Part 2: 6. F

7. T

8. F

9. F

10. F

Part 3: 11. Every corner

12. ecstatic

13. Schools/swathes of planet

14. jammers

15. To gather intelligence

Part 4: 16. bizarre

17. pattern of pouches

18. heat signatures

19. startling

20. white blood cells

21. pink lakes

22. intrigues

23. alien-looking

24. flora and fauna

25. volcanic crater lakes II: Grammar& Vocabulary Part 1: 0,1 điểm/ 1 câu 26. D

27. B

28. B

29. C

30. C

31. A

32. D

33. D

34. C

35. D

Part 2: 36. whichever → whatever

37. In midday → at midday

38. exhibited → exhibiting

39. lain → laid

40. for → to Part 3: 41. in with

42. out on

Part 4: 46. counter-intuitive 47. misperceive

43. down

44. on

45. on/upon


48. varying 49. outlying 50. characteristics 51. markedly 52. homogeneity 53. accentuate 54. upturned 55. fleshy III. READING. Part 1: 56. B

57. B

62. C

58. A

63. C

59. D 64. D

60. A

61. B

65. A

Part 2: 1. everything

2. But/Yet

3. without

4. in

5. What

6.

7. back

8. Another

9. with

10. and

Such/These/Those Part 3: 1. B

2. C

3. D

4. C

5. B

6. A

7. D

8. D

9. A

10. B

78. AP

79. RH

80. PJ

81. BB

82. TB

83. SB

84. PJ

85. PL

86. FALSE

87. TRUE

88. TRUE

89. FALSE

90. NOT GIVEN

Part 4:

Part 5. 1. F

2. C

3. J

4. D

5. G

6. A

7. H

8. E

9. B

10. I


TRANSCRIPTS: PART 1: Interviewer: Rachel White shot to fame in the 19905 with the publication of her first novel Crying Wolf. She has since become a well established author and critic and I have her in the studio with me today to talk about her work and her inspiration as a write. We1come to the programme, Racchel. Rachel: Thank you. Interviewer: Rachel. If we can talk about inspiration first. You always say that you were inspired to write novels by other famous novelists but was there a particular moment when true inspiration occurred and writing, as such, fall into place for you? Rachel: Oh yes I was a fairly academic girl at school, which, well, it sort of went against the grain I was in my fifth year and, um, I was really keen to shake off this reputation I was getting for being a model pupil, particularly in the literature classes, when one day the teacher started the lesson by reeling of all these compliments about an essay I'd done on the author Lawrence Durrell, one of my personal favourites_ This resulted in titters all round and all the efforts I'd been makinq to keep a low profile were completely dashed. Then, to cap it all, he got me to read it aloud. Interviewer: Never a happy experience. But deep down was it

an essay you were

particularly proud of? Rachel: No, and what's more, it went on for ten pages but most of it I'd pinched from a couple of books on literary criticism that I had at home ~ I'd reading Durrel but I hadn’d succeeded, although I must say, it was unusual for someone my age to be reading criticiam at all. So, I stood there spouting this stuff about a book and a writer I didn't know. And then suddenly I camp to a quote and I was stunned : it was sheer poetry and I thought I must read this. Interviewer: Ann you couldn’t help voicing this observation. Rachel: Mm. I think I was loathed from that moment on but it didn’t matter then I'd fallen in love with literature.


Interviewer: You're also quoted as saying that the classics and I'm thinking particularly of Jane Austen were terribly important to you. You say you were struck by the timelessness of Austen. Rachel: Yeah. I think Jane Austen is one of those writers that people like to 'tick off' on their personal book list. so they flick through, say, Pride and Prejudice on a quiet day and then they make a quick retreat to their personal favourite. I felt fortunate in some way that, at 16, I could appreciate the insight that she'd had. Interviewer: You're well known in literary circles for your essay Literature and the Young Mind'. Tell us more about that. Rachel: I think literature has really made an impact when you look back and realise that aspects of your own philosophy of life have evolved in response to things that you read - obviously experience plays a large part. too, bul that comes later and is often a reinforcement I think a lot of writers share an appreciation of the power that literature had over them as young people and I just wanted to voice that. I owe a debt to Austen, for example, because I refrain from forming judgements about people on the basis of first impressions. Interviewer: So once you were bitten by the literature bug if you like, how long did it take? Did you start writing at an early age or did that come later? Rachel: No, after the Durrell essay there was no turning back. Of course, it was all selfconscious, self-indulgent stuff about family and boyfriends and a little striving for a better self, but not much. All par for the course, I think. Interviewer: Something you have to get out of your system. Rachel: That's what they say. Anyway, I think I've done that now and I can turn my mind to some of the more deeply held ideas that I have and bring those to the surface. Interviewer: Would you like to see your own books described as inspirational? Rachel: I don't think any writer sets out to stir other writers into action. It's much more a question of art, of creating something that works, that has resonance and that provides fulfilment for the writer and reader. Anything more than that is a bonus. Interviewer: Thank you very much, Rachel White.


PART 2: A (girl):

I really like The Notebook; it’s a romantic classic. The story is about a

poor boy, called Noah, who meets a rich girl, called Allie, and they fall in love over one summer. But of course there are problems. Allie’s parents don’t approve of Noah because he’s poor and they get separated, then a lot of time passes before they can get together again. She goes away to college and he writes to her every day, but she doesn’t get the letters. Then after quite a long time , Noah buys an old house which he promised to restore for Allie. He does it up and she sees a newspaper article about it and goes to find him again. You can probably guess the rest, but it’s a bit complicated because she is engaged to another man. Noah is played by Ryan Gosling, who’s my favourite actor, and he stops the film being too soppy – but you still cry a lot at the end. Rachel McAdams is really good too, as Allie. B (boy):

I think my favourite film is Skyfall, the last James Bond movie. It stars

Daniel Craig again and Javier Bardem is this really evil baddie. They’re both great, and so is Judi Dench as M, Bond’s boss. It’s a bit different to other Bond films. The story is more important, although there is still a lot of action. It’s really well filmed, especially in the scenes which are shot in Shanghai and Scotland, and it’s really exciting, of course. Bond is also a bit different – although he does amazing things, he seems older and that makes him more human and the story more realistic. Oh, the song by Adele is great, too. You know it won an Oscar? C (girl):

It’s not a very recent film, but I love Pride and Prejudice – the version

with Keira Knightley as Elizabeth Bennet and Matthew Macfadyen as Mr Darcy. The story is really clever and interesting, besides being romantic. It’s all about how we tend to jump to conclusions about people and we’re often completely wrong. Elizabeth thinks Mr Darcy is stuck up and snobbish at the beginning of the film, but at the end she realises she has been completely wrong and he isn’t like that at all. She also realises he has an enormous house and that maybe helps her fall in love with him too! After I saw the film I read the book by Jane Austen and I really recommend that too. Although it was published 200 years ago, it’s still a great read and really funny in parts.


D (boy): My favourite film has to be The Matrix. Even though it was made back in 1999, it still feels really modern. It’s about this ordinary man, Neo, played by Keanu Reeves, who works with computers during the day and is a hacker at night. One day he is contacted by a woman who introduces him to a very strange man: Morpheus. Morpheus explains to Neo that what he thinks of as real is actually fiction and the world is run by evil machines who have imprisoned and tricked the human race. Laurence Fishburne is terrific as Morpheus, and Keanu Reeves is excellent, but the best thing about the film is the special effects. I can watch it again and again. E (girl):

Let the Right One In is my favourite film. There are two versions, the first is

Swedish and the second American – I prefer the original, Swedish version. It’s a modern vampire film and also a kind of love story, but it’s really unsentimental and a bit scary in places. Oskar is a young boy with a lot of problems. He is being bullied at school and wants revenge on the bullies. Then some new neighbours move in next door and he becomes friends with Eli, a beautiful but strange girl. I won’t tell you any more, because it will spoil the story, but expect a serial killer, a lot of violence and blood-drinking. If you like that kind of thing, you’ll love this. PART 3: A satellite start-up company has successfully launched its first satellites in a project aimed at delivering "affordable" Internet access to every corner of the globe. The Londonbased company is called OneWeb. Its first six satellites were put into orbit on Wednesday in a launch from French Guiana. A Russian Soyuz rocket transported the pioneering spacecraft. OneWeb said it eventually wants to have around 900 spacecraft in an extensive network in space. Only 648 satellites are needed to provide global Internet coverage. OneWeb will start sending more Internet satellites into space towards the end of this year. The company intends to launch several rockets every month, each carrying 39 satellites. OneWeb CEO Adrian Steckel was ecstatic about the launch of the multi-billion dollar project. He said it would greatly transform the lives of millions of people. He told reporters: "We're going to connect lots of people who are not currently connected. We're going to start by focusing on connecting schools, connecting boats, connecting planes, and


connecting huge swathes of the planet." Not everyone is happy at the prospect of a global Internet network. Reports are that China is testing jammers to block the network. North Korea might also fear the network could damage its national security. Russia has voiced concerns that OneWeb's network could be used to gather intelligence. PART 4: Have you ever heard about something that is unexplained? There are a lot of bizarre things in the world such as animals raining from the sky, or weird unexplained heat anomalies found inside the Great Pyramid of Giza. Sometimes the answer is found, and sometimes these things remain a mystery that science cannot explain. Check out these 10 Natural Phenomena that Science Can’t Explain. 10. Mammatus clouds - which means "mammary cloud", is a cellular pattern of pouches hanging underneath the base of a cloud, these are typically heavy cumulonimbus rainclouds. Mammatus are most often associated with severe weather, and they can appear before or after a storm. 9. 'Heat anomaly' found in Great Pyramid of Giza - Scientists have been studying the Great Pyramid of Giza, which is the oldest of the Seven Wonders of the Ancient World, and have found strange heat signatures that have no explanation. 8. Oakville Blobs - On August 7, 1994 during a rainstorm, blobs of translucent gelatinous substance fell in the town of Oakville, Washington. Several attempts were made to identify the blobs. 7. Lake Hillier, Australia - Australia is the home of unusual ‘pink lakes’. Lake Hillier is one of them. Located on Middle Island, this body of water is famous for it’s remarkable bubble gum pink color. 600 meters long and surrounded by a rim of sand and thick woods of eucalyptus trees, this pink lake still surprises and intrigues scientists.


6. Socotra Island, Yemen - Want to feel like you’re visiting another planet? Check out Socotra Island off the coast of Yemen. It has been called the most alien-looking place on Earth, and a ‘lost world’ because the heat and drought conditions of the island have produced around 800 rare species of flora and fauna, and a third of those can’t be seen anywhere else on the planet. 5. Kelimutu, Indonesia - Mount Kelimutu is a volcano in Flores, Indonesia. The mountain has three volcanic crater lakes that vary colors from each other at different times, making them surreal and exciting to be discovered.

4. Brain Injury Turns Man Into Math Genius - After being brutally attacked in 2002, Jason Padgett now sees the world through a geometric lens. His devastating injury seems to have unlocked part of his brain that makes everything in his world appear to have a mathematical structure. 3. Stonehenge - For centuries, historians and archaeologists have puzzled over Stonehenge, which took Neolithic builders around 1,500 years to build. Located in Southern England, it is comprised of roughly 100 massive upright stones placed in a circular layout. 2. Animal rain phenomenon - The origins of this bizarre animal rain phenomenon have evolved throughout history. The first reported phenomenon occurred in the first century AD, when Roman naturalist Pliny the Elder documented fish and frogs raining from the sky. 1. Tsingy de Bemaraha, Madagascar - The Tsingy de Bemaraha is a national park located in the Melaky Region, northwest Madagascar. The site is unique with its biodiversity and exceptional landscape called the Tsingy - which can be translated into " where one cannot walk barefoot".


HỘI CÁC TRƯỜNG CHUYÊN VÙNG

ĐỀ THI CHỌN HỌC SINH GIỎI LẦN THỨ XII MÔN: TIẾNG ANH - KHỐI 11

DUYÊN HẢI VÀ ĐỒNG BẰNG BẮC BỘ TRƯỜNG THPT CHUYÊN NGUYỄN TRÃI

Thời gian: 180 phút

TỈNH HẢI DƯƠNG

Đề thi gồm: 16 trang

ĐỀ ĐỀ XUẤT

SECTION 1: LISTENING (50 points) I. You will hear an interview with a British politician. For questions 1-5, choose the answer (A, B, C or D) which fits best according to what you hear. (10pts) 1. Susan says that she particularly dislikes politicians who A. pretend to feel strongly about issues.

B. disguise their real beliefs.

C. are indecisive about issues.

D. openly treat voters with contempt.

2. When she had her disagreement with Martin Jones, Susan A. decided that personal ambition was not her main motivation. B. began to feel that she had failed as a politician. C. felt that her point of view was not correctly understood. D. regretted the effect it would have on her future in politics. 3. What was Susan's attitude to involving colleagues in the controversy? A. She realized that they were unlikely to share her point of view. B. She was reluctant to do so because she was not sure she was right. C. She thought that involving colleagues would make things worse. D. She felt they should decide for themselves whether she had a point, 4. When asked whether her opinion of her colleagues has changed, Susan says that A. their reaction has made her reluctant to get into the same position again. B. she prefers those who criticized her to those who kept theft opinions private. C. there may come a time when she does not publicly support them on issues. D. politicians place too much emphasis on their personal opinions of each other. 5. Susan thinks she was considered mad by some other politicians because A. her behaviour was out of character.

B. they found her intimidating.

C. she did not conform.

D. her unselfishness shamed them. 1


II. Listen to an extract from a radio programme about the climber Annabelle Bond, and say whether sentences 1-5 are true or false. (10 points) 1. It took her about a year to climb the seven mountains. 2. Only four men have climbed the peaks faster than Annabelle. 3. Annabelle always wanted to be a marathon runner. 4. She raised £8,500 for a cancer charity. 5. She plans to do more climbing in the near future. III. You will hear a guide speaking to tourists who are visiting some Romans remains. Listen and give short answers to the questions. Write NO MORE THAN THREE WORDS AND/OR A NUMBER taken from the recording. (10 points) 1. When did the Romans first come to the Corbridge area? ……………………………………………………………………… 2. Why did the Romans built a series of forts and strongholds? ……………………………………………………………………… 3. What did people begin to search for in 1201? ……………………………………………………………………… 4. How often have archaeological digs taken place since 1934? ……………………………………………………………………… 5. What are the two things that visitors should pay attention to? ……………………………………………………………………… IV.Listen to a piece of news about How Japan's economy is destroying Its Youth. WRITE NO MORE THAN THREE WORDS OR A NUMBER (20 points) In Japan, nearly a million young men have locked themselves in their bedrooms, and refuse to go outside. This 1. ………………….. has been attributed as a side effect to Japan’s highly competitive economy and culture. So, how is Japan’s economy destroying their youth? Japanese culture generally places a significant amount of emphasis on education and employment. 2


But in recent years, Japan has seen a significant downturn in their economic growth. However, Japan’s recession isn’t the same as in most other countries. Stores don’t go out of business, luxury spending continues, 2 . ………………….. of society still seems comfortable. But the lack of available new jobs does have a 3 ………………….. younger generations. With fewer jobs to go around, many Japanese men give up hope and withdraw from society. Furthermore, the longer one is unemployed, the harder it is to find a job, since companies tend to 4. ………………….. out of high school and college. Young adults and adolescents who confine themselves for long periods of time are called “hikikomori”. They are among the generation of youths called “grass eaters”, which are mostly men who shy away from 5………………….. and general social contact. They tend to live at home with their parents, some well into their 40s. Additionally, most Japanese youths are expected to provide for their families, putting tremendous stress and pressure on them to succeed. The weak economy also 6………………….. on relationships, as many Japanese women refuse to date unemployed men, considering them weak and 7…………………... Japan’s government has warned that more than one and a half million are on the verge of becoming hikikomori. An estimated 700,000 hikikomori have already locked themselves away. The 8.………………….. anxious men has created big changes for Japan, including a massive drop in birth rates to record lows, and a 9………………….. in consumer habits away from a more “masculine-driven” culture. Studies have shown that as much as 75% of men in their 20s and 30s consider themselves “grass eaters”. Considering the depth of culture 10 ………………….. success and hard work with self identity in Japan, it will take significant social effort and a better economy to improve this epidemic. SECTION 2: LEXICO - GRAMMAR (30 points) 3


I. Choose the word or phrase that best fits the gap in each sentence (10 points) 1.As we entered the farmyard we were met with a(n) ….. of animal sounds. A.alacrity

B. cacophony

C. depravity

D. intermingling

2.Only when she has realized that the only thing standing in her way is her …. Llfestyle can she make progress in her career. A.Sycophant

B. Sybaritic

C. Unwieldy

D. Inbreeding

3.He is a(n) …. young man who shows no respect for gods A.impious

B. ludicrous

C. precipitous

D. capacious

4. Mark decided to follow in his father’s …………… and become a dentist. A. advice

B. way

C. road

D. footsteps

5. They’re having serious problems. Their relationship is on the …………... A. cliffs

B. rocks

C. stones

D. grass

6. In his …………... days he was quite dandy. A. salad

B. green

C. fruit

D. vegetable

7. My manager is a typical working mother who has to deal with …………... activities every day. A. miscellaneous B. multifarious

C. many

D. manifold

8. Naylor was one of those men who …………... to the challenge of danger. A.raise

B. rise

C. ride

D. arise

9.Like more and more women, she believes marriage would …………... her style. A.restricts

B. impedes

C. obstructs

D. cramps

10. He's not nearly such a good writer as he's …………... up to be. A.creased

B. cracked

C. lined

D. valued

II. The passage below contains 5 errors. IDENTIFY and CORRECT them. Write your answers in the space provided in the column on the right. (5 points) Line 1 One limitation is related to the way the museum presents its exhibitions. As repositories of unique historical objects, art museums are often called ‘treasure houses’. We are reminded of this even before we view a collection by the presence of security guards, attendants, ropes and display 5

cases to keep us away from the exhibits. In many cases, the architectural 4


style of the building further reinforces that notion. In addition, a major collection like those of London’s National Gallery is housed in numerous rooms, each with dozens of works, any one of which is likely to be worth more than all the average visitor possesses. In a society that judges the 10

personal status of the individuals so much by their material worth, it is, therefore, difficult not to be impressed by one’s own relative ‘worthlessness’ in such the environment. Furthermore, consideration of the ‘value’ of the original work in its treasure house setting impresses upon the viewer that, since these works were originally produced, they have been

15

assigned a huge monetary value by some person or institution more powerful than themselves. Evidently, nothing the viewer thinks about the work is going to alter that value, and so today’s viewer is deterred against trying to extend that spontaneous, immediate, self-reliant kind of reading which would originally have met the work.

III. Fill in each blank with an appropriate preposition or particle. (5 points) 1.She kept her emotions tightly reined ……………….. . 2. It rests ……………….. management to justify their actions. 3. There have been concerns as to why the Board didn't act …….. a recommended pay raise. 4. Some of the children have fears about riding ……………….. another storm. 5. I'd never heard about the company before, so I had to think ……………….. my feet. IV. Supply the correct form of the verbs in block capitals in brackets to complete the passage. (10 points) The (1. assert) __________ that mathematics has been a major force in the of modern culture appears to many people (2. credible) __________or, at best, a rank exaggeration. This

(3.

believe)

____________results

from

a

very

common

but

(4.

error)

__________conception of what mathematics really is. Influenced by what was taught in school, the average person regards mathematics as a series of techniques of use only to the scientists, the engineer and perhaps the (5. finance) ____________ . The reaction to such teaching is (6. taste) __________ for the subject and a decision to ignore it. When challenged 5


on this decision, a well-read person can obtain the support of authorities. No less than a (7. person) __________than Schopenhauer, the philosopher, described mathematics as the (8. low) __________activity of the spirit, as is shown by the fact that it can be performed by a machine. Despite such (9. authority)

__________ judgments, the layman’s decision to

ignore mathematics is wrong. The subject is not a series of techniques. These are indeed the least important aspects. The techniques are mathematics stripped of motivation, (10 reason) ________ , beauty and significance. SECTION 3: READING (60 points) I. Read the text below and decide which answer (A, B, C or D) best fits each gap. (10 points) THE TRADE IN RHINO HORN Last year thieves broke into a Scottish castle and stole only one thing: a rhino horn, which at 1.5 metres was the longest in the world. In China pharmaceutical factories have been building up collections of antiques made from rhino horn, for the sole (1)………of smashing them to powder to make the essential ingredient of many of their medicines. And in

Africa

poachers

continue

to

die

in

the

(2)………..for

the

black

rhino.

Recently, conservations met to (3)……… a campaign to persuade countries where rhino horn is (4)……… part of the traditional medicine to (5)………. to substitutes. The biggest threat to the survival of the rhinoceros is the (6)……….. of certain countries to enforce a ban on

domestic

(7)……

in

rhino

horn.

The rhino horn is included in many (8)……….. for disorders ranging from fevers to nosebleeds. Horn, like fingernails, is made of keratin and has no proven medicinal (9)………… Traditional substitutes, such as horn from buffalo or antelope, are regarded as second best. The battle is thought to be winnable. But it may be harder than the battle against the trade in ivory, for there is a (10)………between the two commodities. Ivory is a luxury; rhino horn, people believe, could save the life of their child.

6


1. A. reason

B. intention

C. need

D. purpose

2. A. chance

B. search

C. fight

D. race

3.A. design

B. plan

C. programme

D. form

4.A. hardly

B. even

C. nearly

D. still

5.A. vary

B. switch

C. modify

D. adjust

6. A. rejection

B. denial

C. refusal

D. protest

7. A. business

B. commerce

C. selling

D. trading

8. A. recipes

B. aids

C. remedies

D. doses

9. A. capacity

B. values

C. control

D. powers

10.A. variation

B. difference

C. gap

D. comparison

II. Fill each gap in the passage below with ONE appropriate word in the space provided. (15 points) Contemporary art matters Contemporary art plays on the emotions and (1) …………… the mind. It can send powerful messages, (2) …………… political, social or environmental issues. It can also lead to (3) …………… reactions, including outrage from those who like to be shocked. It is available to everybody. Admission to contemporary art museums is usually free, whereas viewing traditional art often (4) …………… going to expensive art galleries to see the (5) …………… . Major museums may contain thousands of works, and although inevitably some will not be to everyone’s (6) …………… , many will be quite fascinating. They may even (7) …………… the visitor to create their own pieces; if not for display in a museum, then as a form of street art – which is a wonderful way for young people to (8) …………… their feelings and ideas. In fact, creating works of contemporary art can be therapeutic, often proving (9) …………… effective in helping people (10) …………… recover from emotional or psychological problems.

III. Read the following text and choose the best answer. (10 points) 7


AROUND THE WORLD IN 94 DAYS In February 2001, at the age of 24, Ellen Mac Arthur became the youngest and fastest ever woman to sail round the world. After 94 days alone on board her yacht Kingfisher, she finished second to Michel Desjoyaux of France in the single-handed Vende Globe event. In sport, like life, the winner is usually feted, and runners-up quickly forgotten. This time the roles were reversed and it was Ellen, weighing just 50 kilos and barely 1m60 tall, that really captured people's imaginations and emotions. One newspaper in France, where she was and is a real heroine, summed up the national mood there with the headline 'Well done, Michel, brave Ellen'. As with many spectacular achievers, the signs were there from an early age, even in the unpromising nautical terrain of landlocked Derbyshire. Her great-grandparents were sailing people and a great-uncle was a merchant seaman, but any real link with the sea is tenuous. There was, however, an Auntie Thea who lived on the east coast of England and had a 26foot sailing boat called Cabaret. It took just one trip on the open sea with her aunt to spark off Ellen's lifelong passion. She was eight years old. After that she began saving her pocket money and spent all her spare time reading sailing books in the library, absorbing information like a sponge. With her savings and the help of her grandmother she bought an 8-ÂŁ00t fibre glass dinghy, and from that moment on there was no keeping her away from the water. Sailing round Britain single-handed at the age of 18 was just the start; Ellen had long since set her sights on the Vende. But finding the money to undertake round-the-world voyages is no easy feat. She wrote 2,000 letters requesting sponsorship and received just two replies, from the Kingfisher company who were looking to expand into France. And in terms of race preparation, if thoroughness was the key to success, Ellen could certainly be considered one of the favourites. In the eight months leading up to the start of the race, she sailed no fewer than 60,000 miles at the helm of her 60-ÂŁ001 Kingfisher, far more than the rest of the fleet put together in the same period. During her three months at sea MacArthur negotiated deadly icebergs, gigantic waves and gale-force winds. She endured the freezing cold of the Antarctic and suffered the blistering heat of the windless doldrums. Racing conditions meant sleeping in 10-minute 8


bursts, a survival suit that stayed on for weeks at a time and hands and wrists covered in sores and cuts. Food was dried or frozen. Water came from a desalinator, which passes sea water through a membrane. 'You don't really wash in the icy waters of the southern ocean,' she laughs. 'Anyway, there's no one to tell you that you smell.' As Kingfisher crossed the finishing line Ellen was surrounded by hundreds of spectator boats and a cheering crowd of 200,000 lined the shore. Stepping off her yacht she looked remarkably composed and seemed to take the change from solitude to public adulation very much in her stride. Her thoughts, she later confessed, were on the realization that she had fulfilled the ambition that had dominated her life for the previous four or five years. 'Throughout that time my sole focus had been crossing the finishing line, and in the fastest possible time.' Now she could savour that moment. But despite MacArthur's belief that everyone who finishes the Vende is a winner, she still feels a sense of disappointment that, having taken the lead from the eventual winner Michel Desjoyaux 10 days from the finish, she did not quite have the energy or good fortune to turn her advantage into victory. 'You have to believe you can win from the start,' she asserts. 'Deep down you're a competitor, you don't climb the mast and come back black and blue just for a cruise. You do it because it's a race.' The public will now be hoping to see a suitable encore, some new feat of endurance to justify her celebrity status. For Ellen can no longer claim, as she did in her post-race press conference, to be the simple Derbyshire girl with 'no mobile, no credit cards, no money, no nothing'; she is a heroine and an inspiration to others of her generation. As if to reinforce this, and despite her reluctance to take on this role, she later commented: 'If there's one thing I've learned in this past year, it's that deep down in your heart, if you have a dream, then you can and must it happen.'

1. The word feted in the second paragraph means A. to make somebody pleasant.

B. to praise somebody.

C. to make somebody happy.

D. to give somebody a reward.

2. At the time of her achievement we learn that Ellen A. enjoyed only short-lived success. 9


B. was more famous in France than anywhere else. C. attracted more attention than Michel Desjoyaux. D. became popular because of her size. 3. The word landlocked in the third paragraph means A. having no seaport.

B. having no fresh water .

C. having no land.

D. having no sea coast.

4. Where did Ellen's initial interest in sailing come from? A. She came from a family of sailing enthusiasts: B. She went to see one of her relatives. C. She read widely on the subject. D. She lived near the sea. 5. The word single-handed in the fourth paragraph means A. without any help from anyone else. B. using only one of one's hands to row. C. on a boat with only one paddle. D. on a boat with only one sail. 6. What do we learn about Ellen at the start of the race? A. People thought she had a very good chance of winning. B. She was a more experienced sailor than the other racers. C. She had been waiting for this moment since she was 18. D. She had gone to great lengths to achieve her ambition. 7. The writer suggests that one cause of discomfort for Ellen at sea was A. the shortage of water.

B. her failure to sleep.

C. extremes of temperature.

D. a lack of cooking facilities.

8. According to the writer, when Ellen finished the race, she was A. overwhelmed by her new-found fame. B. surprised by the number of people who came to greet her. C. able to reflect on her achievement. D. delighted to be amongst people again. 9. According to the writer, Ellen 10


A. thinks she deserved to win the race. B. has mixed feelings about the outcome of the race. C. knew she would win the race. D. thinks Michel Desjoyaux was lucky to beat her. 10. Which of the following views does the writer express in the last paragraph? A. She has the power to motivate. B. She has no right to fame yet. C. Her comments lack depth.

D. She needs to change her lifestyle.

IV. Read the following extract and answer questions 1–10. (10 pts) TERROR IN THE MOUNTAINS A What is incredibly beautiful yet absolutely terrifying and deadly at the same time? For anyone above the snowline in the mountains, there is little doubt about the answer. Avalanche – the word strikes fear into the heart of any avid skier or climber. For those unfortunate enough to be caught up in one, there is virtually no warning or time to get out of danger and even less chance of being found. The ‘destroyer’ of the mountains, avalanches can uproot trees, crush whole buildings, and bury people metres deep under solidified snow. Around the world, as more and more people head to the mountains in winter, there are hundreds of avalanche fatalities every year. B A snow avalanche is a sudden and extremely fast-moving ‘river’ of snow which races down a mountainside (there can also be avalanches of rocks, boulders, mud, or sand). There are four main kinds. Loose snow avalanches, or sluffs, form on very steep slopes. These usually have a ‘teardrop’ shape, starting from a point and widening as they collect more snow on the way down. Slab avalanches, which are responsible for about 90% of avalancherelated deaths, occur when a stiff layer of snow fractures or breaks off and slides downhill at incredible speed. This layer may be hundreds of metres wide and several metres thick. As it tends to compact and set like concrete once it stops, it is extremely dangerous for anyone buried in the flow. The third type is an isothermal avalanche, which results from heavy rain leading to the snowpack becoming saturated with water. In the fourth type, air mixes in with loose snow as the avalanche slides, creating a powder cloud. These powder snow avalanches

11


can be the largest of all, moving at over 300kmh, with 10,000,000 or more tonnes of snow. They can flow along a valley floor and even a short distance uphill on the other side. C Three factors are necessary for an avalanche to form. The first relates to the condition of the snowpack. Temperature, humidity, and sudden changes in weather conditions all affect the shape and condition of snow crystals in the snowpack which, in turn, influences the stability of the snowpack. In some cases, weather causes an improvement in avalanche conditions. For example, low temperature variation in the snowpack and consistent belowfreezing temperatures enable the crystals to compress tightly. On the other hand, if the snow surface melts and refreezes, this can create an icy or unstable layer. D The second vital factor is the degree of slope of the mountain. If this is below 25 degrees, there is little danger of an avalanche. Slopes that are steeper than 60 degrees are also unlikely to set off a major avalanche as they ‘sluff’ the snow constantly, in a cascade of loose powdery snow which causes minimal danger or damage. This means that slabs of ice or weaknesses in the snowpack have little chance to develop. Thus, the danger zone covers the 25- to 60-degree range of slopes, with most avalanches being slab avalanches that begin on slopes of 35 to 45 degrees. E Finally, there is the movement or event that triggers the avalanche. In the case of slab avalanches, this can be a natural trigger, such as a sudden weather change, a falling tree or a collapsing ice or snow overhang. However, in most fatal avalanches, it is people who create the trigger by moving through an avalanche-prone area. Snowmobiles are especially dangerous. On the other hand, contrary to common belief, shouting is not a big enough vibration to set off a landslide.

For questions 41–45, choose the correct heading for each paragraph A–E from the list of headings below (i-viii). There are more headings than paragraphs. Write your answers in the corresponding numbered boxes.

12


List of headings i. ii. iii. iv. v. vi. vii. viii.

Examples of Major Avalanches Stability of the Snowpack What Sets Off an Avalanche? An Expert’s Comments Steepness of Mountains Avalanche Peril An Avalanche Risk Table Types of Avalanche

1. Paragraph A

2. Paragraph B

4. Paragraph D

5. Paragraph E

3. Paragraph C

For questions 6–10, Complete the table below. Use NO MORE THAN TWO WORDS from the reading passage for each answer. Write your answers in the spaces provided. Type of avalanche

Characteristics

Loose avalanches

also known as sluffs; steep slopes; (6)____________ shape; minor risk

Slab avalanches

thick layer of snow breaks off; set very hard once they stop; cause about 90% of (7)__________

Isothermal avalanches

caused by weight of (8)__________ mixed in with the snow

Powder snow avalanches

Develop a cloud of loose snow mixed with air; (9)__________ of all types of avalanche; more fast and cover a huge distance, even travelling (10)__________

V. You are going to read an article containing reviews of recently-published books. For questions 1-10, choose from reviews (A-F). The reviews may be chosen more than once. Book Corner A round-up of the latest fiction and non-fiction from Beth Young.

13


A Reading a new novelist is a bit like asking a stranger out on a date. You never quite know if this is the start of a beautiful relationship. You check the blurbs, the publicity photograph, and flick through the book to look for the two essentials: entertainment and substance. Beginner’s Greek by James Collins is certainly big on the latter, weighing in at 400-plus pages. And the quotes on the back cover have the effect of a bunch of friends saying to you, ‘Go on, you’ll get on brilliantly’. Early indications are that this blind date could lead to a deeper relationship. Beginner’s Greek is described by The New York Times as a “great big sunny lemon chiffon pie of a novel” about romantic love amongst the American middle classes. It is indeed delicious. B In Manil Suri’s second outing The Age of Shiva we have a broad-sweeping, epic novel with an unforgettable heroine so wilful yet flawed that it calls to mind that other famous leading lady, Scarlett O’Hara in Gone With the Wind. The story begins at a firework party in Delhi where Meera falls disastrously in love. We follow her journey to Bombay, marriage and obsessive motherhood, with occasional flashbacks to a childhood that was marred by political turmoil. Mathematics professor, Suri, captures the fluidity of the role of women with a beautiful kind of precision. C Devotees of playwright David Mamet, whose screen work includes Wag The Dog and the award-winning Glengarry Glen Ross may be less than enamoured of Ira Nadel’s new biography, David Mamet: A Life in the Theatre. It may seem churlish to question the minutia of incidents that abound in this comprehensive tome, but whilst Nadel is clearly striving for accuracy one feels there ought to have been more sifting, more mining for the gold amongst the biographical trivia. In addition, Nadel’s tone is somewhat dry and academic and seems at odds with the brilliance of David Mamet’s own writing. That said, the book offers a sound introduction to the life and career of the man hailed as one of America’s most outstanding writers. D Can any Mother help me? is the true story of a desperately lonely mother who, in 1935, appealed to other women through the letters page of a women’s magazine. Writing under a pseudonym, the woman known as Ubique (meaning ‘everywhere’) little realised that she 14


would be the trigger for the launch of a new and private magazine that would last for the next fifty years. The Cooperative Correspondence Club was formed to offer comfort and support to wives, often well-educated women, who craved stimulation beyond the drudgery of family life. Jenna Bailey has done a superb job of organising and editing this compendium, adding her own insightful commentary. E Subtitled, The Life and Times of Henry Howard, Earl of Surrey, Jessie Child’s debut historical biography, Henry VIII's Last Victim, was the worthy winner of last year’s Elizabeth Longford Prize. Henry Howard’s victim status is owing to the fact that he was the final person to be executed by King Henry VIII, a mere nine days before the king himself expired. Although killed ostensibly for treason, the Earl of Surrey’s only real crime it seems was leading an unsuccessful army campaign in France. Only 29, he was also a distinguished poet with a fine literary voice, a persona which refutes his reputation as the spoilt son of the Duke of Norfolk. F This is the 25th outing for T. Keneally but he’s lost none of his writing powers. The Widow and Her Hero takes real life events during the Second World War as its inspiration and builds a tale of love and intrigue. Grace looks back on her life to recall her courtship with the hero of the title, the handsome Captain Leo Waterhouse. Leo is tragically killed whilst on a secret mission but it is many years before Grace discovers the facts about his death. Keneally made fans galore when Schindler’s Ark was published and later made into the award-winning Steven Spielberg film, Schindler’s List. The Widow and Her Hero will bring him even more fans. In which review are the following mentioned? 1) A story in which someone is unaware of the impact of their action. _________ 2) A description of the opening scene. _________ 3) An author who exemplifies source material with their own analysis. _________ 4) A humorous comparison with a real-life situation.

_________

5) A character who finds out the truth about a situation. _________ 6) A hint that the author’s future writing career will be positive. _________ 15


7) A book that would be appreciated by people without much previous knowledge of the subject. _________ 8) A book which has already won critical acclaim. _________ 9) A book which includes too much factual detail. _________ 10) A mention of the profession of the author. _________ SECTION 4. WRITING I. Read the following extract and use your own words to summarise it. Your summary should be about 140 words long. ( 15 points) Scattered through the seas of the world are billions of tons of small plants and animals called plankton. Most of these plants and animals are too small for the human eye to see.They drift about lazily with the currents, providing a basic food for many larger animals. Plankton has been described as the equivalent of the grasses that grow on land. However, plankton is richer in food value than the land grasses. One scientist has estimated that while grasses of the world produce about 49 billion tons of valuable carbohydrates each year, the sea’s plankton generates more than twice as much. Despite its enormous food potential, little effort was made until recently to farm plankton as we farm grasses on land. Now, marine scientists have begun to study this possibility to look for different means of feeding an expanding world population. One type of plankton that seems to have great harvest possibilities is a tiny shrimp-like creature called krill. Growing to two or three inches long, krill provide the major food for the giant blue whale, the largest animal ever to inhabit the Earth. Krill swim about just below the surface in huge schools mainly in the cold Antarctic. Because of their pink colour, they often appear as a solid reddish mass when viewed from a ship or the air. Krill are very high in food value. A pound of krill contains about 460 calories. If the krill can feed such huge creatures as whales, many scientists reason, they must be certainly a new food source for humans.

16


II. The figure shows demographic trends in Scotland. Summarise the information by selecting and reporting the main feature. You should write about 150 words. (15 pts)

III. The government should allocate more funding to teaching science rather than other

subjects in order for its country to develop and progress. To what eextent xtent do you agree? In about 300 - 350 words, write an essay to express your opinion. ( 30 points) Người ra đề Nguyễn Thị Bích Vân ( 0904114546)

17


HỘI CÁC TRƯỜNG CHUYÊN VÙNG DUYÊN HẢI VÀ ĐỒNG BẰNG BẮC BỘ

ĐỀ THI CHỌN HỌC SINH GIỎI LẦN THỨ XII MÔN: TIẾNG ANH - KHỐI 11

TRƯỜNG THPT CHUYÊN NGUYỄN TRÃI TỈNH HẢI DƯƠNG

Thời gian: 180 phút Đề thi gồm: 8 trang

ĐÁP ÁN

SECTION 1: LISTENING (50 points) I. You will hear an interview with a British politician. For questions 1-5, choose the answer (A, B, C or D) which fits best according to what you hear. 1. B

2. A

3. D

II.

Listen to the conversation. Write T for True or F for False for each statement.

1T

2F

3F

4. C

5.C

4F

5F

III. You will hear a guide speaking to tourists who are visiting some Romans remains. Listen and give short answers to the questions. Write NO MORE THAN THREE WORDS AND/OR A NUMBER taken from the recording. (10 points) 1. 79/ seventy-nine 2. To establish position 3. Treasure 4. Once a year 5. Loose stones, walls IV.Listen to a piece of news about How Japan's economy is destroying Its Youth. WRITE NO MORE THAN THREE WORDS OR A NUMBER 1. phenomenon of isolation 4. hire fresh 7. Effeminate 9. Shift

2. the fabric 5. sexual relationships 8. influx of socially 10. associating

SECTION 2: LEXICO - GRAMMAR (30 points) I. Choose the word or phrase that best fits the gap in each sentence 1.C 2. B 3. A 4. D 5. B 6. A 7. B 8. B 9. D 10. B

3. massive impact on 6. puts a strain


II. The passage below contains 5 errors. IDENTIFY and CORRECT them. Write your answers in the space provided in the column on the right. Line

Error

Correction

1

exhibitions

Exhibits

8

those

That

10

individuals

Individual

12

the

An

17

against

From

III. Fill in each blank with an appropriate preposition or particle. 1. In 2. With 3. On 4. Out 5. On IV. Supply the correct form of the verbs in block capitals in brackets to complete the passage. 1. assertion 6. distaste

2. incredible 7. personage

3. disbelief 8. lowest

4. erroneous 9. authoritative

5. financier 10. reasoning

SECTION 3: READING (60 points) I.

Read the text below and decide which answer (A, B, C or D) best fits each gap.

1. D 6.C

2. B 7.D

II.

3.B 8.C

4.D 9.D

5.B 10.B

Fill each gap in the passage below with ONE appropriate word in the space provided. 1.stimulate

2. highlighting

3. Highlighting

4. Means

5. exhibits

6. Taste

7. Inspire 8. Express

9. Highly

10. Recover

III. Read the following text and choose the best answer. 1. B

2. C

3. D

4. B

5. A

6. D

7. C

8. C

9. B

IV.Read the following extract and answer questions 1–10. 1. vi 6. teardrop

2. viii 7. deaths

3. ii 8. water

4. v 9. largest

5. iii 10. uphill

10. A


V. 1.D 2.B 3.D 4. A 5. F 6. A 7. C 8. E 9. C 10. B SECTION 4. WRITING (60p.) I.

Write a summary of maximum 140 words. (15p.) The text demonstrates the food potential of plankton and describes a specific type of the creature called krill. Plankton, most of which are invisible to naked eyes, are integral to many animals’ diets. In such regard, it far surpasses its dry land equivalent, grass, producing nearly 100 billions of carbonhydrates annually. Despite such potential, only recently has the farming of these organisms received widespread attention to help feed the growing population. Krill, a tiny redcoloured shrimplike kind of plankton are among the most promising subjects for cultivation due to the amount of calories they contain. These creatures travel in huge schools in Antarctic and are the food for the largest animal on Earth, the giant blue whale.

1. Completion: 2 pts 2. Content: 7 pts a. Cover the main information in the passage. 3. Organisation: 3 pts b. The ideas are well organized. c. The description is in one paragraph. 4. Language: 7 pts d. Use a wide range of vocabulary and structure e. Good grammar 5. Punctuation and spelling: 1 pt II.

Report writing (15p.) 1. Completion: 2 pts 2. Content: 7 pts a. Cover the main information in the table yet not go into too many details. b. Make general remarks and effective comparisons. 3. Organisation: 3 pts c. The ideas are well organized d. The description is sensibly divided into paragraphs 4. Language: 7 pts e. Use a wide range of vocabulary and structure f. Good grammar 5. Punctuation and spelling: 1 pt


III. Write an essay (30p.) 1. Completion: (3 pts.) 2. Content: (10 pts) 3. Provide relevant and convincing ideas about the topic, supported by specific examples and/or reasonable justification. 4. Organization: (10 pts) a. Ideas are well organized and presented with unity, cohesion and coherence. 5. Language: (10 pts) b. Demonstrate of a wide range of vocabulary and structures. c. Good use of grammatical structures. d. Present the ideas with clarity. 6. Punctuations and spelling. (2 pts)

Part1 PROFICIENCY PRACTICE TEST


part 2


You will hear a guide speaking to tourists who are visiting some Romans remains. Listen and give short answers to the questions. Write NO MORE THAN THREE WORDS AND/OR A NUMBER taken from the recording. (10 points) Guide: Good morning and welcome to this morning’s guided tour of the Roman remains in the area. We will start our day by visiting the garrison town of Corbridge- then, this afternoon, we will be visiting part of the wall which the Romans built to keep their enimies out of the country. The part we are going to visit is remarkably well-preserved and it is easy to imagine what it must have been like all those years ago. The Romans actualy arrived in this country in the earlier part of the first century AD, but it wasn’t until the year 79 tht they came to the Corbridge area. They builta series of forts and strongholds so that they could estabish their position in the whole region. What YOU ARE GOING TO SEE THIS MORNING ARE THE RESULTS of EXCAVATIONS WHICH HAVE taken place fairly recently. The first early archaeological dig was as far back as 1201 and then, believe it or not, they were looking for treasure. Unfortunately, or perhaps fortunately for us, nothing interesting was found, so they abandoned the site and it was not until the early nineteenth century that another and much more ambitious, excavation was carried out. The result was a number exciting discoveries. And in the mid-nineteenth century , they discovered part of a Rome bridge and other structures, inlcuding the baths and other dwellings in the town .More recently, in fact every since 1934, digs have taken place. These have resulted in the discovery of a large collection of scuplture, coins, potttery and small objects; and some of these have become very famous indeed. Most of them are nore housed in the site mueum, which we will be visiting before we visit the site itself. Unfortunately, for security reasons, some have had to be taken to the city museum, which you will find is about 15 miles away. As you can see from this map, the site museum has laid out in a circle. The first alf concentrates on the history. You will aslo find display cases of objects which would have been in use at the time. In the second half of the museum, each section’s related to a particular aspect of life in the town, such as trade, domestic life and religious observance. After isiting the museum we will be walking round the site, and we would ask you please, to pay particular attention to any loose stones lying around the site. And keep children off the walls, as these can be extremely dangerous. At the end of the tour we will be paying a visit to the gift shop and café ,which you probably saw as you came in- near the site entrance and the car park. Now, if you have left hats and sunglasses in the coach. I’d advise you to go and retrieve them now as the coach will be locked for the next few hours and the sun is very hot today.


Part 3

In this week’s edition of Amazing Feats we are reporting on the extraordinary achievement of the climber, Annabelle Bond. It took a huge physical effort, and a considerable amount of money, but Annabelle Bond has just flown back to her home in London after becoming the fastest woman ever to climb the highest mountain in each of the seven continents. The feat took her to Nepal and Tanzania, Argentina, Alaska and Antarctica, Australia and Russia, with the odd avalanche in between. She scaled the summits in 360 days. Not only is this the fastest time ever for a woman but it’s also the fourth fastest for a climber of either sex. According to Annabelle, the best experience of all was seeing the top of Everest. Annabelle Bond is the daughter of Sir John Bond, the head of the bank HSBC. She was born in Singapore in 1969 and grew up in Jakarta and Hong Kong, coming to Europe at the age of eight to attend boarding school, first in England and then in Switzerland. Her grandmother, Christine, was a pioneering female climber in the 1920s. Returning to Hong Kong in 1991, Ms Bond spent the next eight years working as an estate agent. It was then that she started running mountain half-marathons and setting records, fitting her training in between her appointments with clients. Initially, she had been the kind of person who did not think that she could run for more than an hour. However, she finally decided that buying and selling houses was not what she really wanted to do and in 2000, she left the estate agent’s where she’d been working and headed for Sun Valley, Idaho, to ski and climb. A trip to Everest base camp, where she fell madly in love with the dramatic scenery, further whetted her appetite for climbing and she decided to attempt to climb the highest peak in each of the seven continents: Mount Everest in Asia, Mount Elbrus in Europe, Aconcagua in South America, Mount McKinley in North America, Kilimanjaro in Africa, Mount Kosciuszko in Australia and Vinson Massif in Antarctica. Ms Bond was twice admitted to hospital with frostbite and said she felt lucky to survive Everest after two climbers who joined their party died on the descent. In Ms Bond’s view, however, the biggest problem had been keeping to the schedule, as each mountain had to be climbed in the right season. It was also a challenge to stay fit and healthy while living on little other than freeze-dried food. She said the ascents had raised £850,000 for the Eve Foundation, a charity that was set up in memory of a friend who died of cancer. With the money she raised she was also able to pay for the education of the children of her Sherpas in Nepal. She is swift to point out that it has all been something of a team achievement, the climbs being completed with guides from a New Zealand adventure company and filmed by a cameraman. However, fellow climbers agree that getting up Everest is a feat in itself. Tom Prentice, the author and former editor of Climber magazine, said that it was a fantastic achievement for anyone to have climbed the seven highest summits on all the continents. He added, however, that there were many much harder peaks than some of the summits on the list. Ms Bond is aware that she has had ‘mixed reviews’ in the climbing world but she’s been impressed by the lack of chauvinism she has encountered and said everyone had been very supportive. When she returned to London, she said-- she was happy to be back but was feeling a bit jetlagged. The pleasures of life back in London for Ms Bond are those of home comforts like a shower and a loo, and seeing family and friends again after nearly a year living mainly in tents. So, what next for Annabelle Bond? She’s in demand as a celebrity speaker, but says that she isn’t sure quite what she will be doing in the future. Although she says she certainly isn’t rushing off to climb any mountains, she nevertheless wants to do something that will push her to the limit again. Watch this space.


Bond’s view, however, the biggest problem had been keeping to the schedule, as each mountain had to be climbed in the right season. It was also a challenge to stay fit and healthy while living on little other than freeze-dried food. She said the ascents had raised £850,000 for the Eve Foundation, a charity that was set up in memory of a friend who died of cancer. With the money she raised she was also able to pay for the education of the children of her Sherpas in Nepal. She is swift to point out that it has all been something of a team achievement, the climbs being completed with guides from a New Zealand adventure company and filmed by a cameraman. However, fellow climbers agree that getting up Everest is a feat in itself. Tom Prentice, the author and former editor of Climber magazine, said that it was a fantastic achievement for anyone to have climbed the seven highest summits on all the continents. He added, however, that there were many much harder peaks than some of the summits on the list. Ms Bond is aware that she has had ‘mixed reviews’ in the climbing world but she’s been impressed by the lack of chauvinism she has encountered and said everyone had been very supportive. When she returned to London, she said-- she was happy to be back but was feeling a bit jetlagged. The pleasures of life back in London for Ms Bond are those of home comforts like a shower and a loo, and seeing family and friends again after nearly a year living mainly in tents. So, what next for Annabelle Bond? She’s in demand as a celebrity speaker, but says that she isn’t sure quite what she will be doing in the future. Although she says she certainly isn’t rushing off to climb any mountains, she nevertheless wants to do something that will push her to the limit again. Watch this space.

part 4 In Japan, nearly a million young men have locked themselves in their bedrooms, and refuse to go outside. This phenomenon of isolation has been attributed as a side effect to Japan’s highly competitive economy and culture. So, how is Japan’s economy destroying their youth? Japanese culture generally places a significant amount of emphasis on education and employment. But in recent years, Japan has seen a significant downturn in their economic growth. However, Japan’s recession isn’t the same as in most other countries. Stores don’t go out of business, luxury spending continues, the fabric of society still seems comfortable. But the lack of available new jobs does have a massive impact on younger generations. With fewer jobs to go around, many Japanese men give up hope and withdraw from society. Furthermore, the longer one is unemployed, the harder it is to find a job, since companies tend to hire fresh out of high school and college. Young adults and adolescents who confine themselves for long periods of time are called “hikikomori”. They are among the generation of youths called “grass eaters”, which are mostly men who shy away from sexual relationships and general social contact. They tend to live at home with their parents, some well into their 40s. Additionally, most Japanese youths are expected to provide for their families, putting tremendous stress and pressure on them to succeed. The weak economy also puts a strain on relationships, as many Japanese women refuse to date unemployed men, considering them weak and effeminate. Japan’s government has warned that more than one and a half million are on the verge of becoming hikikomori. An estimated 700,000 hikikomori have already locked themselves away. The influx of socially anxious men has created big changes for Japan, including a massive drop in birth rates to record lows, and a shift in consumer habits away from a more “masculine-driven” culture. Studies have shown that as much as 75% of men in their


20s and 30s consider themselves “grass eaters�. Considering the depth of culture associating success and hard work with self identity in Japan, it will take significant social effort and a better economy to improve this epidemic.


KỲ THI HỌC SINH GIỎI CÁC TRƯỜNG THPT CHUYÊN KHU VỰC DUYÊN HẢI VÀ ĐỒNG BẰNG BẮC BỘ LẦN THỨ XII, NĂM 2019 ĐỀ THI MÔN: TIẾNG ANH 11 Thời gian: 180 phút (không kể thời gian giao đề) Ngày thi: 20/4/2019 ( Thí sinh làm bài trực tiếp vào đề) (Đề thi gồm 17 trang)

ĐỀ CHÍNH THỨC

Điểm Bằng số

Bằng chữ

Giám khảo 1

Giám khảo 2

Số phách

A. LISTENING (50pts) HƯỚNG DẪN PHẦN THI NGHE HIỂU  Bài nghe gồm 4 phần; mỗi phần được nghe 2 lần, mỗi lần cách nhau 05 giây; mở đầu và kết thúc mỗi phần nghe có tín hiệu. Thí sinh có 20 giây để đọc mỗi phần câu hỏi.  Mở đầu và kết thúc bài nghe có tín hiệu nhạc.  Mọi hướng dẫn cho thí sinh (bằng tiếng Anh) đã có trong bài nghe. Part 1: You will hear part of a discussion on a current affairs programme between Nick Barnes and Alison Tempra about the performance of the company Facebook since it floated on the stock exchange, hosted by Emily Dunne. Choose the answer (A, B, C or D) which fits best according to what you hear. (10 pts) 1. What does Alison think is cause for optimism? A. The company kept its costs low. B. The loss generated was less than expected. C. There appears to be good revenue potential. D. The company hasn't started to advertise yet. 2. According to Nick, the increasing popularity of smaller devices ______. A. represents untapped potential for Facebook. B. is a significant challenge to Facebook increasing its revenue. C. puts Facebook at a competitive advantage. D. gives the company an opportunity to advertise more. 3. In what situation does Alison believe Facebook users might abandon the company? A. if they are given the option of watching adverts on the certain apps and sites B. if a free social network becomes available on the net C. if the company pushes advertisements onto users too forcefully D. if sites and apps start to appear which put users of using Facebook Page 1 of 17


4. What do we learn about the company's performance? A. The share price has now dropped by over one-third. B. There has been a 6% improvement in the share price overnight. C. $38 has been wiped off the share price. D. It has become the biggest flop in history. 5. Nick believes that Google _______. A. will inevitably prevail over Facebook in time. B. was short-sighted to invest everything it had into one project. C. technology will be made redundant by what Facebook offers users. D. will become profit-making in a matter of time. Your answers: 1. 2. 3. 4.

5.

Part 2: You will hear the minister talking about crime. Are these statements true or false? Write T (true) or F (false). (10 pts) 6. Peter Miles appointed a new head of the police service. 7. The government has spent a lot of money on CCTV. 8. The government copied the Make Amends scheme from another country. 9. More people get mugged now than ten years ago. 10. The figures in the report are completely accurate. Your answers: 6.

7.

8.

9.

10.

Part 3. You will hear a student called Tina asking Professor Van Diezen for advice on choosing courses. Listen and answer the following questions, using NO MORE THAN FIVE WORDS for each answer. (10 pts) 11. What is the defining characteristic of a specialised course? _________________________________________________________________________ 12. For whom the Microbiology courses are available? _________________________________________________________________________ 13. Who are interested in Microbiology courses? _________________________________________________________________________ 14. Why will a Medical Science course be opened next year? _________________________________________________________________________ 15. Which is the quickest increasing subject in enrolment? _________________________________________________________________________ Part 4. Listen to the news and fill in the following blank with a suitable phrase. Write NO MORE THAN THREE WORDS in each blank. (20 pts) Today the U.S. Supreme Court delivered a (16) __________ to American business and an equally sweeping defeat to American workers. The (17) __________upheld and extended the growing practice adopted by American businesses, namely requiring workers to agree as a condition of employment not to go to court over wage and hour disputes but to instead submit their claims to (18) __________ individually. Lawyer Ron Chapman, who represents management in (19) __________, said he expects small and large businesses alike to immediately move to impose these binding arbitration contracts in order to eliminate the fear of costly class-action (20) __________ from juries.

Page 2 of 17


The lead (21) __________ in the case was an IT worker at Epic, the giant health care software development company. That law guarantees the rights of workers to engage in activities for the purpose of (22) __________ or any other concerted activities. Justice Ruth Bader Ginsburg, in a (23) __________ from the bench, called out the majority for what she said was an egregiously wrong decision. Labor law experts said today's decision likely will present increasing problems for the #MeToo movement and for other civil rights class actions claiming discrimination based on (24) __________. There's no transparency in most binding arbitration agreements, and they often include (25) __________. Yale law professor Judith Resnik observes that today's decision applies to all manner of class actions. Your answers: 16. 21. 17.

22.

18.

23.

19.

24.

20.

25.

B. LEXICO AND GRAMMAR (30 points) Part 1. Choose the answer A, B, C, or D that best completes each of the following sentences. Write your answers in the corresponding numbered boxes. (10 points) 1. I’m sure there’s a definite __________ of envy in her nasty comments about you. A. factor B. ingredient C. component D. element 2. She had an unclear __________ of him in her mind, although she knew he would be old. A. sight B. picture C. vision D. figure 3. Derek had no experience of white-water canoeing, so it was extremely__________ of him to try and shoot the rapids. A. hazardous B. intrepid C. perilous D. foolhardy 4. Money was short and people survived by __________ and saving. A. scrimping B. scavenging C. scouring D. scrounging 5. The matter has been left in __________ until the legal ramifications have been explored. A. recess B. suspension C. abeyance D. waiting 6. He was absolutely __________ with anger when he found that I had scratched his car. A. burned B. carmine C. fickle D. livid 7. In these times of high unemployment everyone thought my giving up my job was______ madness. A. sheer B. steep C. high D. deep 8. Most teenagers go through a rebellious __________ for a few years but they soon grow out of it. A. stint B. span C. duration D. phase 9. I knew my mother would __________ a face the minute she saw my new hair cut. A. drag B. lift C. pull D. race 10. The president was eventually __________ by a military coup. A. disposed B. despised C. deposed D. dispersed Your answers: 1. 2. 3. 4. 5. 6. 7. 8. 9. 10. Page 3 of 17


Part 2. The passage below contains 5 mistakes. Identify the mistakes and write the corrections in the corresponding numbered boxes. There is an example at the beginning. (5 points) 1 2 3 4 5 6 7 8 9 10 11 12 13 14 15

The key ingredient in the body’s psychological response to danger is adrenaline. The body produces this chemical in the center of the adrenal glands atop the kidneys. When a physically or mentally stressful situation rises, a flood of adrenaline into the blood flow prepares the body to act swiftly and forcefully to protect it. The heart beats faster. Blood is directed away from the skin and toward such structures as the skeletal muscle and the brain- all to provide the oxygen necessary to run fast, lift heavy objects, and think quickly. This physiological reaction to risk is the wellknown “flight- to- flight”. Although activated by a threat, the sudden release of adrenaline and the body’s responses to it produce a distinctively pleasurable feeling once the danger has passed. Even people who are not seeking danger but who confront it accidentally will speak afterwards or an exciting “adrenaline rush.” To some people, the pleasure of such an experience is so intensive that they rate it among life’s most desirable sensations. This hints at a likely biochemical- psychological mechanism motivating some, perhaps most, risk-seeking individuals.

Your Answers: Number 1 2 3 4 5

Line

Mistake

Correction

Part 3. Complete each of the following sentences with a suitable preposition or particle. Write your answers in the space provided. (5 points) 1. Don’t try to palm me ______ with your lies and excuses. 2. He is a leader __________ name only: his deputy has effectively taken over. 3. This will go __________ in history as the most important event of the century. 4. It might be a good idea to butter __________ the boss before you ask for a rise. 5. My mum draws heavily __________ her teaching experience each time she gives a presentation. Your answers: 1.

2.

3.

4.

5.

Part 4. Give the correct form of each bracketed word in the following passage. Write your answers in the space provided. (10 points) The University of Southern California rescinded the admissions of a half-dozen students, and several other colleges and universities pledged to take a closer look at their admissions processes as (1. FALL)_____________continued from an admissions scandal that implicated coaches, athletic department administrators and 33 wealthy, well-connected parents who allegedly schemed to get their children admitted to prestigious colleges. A (2. CLASS) _____________ lawsuit was filed Wednesday on behalf of applicants who were denied admission to several universities affected by the scandal. It alleged that those schools failed to take adequate steps to (3. SAFE) _____________ against fraud, Page 4 of 17


depriving the applicants of a fair shot. And it emerged that the genesis of the FBI investigation came when an investor tipped off agents to the admissions scheme after he was caught (4. COMMIT) _____________ securities fraud. On Tuesday, 50 people were charged or indicted in connection with the scheme, including actors Felicity Huffman and Lori Loughlin and Loughlin’s husband, clothing designer Mossimo Giannulli. Other parents charged include the owner of a Napa Valley (5. WINE)_____________, the wife of an NFL legend, (6. RANK) _____________ lawyers, private equity investors, real estate moguls and a media company owner. Questions remain about what will happen to the college students who were admitted under circumstances being scrutinized by the FBI. According to the criminal complaint in the case, their parents paid hundreds of thousands of dollars to submit fraudulent test scores and fake athletic (7. CREDENCE) _____________ on their behalf. Prosecutors say that with the help of a corrupt college consultant, the parents paid off coaches so their children could pose as athletic (8. RECRUIT) _____________, allowing them to attend selective schools despite (9. LACK) _____________ academic records. Court documents allege the man at the (10. CENTER) _____________of the scandal, William “Rick” Singer, has been helping parents get their children into selective schools this way since at least 2011. Your answers: 1. 2. 3. 4. 5.

6. 7. 8. 9. 10.

C. READING (60 points): Part 1. Read the following passage and decide which answer (A, B, C or D) best fits each gap. Write your answers in the corresponding numbered boxes. (10 points) OK The word OK is ubiquitous in modern English but its origins remain (1) __________ in mystery. Over the years, many theories have been (2) __________ regarding its derivation but none of them is (3) __________ convincing. The first recorded written use of OK was in 1839, when it appeared in a newspaper article in Boston, Massachusetts. There was a (4) __________ for wacky acronyms at the time, just as today's text messages use things like 'LOL', and OK allegedly originated as a misspelling of All Correct. But (5) __________ many of these acronyms flourished briefly and then gradually (6) __________ out of use, OK has proved to be remarkably (7) __________. It first reached England in 1870, where it appeared in the words of a popular song, and today is in (8) __________ use across the English-speaking world. As part of a phrase ' …. rules OK', it has been a mainstay of urban graffiti since the 1930s and in 1969 it had the (9) __________of being the first word spoken on the moon. In short, it's a (10) __________ useful word. 1. A. shrouded B. smothered C. clothed D. draped 2. A. laid out B. put forward C. drawn up D. brought about 3. A. widely B. mainly C. wholly D. largely 4. A. hype B. craze C. rage D. whim 5. A. despite B. albeit C. whereas D. providing 6. A. slipped B. faded C. crept D. strayed 7. A. resolute B. stalwart C. steadfast D. resilient Page 5 of 17


8. A. staunch 9. A. credit 10. A. phenomenally Your answers: 1. 2.

3.

B. relentless B. pride B. ordinarily

4.

C. durable C. honour C. conclusively

5.

6.

7.

D. constant D. acclaim D. controversially

8.

9.

10.

Part 2. Read the text below and think of the word which best fits each space. Use only one word in each space. Write your answers in the space provided. (15 points) Over the last hundred years, much of the art of using herbs in cooking and medicine has been lost, especially in industrialised societies. Until recently, few people in the crowded cities had the space to grow plants or vegetables, and so (1) __________ in the country did knowledge of herbs linger on. (2) __________ the advent of refrigeration, however, which meant that the strong smell of old meat no longer had to be disguised, and the appearance of packaged food and easily-available medicines, the growing of herbs declined rapidly. Nowadays, there is hardly anyone who does not have a small patch of garden, or a (3) __________ sill or balcony or balcony large enough for a pot or two of herbs. These facts, (4) __________ with the beginnings of a revolt (5) __________ standardised foods and perhaps also a mistrust of the (6) __________ effects of some of today's medicines, mean that herbs have taken (7) __________ a new popularity. The culinary uses of herbs are endless and they can be used to good effect all year (8) __________, in dried form or cut fresh. (9) __________ aids to beauty and for medicinal (10) __________, there is now a vast range of available. Herbs are for all occasions and all seasons. Your answers 1. 2. 3. 4. 5. 6.

7.

8.

9.

10.

Part 3: Read the following passage and choose the best answer. Write your answers in the corresponding numbered boxes. (10 points) AROUND THE WORLD IN 94 DAYS In February 2001, at the age of 24, Ellen Mac Arthur became the youngest and fastest ever woman to sail round the world. After 94 days alone on board her yacht Kingfisher, she finished second to Michel Desjoyaux of France in the single-handed Vende Globe event. In sport, like life, the winner is usually fĂŞted, and runners-up quickly forgotten. This time the roles were reversed and it was Ellen, weighing just 50 kilos and barely 1m60 tall, that really captured people's imaginations and emotions. One newspaper in France, where she was and is a real heroine, summed up the national mood there with the headline 'Well done, Michel, brave Ellen'. As with many spectacular achievers, the signs were there from an early age, even in the unpromising nautical terrain of landlocked Derbyshire. Her great-grandparents were sailing people and a great-uncle was a merchant seaman, but any real link with the sea is tenuous. There was, however, aunt Auntie Thea who lived on the east coast of England and had a 26-foot sailing boat called Cabaret. It took just one trip on the open sea with her aunt to spark off Ellen's lifelong passion. She was eight years old. After that she began saving her pocket money and spent all her spare time reading sailing books in the library, absorbing

Page 6 of 17


information like a sponge. With her savings and the help of her grandmother she bought an 8-£00t fibre glass dinghy, and from that moment on there was no keeping her away from the water. Sailing round Britain single-handed at the age of 18 was just the start; Ellen had long since set her sights on the Vende. But finding the money to undertake round-the-world voyages is no easy feat. She wrote 2,000 letters requesting sponsorship and received just two replies, from the Kingfisher company who were looking to expand into France. And in terms of race preparation, if thoroughness was the key to success, Ellen could certainly be considered one of the favourites. In the eight months leading up to the start of the race, she sailed no fewer than 60,000 miles at the helm of her 60-£001 Kingfisher, far more than the rest of the fleet put together in the same period. During her three months at sea MacArthur negotiated deadly icebergs, gigantic waves and gale-force winds. She endured the freezing cold of the Antarctic and suffered the blistering heat of the windless doldrums. Racing conditions meant sleeping in 10-minute bursts, a survival suit that stayed on for weeks at a time and hands and wrists covered in sores and cuts. Food was dried or frozen. Water came from a desalinator, which passes sea water through a membrane. 'You don't really wash in the icy waters of the southern ocean,' she laughs. 'Anyway, there's no one to tell you that you smell.' As Kingfisher crossed the finishing line Ellen was surrounded by hundreds of spectator boats and a cheering crowd of 200,000 lined the shore. Stepping off her yacht she looked remarkably composed and seemed to take the change from solitude to public adulation very much in her stride. Her thoughts, she later confessed, were on the realization that she had fulfilled the ambition that had dominated her life for the previous four or five years. 'Throughout that time my sole focus had been crossing the finishing line, and in the fastest possible time.' Now she could savour that moment. But despite MacArthur's belief that everyone who finishes the Vende is a winner, she still feels a sense of disappointment that, having taken the lead from the eventual winner Michel Desjoyaux 10 days from the finish, she did not quite have the energy or good fortune to turn her advantage into victory. 'You have to believe you can win from the start,' she asserts. 'Deep down you're a competitor, you don't climb the mast and come back black and blue just for a cruise. You do it because it's a race.' The public will now be hoping to see a suitable encore, some new feat of endurance to justify her celebrity status. For Ellen can no longer claim, as she did in her post-race press conference, to be the simple Derbyshire girl with 'no mobile, no credit cards, no money, no nothing'; she is a heroine and an inspiration to others of her generation. As if to reinforce this, and despite her reluctance to take on this role, she later commented: 'If there's one thing I've learned in this past year, it's that deep down in your heart, if you have a dream, then you can and must it happen.' 1. The word “fêted” in the second paragraph means A. to make somebody pleasant. B. to praise somebody. C. to make somebody happy. D. to give somebody a reward. 2. At the time of her achievement we learn that Ellen A. enjoyed only short-lived success. B. was more famous in France than anywhere else. C. attracted more attention than Michel Desjoyaux. D. became popular because of her size. 3. The word “landlocked” in the third paragraph means A. having no seaport. B. having no fresh water . C. having no land. D. having no sea coast. Page 7 of 17


4. Where did Ellen's initial interest in sailing come from? A. She came from a family of sailing enthusiasts. B. She went to see one of her relatives. C. She read widely on the subject. D. She lived near the sea. 5. The word “single-handed� in the fourth paragraph means A. without any help from anyone else. B. using only one of one's hands to row. C. on a boat with only one paddle. D. on a boat with only one sail. 6. What do we learn about Ellen at the start of the race? A. People thought she had a very good chance of winning. B. She was a more experienced sailor than the other racers. C. She had been waiting for this moment since she was 18. D. She had gone to great lengths to achieve her ambition. 7. The writer suggests that one cause of discomfort for Ellen at sea was A. the shortage of water. B. her failure to sleep. C. extremes of temperature. D. a lack of cooking facilities. 8. According to the writer, when Ellen finished the race, she was A. overwhelmed by her new-found fame. B. surprised by the number of people who came to greet her. C. able to reflect on her achievement. D. delighted to be amongst people again. 9. According to the writer, Ellen A. thinks she deserved to win the race. B. has mixed feelings about the outcome of the race. C. knew she would win the race. D. thinks Michel Desjoyaux was lucky to beat her. 10. Which of the following views does the writer express in the last paragraph? A. She has the power to motivate. B. She has no right to fame yet. C. Her comments lack depth. D. She needs to change her lifestyle. Your answers 1. 2.

3.

4.

5.

6.

7.

8.

9.

10.

Part 4. Read the passage and do the tasks that follow. Write your answers in corresponding numbered boxes. (10 points) WHERE HAVE ALL OUR BIRDS GONE? People have been listening to skylarks singing in Britain for 10,000 years. But now they, and many other much loved species, are vanishing fast. David Adam finds out why. A family of Starlings has chosen a post box for the third year running in an Essex seaside town to raise their young brood. A. The B1042 that winds from the Bedfordshire town of Sandy towards the village of Potton is a difficult road to cross. Fast and twisty, there are several blind bends where pedestrians must take their lives into their hands. That is trickier than it sounds, for most pedestrians who cross the B1042 already have a pair of binoculars in their hands. The road separates the grand headquarters of the RSPB, home to hundreds of birdwatchers, from

Page 8 of 17


some unkept fields, home to hundreds of watchable birds – hence the regular skips across the tarmac. The skips, though, are now less regular for many RSPB staff, for the star attraction of the neighbouring fields has flown. Until a year ago, a clutch of woodlark nested there, one of Britain's rarest birds with just 1,000 or so thought to remain. Then their home was ploughed up and replaced with a giant field of swaying hemp plants. The woodlark have not been seen since. B. It is not just the professional birdwatchers of the RSPB who have seen their local landscape transformed. Across Britain, and with little fanfare, the face of the countryside has subtly changed in recent years. Farm fields that stood idle for years under EU schemes to prevent overproduction, such as the one across the road from the RSPB, have been conscripted back into active service. The uncultivated land, previously a haven for wildlife, has been ploughed, and farmers have planted crops such as wheat and barley, with occasional hemp for use in paper and textiles. C. As a result, the amount of land available for birds such as the woodlark has halved in the last two years. Without efforts to stem this loss of habitat, conservation experts warn that the countryside of the future could look and sound very different. Starved of insects in the spring and seeds through the winter, the metallic-sounding corn bunting and plump grey partridge, formerly one of the most common birds on UK shores, are on the brink. And the skylark, whose twittering has provided the soundtrack to millions of countryside walks and inspired Percy Bysshe Shelley, in Ode to a Skylark, to praise its "profuse strains of unpremeditated art", is struggling and could soon vanish from many areas. Numbers fell 53% from 1970 to 2006. "This is not just about birdwatchers. These birds are part of our common heritage," says Gareth Morgan, head of agriculture policy at the RSPB. D. Government figures show that populations of 19 bird species that rely on farmland have halved since serious counting started in the 1970s – a decline conservationists blame on intensive farming methods, with insecticide and herbicide sprayed on to monoculture fields shorn of vibrant hedges. The unmistakable yellowhammer, which likes to sing while perched as a dash of colour on hedges and bushes, has steadily disappeared with the hedges and bushes. And a startling 80% drop across England in 40 years has diluted the shifting Rorschach blots painted on the dusk sky by massed flocks of starling – though urban changes are blamed for this too. E. Farmland birds may sound a niche problem, and you may think that the rest of the countryside is doing OK, but for most people, farmland is the British countryside. About 75% of Britain is farmed, and about half of that is arable fields. Take a train between two UK towns, particularly in eastern counties, and almost all of the countryside you see is farmland. F. As Simon Gillings of the British Trust for Ornithology (BTO) puts it: "For most people, farmland is the countryside and farmland birds are the birds they see." If birds are struggling, then it is a fair bet that other wildlife is too. "Birds are indicative of other things," Gillings says. "If birds are declining then what does that say about the plants and insects they rely on? It's all linked together." Page 9 of 17


Questions 1- 6 The Reading Passage has six paragraphs, A-F. Choose the correct heading for each paragraph from the list of headings. Write the correct number i-ix in boxes 1-6. List of Headings i. Not only birds suffer ii. Vanishing of habitats gives rise to the drop in bird species. iii. Cultivating fame fields is profitable for farmers iv. A niche and minor problem v. Who should be blamed? vi. Woodlark and other birds are on the brink vii. Hedges and bushes are blamed for the reduction viii. The rapid disappearance of bird species in Britain ix. The countryside is the farmland x. A major change in local landscape – more land is cultivated. xi. Farmland is taking an insignificant share. 1. Paragraph A __________ 2. Paragraph B __________ 3. Paragraph C __________ 4. Paragraph D __________ 5. Paragraph E __________ 6. Paragraph F __________ Your answers: 1. 2.

3.

4.

5.

6.

Questions 7-10 Do the following statements agree with the information given in the READING PASSAGE? Write: YES NO NOT GIVEN

if the statement agrees with the information if the statement contradicts with the information if there is no information on this

7. The RSPB is a very famous international organisation in birds conservation. 8. In EU countries farm fields are left uncultivated in order to increase their fertility. 9. The loss of habitats means a more demanding survival for many bird species. 10. In the 1970s governments only counted 19 bird species that depended on farmland. Your answers 7.

8.

9.

10.

Part 5. Read the text, identify which section A–F each of the following is mentioned. Write ONE letter A–F in the corresponding numbered space provided. Each letter may be used more than once. (15 points) BOOK REVIEWS ON EARTH EXPLORATION A. Complete Guide to Life in a Cold Climate by Richard Sale This book is packed with information and deserves to be the ultimate Arctic wildlife guide for a long time to come. It begins well, with an introduction to Arctic geology, climate and Page 10 of 17


habitats, an overview of all the people living and working in the region. The bulk of the book is an extensive field guide to Arctic birds and mammals, with distribution maps and information on confusing species. Its scope is broad and generous, but I have a few niggles. It should really include Arctic plants, fish and invertebrates. This would have doubled the size of the book and made it unwieldy and impractical, but it's fair to say that the title is misleading. But I'm being picky here, and these minor shortcomings don't detract from the overall value of the book. B. Burton Holmes Travelogues Burton Holmes was the greatest traveler not just of his own time but perhaps of all time. A pretty big claim, but there's evidence to back it up. Over a 60-year period, Holmes visited nearly every country on the planet, photographed all he saw, and invented the term 'travelogue'. His pictures are stunning, both as social history and as art. Holmes photographed everything: the dead on battles; the running of bulls in Spain; a mule train in Death Valley. A sequence of Vesuvius erupting in 1906 includes a shot of a woman under an ash-strewn sky that is positively apocalyptic, but Holmes' work wasn't restricted to the large canvas - he was as capable of capturing an intimate portrait of a chicken vendor in a Bangkok market as he was revealing the vastness and intricacy of the construction of the Panama Canal. C. No More Beyond by Simon Nasht In Simon Nasht's brilliant biography of Sir Hubert Wilkins, he says that his subject isn't like other great explorers, primarily because most of us have never heard of him. He had no lust for fame, instead being driven by a thirst that led him to remote environments and places that cried out for exploration, rather than awards the popular challenges so desired by newspaper editors of the day. Nasht couldn't believe "a man could achieve so much and yet be so little remembered." In 1917 Wilkins was under the command of veteran polar explorer and photographer Frank Hurley in the Australian Flying Corps. Their mutual interests were vital to the development of aerial photography as an integral part of modern geography. D. Farmland Wildlife by James McCallum As a refuge for wildlife, British farmland has had a bad press in recent years. Fortunately, the artist's beautiful visual journey through the seasons presented in this book reveals that there is still an abundance of wildlife if you know where to look for it and what to look for. McCallum shuns detailed portraiture in favour of sketches capturing the spirit of his subjects-and hooray for that. If I need precise anatomical detail, I can look at a photograph. But if I want to grasp how a stoat rolls an egg, how a male whitethroat makes his fluttering display-flights or how long-tailed tits work together to build their nests, then I need something more-and McCallum is stunningly good at translating these complex movements and behaviours onto the page. His simple explanatory captions-taken from his field notebook-are a bonus. E. Troubled Waters by Sarah Lazarus Sometimes it seems as though the size of books on whales is led by the size of the subject matter. This, however, is a small, readable book. There are no detailed species accounts and the text is almost entirely devoted to the threats that whales and dolphins face, such as chemical and noise pollution, ship strikes and entanglement in fishing nets. A careful read reveals factual errors but, on the whole, these do not affect the thoughtful and concise

Page 11 of 17


discussion. It is notoriously difficult to get to the bottom of the whaling issue, and here Lazarus struggles a bit. The International Whaling Commission comes in for a lot of criticism, which would perhaps have been better directed at the three of its members who have chosen not to abide by the spirit of its conservation decisions. F. The High Lowlands by Derek Ratcliffes For some, the south of Scotland is the plainer and less charismatic sibling of the breathtaking Highlands and the rugged West Coast. But it's every bit as wild as those famed areas, but with a gentler appeal. This book describes an unexpected Eden, a place whose heart pulses to a different beat. This is an epic piece of writing, its subject matter covered in a manner more akin to the journals of a Victorian chronicler than a modern natural history book. Derek Ratcliffe's recordings of the natural goings-on in this lonely land spanned 50 years. His intimacy is apparent on every page. Everything is catalogued and described in meticulous detail, and few questions are left unanswered. It's a great pity that Derek did not live to see his life's work in print. This is a book for everyone, but it's a huge volume that you couldn't take with you on holiday unless you've got a pretty hefty rucksack and a strong back. Your answers Someone who left almost no stone unturned around the world. The surprising anonymity of someone. Disappointment that flora and other fauna are not mentioned. A book that is physically difficult to carry around with you. Some details are inaccurate in this book. Information written like an old-fashioned diary. Text that adds something to the images. A collaboration that produced great results. A book that covers all of nature's seasons.

1. .............. 2. .............. 3. .............. 4. .............. 5. .............. 6. .............. 7. .............. 8. .............. 9. ..............

A wide variety of subject matter.

10. ..............

D. WRITING (60 points) Part 1. Read the following extract and use your own words to summarize it. Your summary should be about 140 words long. (15 points) The temperature of the Sun is over 5,000 degrees Fahrenheit at the surface, but it rises to perhaps more than 16 million degrees at the center. The Sun is so much hotter than the Earth that matter can exist only as a gas, except at the core. In the core of the Sun, the pressures are so great against the gases that, despite the high temperature, there may be a small solid core. However, no one really knows, since the center of the Sun can never be directly observed. Solar astronomers do know that the Sun is divided into five layers or zones. Starting at the outside and going down into the Sun, the zones are the corona, chromosphere, photosphere, convection zone, and finally the core. The first three zones are regarded as the Sun's atmosphere. But since the Sun has no solid surface, it is hard to tell where the atmosphere ends and the main body of the Sun begins. The Sun's outermost layer begins about 10,000 miles above the visible surface and can be seen during an eclipse such as the one in February 1979. At any goes outward for millions of miles. This is the only part of the

Page 12 of 17


Sun that other time, the corona can be seen only when special instruments are used on cameras and telescopes to shut out the glare of the Sun's rays. The corona is a brilliant, pearly white, filmy light about as bright as the full Moon. Its beautiful rays are a sensational sight during an eclipse. The corona's rays flash out in a brilliant fan that has wispy spike-like rays near the Sun’s north and south poles. The corona is thickest at the sun's equator. The corona rays are made up of gases streaming outward at tremendous speeds and reaching a temperature of more than 2 million degrees Fahrenheit. The rays of gas thin out as they reach the space around the planets. By the time the Sun's corona rays reach the Earth, they are weak and invisible. …………………………………………………………………………………………………………………………………………………………… ……………………………………………………………………………………………………………………………………………………………… ……………………………………………………………………………………………………………………………………………………………… ……………………………………………………………………………………………………………………………………………………………… ……………………………………………………………………………………………………………………………………………………………… ……………………………………………………………………………………………………………………………………………………………… ……………………………………………………………………………………………………………………………………………………………… ……………………………………………………………………………………………………………………………………………………………… ……………………………………………………………………………………………………………………………………………………………… ……………………………………………………………………………………………………………………………………………………………… ……………………………………………………………………………………………………………………………………………………………… ……………………………………………………………………………………………………………………………………………………………… ……………………………………………………………………………………………………………………………………………………………… ……………………………………………………………………………………………………………………………………………………………… ……………………………………………………………………………………………………………………………………………………………… ……………………………………………………………………………………………………………………………………………………………… ……………………………………………………………………………………………………………………………………………………………… ……………………………………………………………………………………………………………………………………………………………… ……………………………………………………………………………………………………………………………………………………………… ……………………………………………………………………………………………………………………………………………………………… ……………………………………………………………………………………………………………………………………………………………… ……………………………………………………………………………………………………………………………………………………………… ……………………………………………………………………………………………………………………………………………………………… ……………………………………………………………………………………………………………………………………………………………… ……………………………………………………………………………………………………………………………………………………………… ……………………………………………………………………………………………………………………………………………………………… ……………………………………………………………………………………………………………………………………………………………… ………………………………………………………………………………………………………………………………………………………………

Page 13 of 17


……………………………………………………………………………………………………………………………………………………………… ……………………………………………………………………………………………………………………………………………………………… ……………………………………………………………………………………………………………………………………………………………… ……………………………………………………………………………………………………………………………………………………………… ……………………………………………………………………………………………………………………………………………………………… ……………………………………………………………………………………………………………………………………………………………… ……………………………………………………………………………………………………………………………………………………………… ……………………………………………………………………………………………………………………………………………………………… ……………………………………………………………………………………………………………………………………………………………… ……………………………………………………………………………………………………………………………………………………………… ………………………………………………………………………………………………………………………………………………………………

Part 2: Chart description (15 points) The table below shows the weight of people in a particular country from 1999 to 2009. Summarize the information by selecting and reporting the main features, and make comparisons where relevant. Write at least 150 words.

…………………………………………………………………………………………………………………………………………………………… ……………………………………………………………………………………………………………………………………………………………… ……………………………………………………………………………………………………………………………………………………………… ……………………………………………………………………………………………………………………………………………………………… ……………………………………………………………………………………………………………………………………………………………… ……………………………………………………………………………………………………………………………………………………………… ……………………………………………………………………………………………………………………………………………………………… ……………………………………………………………………………………………………………………………………………………………… ……………………………………………………………………………………………………………………………………………………………… ……………………………………………………………………………………………………………………………………………………………… ……………………………………………………………………………………………………………………………………………………………… ……………………………………………………………………………………………………………………………………………………………… ……………………………………………………………………………………………………………………………………………………………… ……………………………………………………………………………………………………………………………………………………………… ………………………………………………………………………………………………………………………………………………………………

Page 14 of 17


……………………………………………………………………………………………………………………………………………………………… ……………………………………………………………………………………………………………………………………………………………… ……………………………………………………………………………………………………………………………………………………………… ……………………………………………………………………………………………………………………………………………………………… ……………………………………………………………………………………………………………………………………………………………… ……………………………………………………………………………………………………………………………………………………………… ……………………………………………………………………………………………………………………………………………………………… ……………………………………………………………………………………………………………………………………………………………… ……………………………………………………………………………………………………………………………………………………………… ……………………………………………………………………………………………………………………………………………………………… ………………………………………………………………………………………………………………………………………………………………

Part 3. Essay writing (30 points) Write an essay of about 300 to 350 words on the following topic: In recent years, more and more people are choosing to read e-books rather than paper books. Do the advantages of reading e-books outweigh its disadvantages? Use your own knowledge and experience to support your arguments with examples and relevant evidence. Give reasons for your viewpoint. …………………………………………………………………………………………………………………………………………………………… ……………………………………………………………………………………………………………………………………………………………… ……………………………………………………………………………………………………………………………………………………………… ……………………………………………………………………………………………………………………………………………………………… ……………………………………………………………………………………………………………………………………………………………… ……………………………………………………………………………………………………………………………………………………………… ……………………………………………………………………………………………………………………………………………………………… ……………………………………………………………………………………………………………………………………………………………… ……………………………………………………………………………………………………………………………………………………………… ……………………………………………………………………………………………………………………………………………………………… ……………………………………………………………………………………………………………………………………………………………… ……………………………………………………………………………………………………………………………………………………………… ……………………………………………………………………………………………………………………………………………………………… ……………………………………………………………………………………………………………………………………………………………… ……………………………………………………………………………………………………………………………………………………………… ……………………………………………………………………………………………………………………………………………………………… ……………………………………………………………………………………………………………………………………………………………… ………………………………………………………………………………………………………………………………………………………………

Page 15 of 17


……………………………………………………………………………………………………………………………………………………………… ……………………………………………………………………………………………………………………………………………………………… ……………………………………………………………………………………………………………………………………………………………… ……………………………………………………………………………………………………………………………………………………………… ……………………………………………………………………………………………………………………………………………………………… ……………………………………………………………………………………………………………………………………………………………… ……………………………………………………………………………………………………………………………………………………………… ……………………………………………………………………………………………………………………………………………………………… ……………………………………………………………………………………………………………………………………………………………… …………………………………………………………………………………………………………………………………………………………… ……………………………………………………………………………………………………………………………………………………………… ……………………………………………………………………………………………………………………………………………………………… ……………………………………………………………………………………………………………………………………………………………… ……………………………………………………………………………………………………………………………………………………………… ……………………………………………………………………………………………………………………………………………………………… ……………………………………………………………………………………………………………………………………………………………… ……………………………………………………………………………………………………………………………………………………………… ……………………………………………………………………………………………………………………………………………………………… ……………………………………………………………………………………………………………………………………………………………… ……………………………………………………………………………………………………………………………………………………………… ……………………………………………………………………………………………………………………………………………………………… ……………………………………………………………………………………………………………………………………………………………… ……………………………………………………………………………………………………………………………………………………………… ……………………………………………………………………………………………………………………………………………………………… ……………………………………………………………………………………………………………………………………………………………… ……………………………………………………………………………………………………………………………………………………………… ……………………………………………………………………………………………………………………………………………………………… ……………………………………………………………………………………………………………………………………………………………… ……………………………………………………………………………………………………………………………………………………………… ……………………………………………………………………………………………………………………………………………………………… ……………………………………………………………………………………………………………………………………………………………… ……………………………………………………………………………………………………………………………………………………………… ……………………………………………………………………………………………………………………………………………………………… ……………………………………………………………………………………………………………………………………………………………… ………………………………………………………………………………………………………………………………………………………………

Page 16 of 17


……………………………………………………………………………………………………………………………………………………………… ……………………………………………………………………………………………………………………………………………………………… ……………………………………………………………………………………………………………………………………………………………… ……………………………………………………………………………………………………………………………………………………………… ……………………………………………………………………………………………………………………………………………………………… ……………………………………………………………………………………………………………………………………………………………… ……………………………………………………………………………………………………………………………………………………………… …………………………………………………………………………………………………………………………………………………………… ……………………………………………………………………………………………………………………………………………………………… ……………………………………………………………………………………………………………………………………………………………… ……………………………………………………………………………………………………………………………………………………………… ……………………………………………………………………………………………………………………………………………………………… ……………………………………………………………………………………………………………………………………………………………… ……………………………………………………………………………………………………………………………………………………………… ……………………………………………………………………………………………………………………………………………………………… ……………………………………………………………………………………………………………………………………………………………… ……………………………………………………………………………………………………………………………………………………………… ……………………………………………………………………………………………………………………………………………………………… ……………………………………………………………………………………………………………………………………………………………… ……………………………………………………………………………………………………………………………………………………………… ……………………………………………………………………………………………………………………………………………………………… ……………………………………………………………………………………………………………………………………………………………… ……………………………………………………………………………………………………………………………………………………………… ……………………………………………………………………………………………………………………………………………………………… ……………………………………………………………………………………………………………………………………………………………… ……………………………………………………………………………………………………………………………………………………………… ……………………………………………………………………………………………………………………………………………………………… ……………………………………………………………………………………………………………………………………………………………… ……………………………………………………………………………………………………………………………………………………………… ……………………………………………………………………………………………………………………………………………………………… ………………………………………………………………………………………………………………………………………………………………

-------------- THE END -------------(Thí sinh không được sử dụng tài liệu. Cán bộ coi thi không giải thích gì thêm)

Page 17 of 17


KỲ THI HỌC SINH GIỎI CÁC TRƯỜNG THPT CHUYÊN KHU VỰC DUYÊN HẢI VÀ ĐỒNG BẰNG BẮC BỘ LẦN THỨ XII, NĂM 2019

ĐỀ CHÍNH THỨC

HƯỚNG DẪN CHẦM MÔN TIẾNG ANH 11

Hướng dẫn chấm gồm 04 trang

A. LISTENING (50pts.) Part 1. You will hear part of a discussion on a current affairs programme between Nick barners and Alison Tempra about the performance of the company Facebook since it floated on the stock exchange, hosted by Emily Dunne. Choose the answer (A, B, C or D) which fits best according to what you hear. (10 points – 2.0 pts for each correct answer) (Chuyên Tuyên Quang) 1. C 2.B 3.C 4.A 5.A Part 2: You will hear the minister talking about crime. Are these statements true or false? Write T (true) or F (false). (10 points – 2.0 pts for each correct answer) (Chuyên Lương Văn Tụy) 6. F 7. T 8. F 9. T 10. F Part 3. You will hear a student called Tina asking Professor Van Diezen for advice on choosing courses. Listen and answer the following questions, using NO MORE THAN FIVE WORDS for each answer. (10 points – 2.0 pts for each correct answer) (Chuyên Lê Quý Đôn – Bình Định) 11. Compulsory and regular. 12. Full-time and flexible-time students. 13. People from off-campus. 14. No experimental facilities now. 15. Environmental Science. Part 4. Listen to the news and fill in the following blank with a suitable phrase. Write NO MORE THAN THREE WORDS in each blank. (20 points – 2.0 pts for each correct answer) (Chuyên Đại Học Sư Phạm Hà Nội) 16. sweeping victory 17. conservative majority 18. binding arbitration 19. labor-management disputes 20. verdicts 21. plaintiff 22. collective bargaining 23. rare oral dissent 24. race, gender and religion 25. non-disclosure provisions Page 1 of 4


B. LEXICO AND GRAMMAR (30 points) Part 1: Choose the answer A, B, C, or D that best completes each of the following sentences. Write your answers in the corresponding numbered boxes. (10 points – 01 point for each correct answer) 1. D Chuyên Thái Bình 2. B Chuyên Bắc Ninh 3. D Chuyên Hưng Yên 4. A Nguyễn Bỉnh Khiêm- Quảng Nam 5. C THPT Hòn Gai 6. D Chuyên Thái Nguyên 7. A Nguyễn Tất Thành- Yên Bái 8. D Lê Khiết- Quảng Ngãi 9. C Chuyên Bắc Giang 10. C Lê Quý Đôn- Bình Định Part 2: The passage below contains 5 mistakes. Identify the mistakes and write the corrections in the corresponding numbered boxes. There is an example at the beginning. (5.0 points – 01 point for each correct answer) (Chuyên Bình Phước) Your Answers Number Line Mistake Correction 1 3 rises arises 2 4 it itself 3 6 muscle muscles 4 10 distinctively distinctly 5 13 intensive intense Part 3. Complete each of the following sentences with a suitable preposition or particle. Write your answers in the space provided. (5.0 points – 01 point for each correct answer) 1. off Chuyên Hùng Vương- Phú Thọ 2. in Chuyên LVT- Ninh Bình 3. down Chuyên Lam Sơn- Thanh Hóa 4. up Chuyên Tuyên Quang 5. on Chuyên Lê Quý Đôn- Quảng Trị Part 4: Give the correct form of each bracketed word in the following passage. Write your answers in the space provided. (10 points - 01 point for each correct answer) (Chuyên Lào Cai) 1. fallout 6. high-ranking 2. class-action 7. credentials 3. safeguard 8. recruits 4. committing 9. lackluster/lacklustre 5. winery 10. epicenter Page 2 of 4


C. READING (60 points): Part 1: Read the following passage and decide which answer (A, B, C or D) best fits each gap. Write your answers in the corresponding numbered boxes. (10 points - 01 point for each correct answer) (Chuyên Lê Quý Đôn- Đà Nẵng) 1. A 2. B 3. C 4. B 5. C 6. A 7. D 8. D 9. C 10. A Part 2: Read the text below and think of the word which best fits each space. Use only one word in each space. Write your answers in the space provided. (15 points – 1,5 points for each correct answer) ( THPT Chu Văn An – Hà Nội)

1. only 6. side

2. With 7. on

3. window 8. round

4. coupled 9. As

5. against 10. purpose

Part 3: Read the following passage and choose the best answer. Write your answers in the corresponding numbered boxes. (10 points - 01 point for each correct answer) (Chuyên Nguyễn Trãi- Hải Dương) 1. B

2. C

3. D

4. B

5. A

6. D

7. C

8. C

9. B

10. A

Part 4: Read the passage and do the tasks that follow. Write your answers in corresponding numbered boxes. (10 points - 01 point for each correct answer) (Chuyên Lê Thánh Tông- Quảng Nam)

1.viii 6.i

2.x 7. Not Given

3.ii 8. No

4.v 9.Yes

5.ix 10. Not Given

Part 5: Read the text, identify which section A–F each of the following is mentioned. Write ONE letter A–F in the corresponding numbered space provided. Each letter may be used more than once. (15 points – 1,5 points for each correct answer) (Chuyên Biên Hòa- Hà Nam) 1. B 2.C 3.A 4.F 5.E 7.D 8.C 9.D 10.B 6. F D. WRITING (60 points) Part 1: Read the following extract and use your own words to summarize it. Your summary should be between 100 and 120 words long. (15 points) (Chuyên Trần Phú- Hải Phòng) Key Instructional Elements:  Has a main idea/concept: 5 points  Includes important facts and details: 3 points  Is in the writer’s own words: 4 points  Reflects underlying meaning: 1 point  Includes details in logical order: 2 points

Page 3 of 4


Part 2: Chart description (15 points) (PT Chuyên Ngoại Ngữ- Hà Nội) 1. Completion: 2 pts 2. Content: 5 pts - Cover the main information in the chart yet not go into too many details. - Make general remarks and effective comparisons. 3. Organisation: 2 pts - The ideas are well organized - The description is sensibly divided into paragraphs 4. Language: 5 pts - Use a wide range of vocabulary and structure - Good grammar 5. Punctuation and spelling: 1 pt Part 3. Essay writing (30 points) (Chuyên Vĩnh Phúc) The mark given to part 3 is based on the following scheme: 1. Content: 9 points a. Providing all main ideas and details as required b. Communicating intentions sufficiently and effectively 2. Organization & Presentation: 9 points a. Ideas are well organized and presented with coherence, cohesion, and clarity b. The essay is well-structured 3. Language: 9 points a. Demonstration of a variety of vocabulary and structures appropriate to the level of English language gifted upper-secondary school students b. Good use and control of grammatical structures 4. Handwriting, punctuation, and spelling: 3 points a. Intelligible handwriting b. Good punctuation and no spelling mistakes

Tổng điểm: 200 điểm Sau khi chấm, giám khảo qui về thang điểm 20 - Không làm tròn điểm

Page 4 of 4


SỞ GD&ĐT BẮC NINH

ĐỀ THI CHỌN HỌC SINH GIỎI VÙNG

TRƯỜNG THPT CHUYÊN

DUYÊN HẢI - ĐỒNG BẰNG BẮC BỘ

BẮC NINH

NĂM HỌC 2018 - 2019 Môn: Tiếng Anh – Lớp 11

(Đề thi đề xuất)

(Thời gian: 180 phút – không kể thời gian giao đề)

SECTION A. LISTENING (50 points) Part 1. For questions 1-5, you will hear part of a interview with a British politician. Choose the answer (A, B, C or D) which fits best according to what you hear. Write your answers in the corresponding boxes provided. (10 points) 1. Susan says that she particularly dislikes politicians who A. pretend to feel strongly about issues. B. disguise their real beliefs. C. are indecisive about issues D. openly treat voters with contempt. 2. When she had her disagreement with Martin Jones, Susan A. decided that personal ambition was not her main motivation. B. began to feel that she had failed as a politician. C. felt that her point of view was not correctly understood. D. regretted the effect it would have on her future in politics. 3. What was Susan’s attitude to involving colleagues in the controversy? A. She realized that they were unlikely to share her point of view. B. She was reluctant to do so because she was not sure she was right. C. She thought that involving colleges would make things worse. D. She felt they should decide for themselves whether she had a point. 4. When asked whether her opinion of her colleagues has changed, Susan says that A. their reaction has made her reluctant to get into the same position again. B. she prefers those who criticized her to those who kept their opinions private. C. there may come a time when she does not publicly support them on issues. Page 1


D. politicians place too much emphasis on their personal opinions of each other. 5. Susan thinks she was considered mad by some other politicians because A. Her behavior was out of character. B. They found her intimidating. C. She did not conform. D. Her unselfishness shamed them. Your answers: 1.

2.

3.

4.

5.

Part 2. For question 6-10, listen to the recording and decide whether the following statements are true (T) or false (F). Write your answers in the corresponding boxes provided. (10 points) 6. The man didn’t read everything on nutritional food labels 7. Alice says that before doing this project, she was unaware of what certain foods contained. 8. Jack agrees with Alice that the nutritional label is misleading. 9. Jack prefers the daily value system to other labelling systems because it is more logical. 10. Both students were surprised at the incorrect percentage of artificial additives given. Your answers: 6.

7.

8.

9.

10.

Part 3. For question 11-15, you will hear part of a lecture on stone tools and pottery making in Ireland in the Neolithic period. Answer the following questions with NO MORE THAN FOUR WORDS. Write your answers in the space provided. (10 points) 11. By which TWO possible ways were ploughs pulled? …………………………………………..…… , ……………………………………… 12. What purposes were water and sand used for in the final stages of axe-making? …………………………………………..……………………………………………… Page 2


13. Which part of the pots was often rubbed to make them watertight? …………………………………………..……………………………………………… 14. Where did the clay often come from? …………………………………………..……………………………………………… 15. Which thing did the look of decoration around the tops of the earliest pots bear resemblance to? …………………………………………..……………………………………………… Part 4. For questions 16 – 25, listen to a piece of news about trade conflict and complete the summary using NO MORE THAN THREE WORDS OR NUMBER for each gap. Write your answers in the corresponding boxes provided. (20 points) Trade war means higher price on a variety of products. For example, costs of imported production materials such as steel and aluminum will be raised, but actually this will probably take a toll on 16. ________. The resulting effects for involved countries are market 17. _______ and unemployment. Trade war can be started with the introduction of tariffs which aim to defense and 18. ________; however, their 19. ________ remains in question while 20. _______ may accompany. The levy on imported Canadian steel and aluminum by the US is considered 21. ________ and _________ as quoted with strong disapproval from the female speaker. Despite seemingly bright prospects in the G7 summit, Trump’s statement came as a 22. ________, which abandoned all the previous efforts to solve the trade conflict. Subsequently, Canada and EU countries have planned to take revenge on America with 23. ________. In fact, trade war can only cause 24. ________ and the issue should come up for further discussion until an agreement on 25. ________ has been reached. Your answers: 16.

21.

17.

22.

18.

23.

Page 3


19.

24.

20.

25.

SECTION B. LEXICO- GRAMMAR (30 points) Part 1. Choose the best option A, B, C, or D to complete the following sentences and write your answers in the corresponding numbered boxes. (10 points) 1. She had an unclear ________ of him in her mind, although she knew he would be old. A. sight

B. picture

C. vision

D. figure

2. Those without the major studios’ huge spending ________ are not convinced by the argument that marketing movies can double their budget. A. force

B. strength

C. weight

D. power

3. The outcome was a ________ circle whereby women's work, perceived as low status, was poorly rewarded and therefore regarded as unimportant. A. relentless

B. vicious

C. brutal

D. merciless

4. The best travel books of this year fall into three main categories: _________ informational, narrative, and anecdotal. A. truly

B. fully

C. literally

D. purely

5. People are dying for _________ of medical treatment. A. need

B. absence

C. want

D. shortage

6. With just a little preparation and regular feeding and watering, you can guarantee that you have a ________ of colour throughout the summer. A. disturbance

B. riot

C. demonstration

D. rally

7. Through a series of protestations and exclamations, coyness and giggling, I ________ that she was talking to her boyfriend. A. assembled

B. amassed

C. harvested

D. gathered

8. Sheila often borrowed wellingtons from her sisters, even though they were about three ________ too big. A. sizes

B. figures

C. pairs

9. Use CafĂŠlux Descaling Granules, available from your ________. Page 4

D. numbers


A. provider

B. seller

C. supplier

D. contractor

10. My profession allows me to both satisfy my own curiosity about the world and also to _______ some measure of international understanding. A. prolong

B. advertise

C. promote

D. elevate

Your answers: 1.

2.

3.

4.

5.

6.

7.

8.

9.

10.

Part 2. Read the passage below which contains 5 mistakes. Identify the mistakes and write the corrections in the corresponding numbered boxes. (5 points) Too many parents have a tough time getting clear and accessible information about the public schools at their communities. That is why President Clinton has announced an initiative requiring all states to produce annual report cards that are easily understood by and widely distributing to parents and the public, for each school, school district and the state as a whole. The report cards will include information to student achievement, teacher professional qualifications, class size, school safety and other factors that will help parents judge the overall performance of the schools. President Clinton’s proposal will help ensure which parents in every state have access to the information they need to determine the quality of their schools and identity areas in which improvement is needed. Your answers: No

Line

Mistake

Correction

1 2 3 4 5

Part 3. Complete each sentence with one suitable particle or preposition. Write your answers in the corresponding numbered boxes. (5 points) Page 5


1. He was punctual _______ the minute. 2. When they decided to get married, they went ______the wishes of their parents. 3. The teacher has obliged Robert to take ______ the offensive remarks he made to Julia. 4. I was completely bowled _______ by their warm reception. 5. Why don’t we have a night out? It would take your mind _______ your work. Your answers: 1.

2.

3.

4.

5.

Part 4. Write the correct form of the words given in the brackets. Write your answers in the corresponding numbered boxes. (10 points) Traffic congestion is now a problem in practically every major city in the world but nobody has yet found a solution to the seemingly inevitable chaos. A metro system is sadly impractical in most cities for geographical reasons. Tram systems are (1. WORK) _______ in old cities where narrow, winding streets make the installation of overhead cables a practical impossibility. Many local governments find the business of coaxing people into buses and (2. ENCOURAGE)________ them from using their cars easier said than done. And yet it is (3. CONCEIVE)______ that the situation should be allowed to remain as it is. The arguments in favor of direct action are now (4. REFUTE) ________ if we are ever to prevent (5. PRECEDE)_______ levels of pollution and economic chaos. It is astonishing how many people set off to climb Mount Olympus in completely unsuitable clothing. The weather conditions on the mountain are notoriously (6. PREDICT) _______ but people are fooled into thinking that just because the bottom is sunny, the summit will be similarly warm and bright. Nothing could be further from the truth. Bearing in mind that “(7. WARN)_______ is forearmed”, consult the local climbing club about likely conditions before setting off. Such local knowledge can be absolutely (8. VALUE) _______ and you would, to put it mildly, be extremely (9. ADVISE)_______ to ignore it. Whatever the likely weather, a good pair of boots is (10. DISPENSE) ________ as is some form of waterproof. And it is a steep climb so it goes without saying that a reasonable level of fitness is essential. Page 6


Your answers: 1.

2.

3.

4.

5.

6.

7.

8.

9.

10.

C. READING (60 points) Part 1. Read the following passage and decide which answer (A, B, C, or D) best fits each gap. Write your answers in corresponding numbered boxes. (10 points) Saving Latin Try telling the Reverend Reginald Foster that Lantin is a dead language. The response will be an 1. ________

rant from a teacher who has dedicated a large

2.______of his life to keeping the forerunner of the English and Romance languages alive. A man on a mission, he speaks only in Latin to his students, 3.________the language to life with his dramatic recitations. But Reverend Foster is not alone. Latin plays a special part in Italian cultural heritage, and politicians and academics have 4. ________concerns that enthusiasm for Latin in schools appears to be on the 5.________because of the popularity of English. Some purists even feel this is 6.______their national 7. ______. They have a point; in my experience Italians seem obsessed with using English words, and will 8._________an English word into a sentence even when a perfectly good native word will 9.__________. But need we really fear for Latin just yet? Maybe not. Even if it is on its last 10._______, it has survived for over 2,000 years. 1. A. impassioned

B. impassive

C. unenthusiastic

2. A. volume

B. chapter

C. act

D. scene

3. A. getting

B. putting

C. bringing

D. setting

4. A. conveyed

B. voiced

C. uttered

D. sounded

5. A. wane

B. fall

C. ebb

D. drop

6. A. deteriorating

B. eroding

C. disintegrating

D. eating

7. A. advancement

B. ontogenesis

C. retrogression

D. identity

Page 7

D. apathetic


8. A. slip

B. push

C. cast

D. post

9. A. answer

B. satisfy

C. suffice

D. content

10. A. laughs

B. leases

C. lengths

D. legs

Your answers: 1.

2.

3.

4.

5.

6.

7.

8.

9.

10.

Part 2. Read the following text and fill in the blank with ONE suitable word. Write your answers in corresponding numbered boxes. (15 points) Greatness It is rare indeed that we have the opportunity to behold (1) _______work the titanic forces that prime the Earth's massifs, those monumental ranges that are the (2) _______ of legend; that represent the pinnacle of human conquest; that tease mankind and dare it to attempt a summit climb, (3) _______the treachery and deadliness of the path that leads to dizzying success and immortality. Many have started the quest and failed, some (4) _______ the ultimate price, and it is not (5) _______ our mountaineers and explorers who seek to surmount the insurmountable; humanity as a (6) _______ has, forever it seems, had a morbid sort of fascination with nature's tallest, indelibly snowcapped peaks. (7) _______ is it only the prospect of their ascent that piques the interest; it is their very existence; we gaze up at them from the depths of normality - from the pitiful elevations of ground level - and all of us, in our own (8) _______, dream of becoming master of their heights and of viewing the world from atop their menacing crests. The photographer captures his dream in that perfect image, content to idealise the prospect of the ultimate challenge - it is for others to master, not him. The writer translates his dreams into prose, romanticising the quest, compelling other (9) _______ courageous souls to take those first brave steps into the unknown, whence they may never return, save in lore. The journalist reports their successes and failures with equal measures of gusto - for him, the story ends well (10) _______ way.

Page 8


Your answers: 1.

2.

3.

4.

5.

6.

7.

8.

9.

10.

Part 3. Read the following passage and choose the best answer to each of the following questions. Write your answers in corresponding numbered boxes. (10 points) COMMUNICATING WITH THE FUTURE In the 1980s the United States Department of Energy was looking for suitable sites to bury radioactive waste material generated by its nuclear energy programs. The government was considering burying the dangerous wastes in deep underground chambers in remote desert areas. The problem, however, was that nuclear waste remains highly radioactive for thousands of years. The commission entrusted with tackling the problem of waste disposal was aware that the dangers posed by radioactive emissions must be communicated to our descendants of at least 10,000 years hence. So the task became one of finding a way to tell future societies about the risk posed by these deadly deposits. Of course, human society in the distant future may be well aware of the hazards of radiation. Technological advances may one day provide the solutions to this dilemma. But the belief in constant technological advancement is based on our perceptions of advances made throughout history and prehistory. We cannot be sure that society won’t have slipped backward into an age of barbarism due to any of several catastrophic events, whether the result of nature such as the onset of a new ice age or perhaps mankind’s failure to solve the scourges of war and pollution. In the event of global catastrophe, it is quite possible that humans of the distant future will be on the far side of a broken link of communication and technological understanding. The problem then becomes how to inform our descendants that they must avoid areas of potential radioactive seepage given that they may not understand any currently existing language and may have no historical or cultural memory. So, any message indicated to future reception and decipherment must be as universally understandable as possible. Page 9


It was soon realized by the specialists assigned the task of devising the communication system that material in which the message was written might not physically endure the great lengths of time demanded. The second law of thermodynamics shows that all material disintegrates over time. Even computers that might carry the message cannot be expected to endure long enough. Besides, electricity supplies might not be available in 300 generations. Other media storage methods were considered and rejected for similar reasons. The task force under the linguist Thomas Sebeok finally agreed that no foolproof way would be found to send a message across so many generations and have it survive physically and be decipherable by a people with few cultural similarities to us. Given this restriction, Sebeok suggested the only possible solution was the formation of a committee of guardians of knowledge. Its task would be to dedicate itself to maintaining and passing the knowledge of the whereabouts and dangers of the nuclear waste deposits. This socalled atomic priesthood would be entrusted with keeping knowledge of this tradition alive through millennia and developing the tradition into a kind of mythical taboo forbidding people to tamper in a way with the nuclear waste sites. Only the initiated atomic priesthood of experts would have the scientific knowledge to fully understand the danger. Those outside the priesthood would be kept away by a combination of rituals and legends designed to warn off intruders. This proposal has been criticized because of the possibility of a break in continuity of the original message. Furthermore, there is no guarantee that any warning or sanction passed on for millennia would be obeyed, nor that it could survive with its original meaning intact. To counterbalance this possibility, Sebeok’s group proposed a “relay system� in which information is passed on over relatively short periods of time, just three generations ahead. The message then to be renewed and redesigned if necessary for the following three generations and so on over the required time span. In this way information could be relayed into the future and avoid the possibility of physical degradation. A second defect is more difficult to dismiss, however. This is the problem of social exclusiveness brought about through possession of vital knowledge. Critics point out that Page 10


the atomic priesthood could use its secret knowledge to control those who are scientifically ignorant. The establishment of such an association of insiders holding powerful knowledge not available except in mythic form to non-members would be a dangerous precedent for future social developments. 1. The word "chambers" in the passage is closest in meaning to________. A. partitions

B. openings

C. cavities

D. fissures

2. What problem faced the commission assigned to deal with the burial of nuclear waste? A. How to reduce the radioactive life of nuclear waste materials B. How to form a committee that could adequately express various nuclear risks C. How to notify future generations of the risks of nuclear contamination D. How to choose burial sites so as to minimize dangers to people 3. In paragraph 2, the author explains the possible circumstances of future societies________. A. to warn about the possible natural catastrophe B. to question the value of advances C. to highlight humankind's inability to resolve problems D. to demonstrate the reason nuclear hazards must be communicated 4. The word "scourges" in the passage is closest in meaning to ________. A. pressures

B. afflictions

C. worries

D. annoyances

5. In paragraph 4, the author mentions the second law of thermodynamics________. A. to support the view that nuclear waste will disperse with time B. to show that knowledge can be sustained over millennia C. to give the basic scientific reason behind the breakdown of material objects D. to contrast the potential life span of knowledge with that of material objects 6. The word "Its" in the passage refers to ________ A. knowledge

B. committee

C. solution

D. guardians

7. In paragraph 5, why is the proposed committee of guardians referred to as the "atomic priesthood"?

Page 11


A. Because they would be an exclusive group with knowledge about nuclear waste sites. B. Because they would use rituals and legends to maintain their exclusiveness C. Because they would be an exclusive religious order D. Because they would develop mythical taboos surrounding their traditions 8. According to the author, why did the task force under Sebeok propose a relay system for passing on information? A. To show that Sebeok 's ideas created more problems than they solved B. To support the belief that breaks in communication are inevitable over time C. To contrast Sebeok's ideas with those proposed by his main critics D. To compensate for the fact that meaning will not stable over long periods of time 9. According to paragraph 7, the second defect of the atomic priesthood proposal is that it could lead to________. A. the nonmembers turning knowledge into dangerous mythical forms B. the possible misuse of exclusive knowledge C. the establishment of a scientifically ignorant society D. the priesthood's criticism of points concerning vital knowledge 10. All of the following are mentioned in the passage as difficulties in devising a communication system with the future EXCEPT________. A. the failure to maintain communication link B. the loss of knowledge about today's civilization C. the inability of materials to endure over time D. the exclusiveness of priesthood Your answers: 1.

2.

3.

4.

5.

6.

7.

8.

9.

10.

Part 4. For questions 1 -10, read the following passage and do the tasks that follow. (10 points) Page 12


One Who Hopes A. Language lovers, just like music lovers, enjoy variety. For the latter there's Mozart, The Rolling Stones and Beyonce. For the former there's English, French, Swahili, Urdu ... the list is endless. But what about those poor overworked students who find learning difficult, confusing languages a drudge? Wouldn't it put a smile on their faces if there were just one simple, easy-to-Iearn tongue that would cut their study time by years? Well, of course, it exists. It's called Esperanto, and it's been around for more than 120 years. Esperanto is the most widely spoken artificially constructed international language. The name derives from Doktoro Esperanto, the pseudonym under which L. L. Zamenhof first published his Unua Libro in 1887. The phrase itself means 'one who hopes'. Zamenhof's goal was to create an easy and flexible language as a universal second language to promote peace and international understanding. B. Zamenhof, after ten years of developing his brain-child from the late 1870s to the early 1880s, had the first Esperanto grammar published in Warsaw in July 1887. The number of speakers grew rapidly over the next few decades, at first primarily in the Russian empire and Eastern Europe, then in Western Europe and the Americas, China, and Japan. In the early years, speakers of Esperanto kept in contact primarily through correspondence and periodicals, but since1905 world congresses have been held on five continents every year except during the two World Wars. Latest estimates for the numbers of Esperanto speakers are around 2 million. Put in percentage terms, that's about 0.03% of the world's population - no staggering figure, comparatively speaking. One reason is that Esperanto has no official status in any country, but it is an optional subject on the curriculum of several state education systems. It is widely estimated that it can be learned in anywhere between a quarter to a twentieth of the time required for other languages. C. As a constructed language, Esperanto is not genealogically related to any ethnic language. Whilst it is described as 'a language lexically predominantly Romanic', the phonology, grammar, vocabulary, and semantics are based on the western Indo-European languages. For those of us who are not naturally predisposed to tucking languages under our belts, it is an easy language to learn. It has 5 vowels and 23 consonants. It has one Page 13


simple way of conjugating all of its verbs. Words are often made from many other roots, making the number of words which one must memorise much smaller. The language is phonetic, and the rules of pronunciation are very simple, so that everyone knows how to pronounce a written word and vice-versa, and word order follows a standard, logical pattern. Through prefixing and suffixing, Esperanto makes it easy to identify words as nouns, verbs, adjectives, adverbs, direct objects and so on, by means of easy-to-spot endings. All this makes for easy language learning. What's more, several research studies demonstrate that studying Esperanto before another foreign language speeds up and improves the learning of the other language. This is presumably because learning subsequent foreign languages is easier than learning one's first, while the use of a grammatically simple and culturally flexible language like Esperanto softens the blow of learning one's first foreign language. In one study, a group of European high school students studied Esperanto for one year, then French for three years, and ended up with a significantly better command of French than a control group who had studied French for all four years. D. Needless to say, the language has its critics. Some point to the Eastern European features of the language as being harsh and difficult to pronounce, and argue that Esperanto has an artificial feel to it, without the flow of a natural tongue, and that by nature of its artificiality, it is impossible to become emotionally involved with the language. Others cite its lack of cultural history, indigenous literature - "no one has ever written a novel straight into Esperanto" - together with its minimal vocabulary and its inability to express all the necessary philosophical, emotional and psychological concepts. E. The champions of Esperanto - Esperantists - disagree. They claim that it is a language in which a great body of world literature has appeared in translation: in poetry, novels, literary journals, and, to rebut the accusation that it is not a 'real' language, point out that it is frequently used at international meetings which draw hundreds and thousands of participants. Moreover, on an international scale, it is most useful - and fair - for neutral communication. That means that communication through Esperanto does not give Page 14


advantages to the members of any particular people or culture, but provides an ethos of equality of rights, tolerance and true internationalism. F. Esperantists further claim that Esperanto has the potential - were it universally taught for a year or two throughout the world - to empower ordinary people to communicate effectively worldwide on a scale that far exceeds that which is attainable today by only the most linguistically brilliant among us. It offers the opportunity to improve communication inbusiness, diplomacy, scholarship and other fields so that those who speak many different native languages will be able to participate fluently in international conferences and chat comfortably with each other after the formal presentations are made. Nowadays that privilege is often restricted to native speakers of English and those who have special talents and opportunities for learning English as a foreign language.

Choose the correct heading for paragraphs A - G from the list of headings below. Write the correct number i - ix. List of Headings i. A non-exclusive language ii. Fewer languages, more results iii. Language is personal iv. What's fashionable in language v. From the written word to the spoken word vi. A real language vii. Harmony through language viii. The mechanics of a language ix. Lost in translation 1. Paragraph A

______

4. Paragraph D

______

2. Paragraph B

______

5. Paragraph E

______

3. Paragraph C

______

6. Paragraph F

______

Page 15


Do the following statements agree with the information given in the Reading Passage? Write_ YES

if the statement agrees with the information

NO

if the statement contradicts the information

NOT GIVEN

if there is no information on this

7. Supporters of Esperanto say it gives everyone an equal voice. 8. Esperanto is the only artificially-constructed language. 9. Esperanto can be learned as part of a self-study course. 10. Esperanto can be used equally in formal and casual situations. Your answers: 1.

2.

3.

4.

5.

6.

7.

8.

9.

10.

Part 5: Answer questions 1-10 by referring to the magazine article below, in which four men are interviewed about their jobs. (15 points) Take a step outside! Do you ever feel ‘stuck in the office’? We spoke to four men with outdoor jobs and asked them about the reality of working outside. A Name: John Hughes

Occupation: Academic Leader for Adventure Recreation

I’ve been working in the outdoor sports and activities industry since I was 22, but whereas I used to go climbing myself, I’ve got more of an academic position now I’m at the polytechnic. I work with students and focus my skills on demonstrating how mountaineering groups should be run and organised. The best aspect is the quality of the students but this is the course’s first year and we still have to see how it works out. But so far, so good. I’ve always got a lot out of my work but a day that jumps to mind was when I was working with the Wild South film crew in Antarctica. We had to climb down into the crater of Erebus and the volcano was semiactive that day, but it was the only day the cameras could go down. The volcano was throwing bits and pieces at us, so it was pretty Page 16


amazing being inside while it was partly erupting. That kind of thing appeals to me. I can’t think of anything that is particularly frustrating, other than that sometimes on a lovely day you have to work inside and on a horrible day you’re outside. Unfortunately, there isn’t currently a way around having enough flexibility in the programme to accommodate that. What does bother me are the endless meetings and things to do within the polytechnic system, which don't always seem relevant to how you might actually improve the programme for students. B Name: Mick Beasley

Occupation: Mountain Guide

For years I was going back and forth between the USA and New Zealand teaching skiing - 25 winters in a row. Eventually, I just wanted to settle down and find a base. That meant I needed a summer job so I started learning about summer mountain guiding. Now I have a great affection for this season and the advantage of working in a small company is that I’m hardly ever indoors. I prefer trips which aren’t technical, but difficult, and where without a guide it would be beyond most people's ability, and it’s rewarding to know they appreciate that fact. I only seal with groups of 4-5 people as it’s hard to find wilderness when you travel in large numbers. I’m at an age when I resent doing things that I don’t enjoy so I offer places to the clients that I want to go to anyway. Dealing with some of the older dilapidated shelters is not always pleasant, especially having to clean up after irresponsible previous occupants. Working with people is not difficult in the mountains; they are so far out of their element that they tend to have faith in you implicitly and are easy to get along with. To do a job like mine I think it’s essential that you attend the best courses, although it’s not a legal requirement. To go through this training and have other people look at your work and get their input is invaluable. C Name: Johnny Kitts

Occupation: Jet-boat driver

I’ve been driving commercially for 18 years on various rivers and every day is a highlight. Probably on a day-to-day basis, it’s just the varying nature of the area we operate in. What isn’t so pleasant is licking hikers up on dangerous parts of the river. We Page 17


do get a lot of hikers wanting to cross the river and try our best to get people in and out of those is no matter what the conditions are like. There are cut-off points that we believe are not safe to operate above, but mostly we try our hardest to get the trekkers out by boat. There’s probably not a dull part with any jet-boat job, especially on commercial trips. You’ve got to be aware of what is happening all the time, you can’t switch off about anything on the trip. That’s when the boat ends up stuck in the shallows. I am now more involved in scheduling other drivers, so I don’t spend as much time on the river as I did, which is a bit of a downpoint at times. For commercial driving it’s learning as you go - time on the river with checks done by the local harbor master. You get a good idea of a driver’s ability after about 25 hours. Some people have trouble reading shallow water and if you’re colour blind this is a major problem as you can’t pick out colouration in the water. It’s experience of the river and conditions that counts and that continues for as long as you jet-boat. D Name: Chris Macrae

Occupation: Snow photographer

All my training for this career has been completely hands-on. I’ve been skiing since I was a kid and was lucky enough to get sponsorship which allowed me to get up to serious competition level. But I’d also been into photography since I was twelve and my camera always went with me to the slopes. This year I was invited to take part in the biggest ski photography competition in the world. There were twelve of the world’s best skiers who teamed up with their selected photographer and we had to create the world’s most startling ski shots. It was overwhelming just to be asked along to the competition and then I was over the moon to actually win something! I’ve been skiing in Alaska for seven years now and every time it’s unique. Basically my whole life is just waiting for that period to come around. In the meantime, waiting on payment from people you work with is a frustration you have to put up with. It’s difficult because sometimes you might spend $400 or $500 on film and then there’s the travel costs, and you might not get paid for months while you wait for

Page 18


magazines to run the shots. And filing photos is not my strong point. It’s the most tedious and most important part of the job. For questions 1-10 answer by choosing from the list of men (A-D). Which person He appreciates the fact that his work has received professional recognition.

1….

His job requires a constant high level of concentration

2….

He has more of a managerial role than he used to.

3….

His current career is the result of a desire to work from a permanent

4….

location. He admits that he does not perform one of his duties particularly well. 5…. He would appreciate greater freedom in one aspect of his work.

6….

He appreciates the ability of the people he is responsible for.

7….

He appreciates the comments that other people have made about his

8….

skills. He likes the fact that he is exposed to constant change in his job.

9….

He resents the bureaucracy that is part of his job.

10….

D. WRITING (60 points) Part 1. Read the following extract and use your own words to summarize it. Your summary should be about 120-140 words long. (15 points) When we study the pronunciation of a language over any period of a few generations or more, we find there are always large-scale regularities in the changes. Such regular changes are often called sound laws. There are no universal sound laws (even though sound laws often reflect universal tendencies), but simply particular sound laws for one given language (or dialect) at one given period. It is also possible that fashion plays a part in the process of change. It certainly plays a part in the spread of change: one person imitates another, and people with the most prestige are most likely to be imitated, so that a change that takes place in one social Page 19


group may be imitated (more or less accurately) by speakers in another group. When a social group goes up or down in the world, its pronunciation of Russian, which had formerly been considered desirable, became on the contrary an undesirable kind of accent to have, so that people tried to disguise it. Some of the changes in accepted English pronunciation in the seventeenth and eighteenth centuries have been shown to consist in the replacement of one style of pronunciation by another style already existing, and it is likely that such substitutions were a result of the great social changes of the period: the increased power and wealth of the middle classes, and their steady infiltration upwards into the ranks of the landed gentry, probably carried elements of middle-class pronunciation into upper-class speech. A less specific variant of the argument is that the imitation of children is imperfect: they copy their parents’ speech, but never reproduce it exactly. This is true, but it is also true that such deviations from adult speech are usually corrected in later childhood. Perhaps it is more significant that even adults show a certain amount of random variation in their pronunciation of a given phoneme, even if the phonetic context is kept unchanged. One such force which is often invoked is the principle of ease, or minimization of effort. The change from fussy to fuzzy would be an example of assimilation, which is a very common kind of change. Assimilation is the changing of a sound under the influence of a neighbouring one. For example, the word scant was once skamt, but the /m/ has been changed to /n/ under the influence of the following /t/. Greater efficiency has hereby been achieved, because /n/ and /t/ are articulated in the same place (with the tip of the tongue against the teeth-ridge), whereas /m/ is articulated elsewhere (with the two lips). So the place of articulation of the nasal consonant has been changed to conform with that of the following plosive. …………………………………………………………………………………………… …………………………………………………………………………………………… …………………………………………………………………………………………… …………………………………………………………………………………………… Page 20


…………………………………………………………………………………………… …………………………………………………………………………………………… …………………………………………………………………………………………… …………………………………………………………………………………………… …………………………………………………………………………………………… …………………………………………………………………………………………… Part 2. Chart description (15 points) The charts below show the result of a survey of adult education. The first chart shows the reason why adults decide to study. The pie chart shows how people think the costs of adult education should be shared. Summarise the information by selecting and reporting the main features, and make comparisons where relevant.Your writing should be at least 150 words.

…………………………………………………………………………………………… Page 21


…………………………………………………………………………………………… …………………………………………………………………………………………… …………………………………………………………………………………………… …………………………………………………………………………………………… …………………………………………………………………………………………… …………………………………………………………………………………………… …………………………………………………………………………………………… …………………………………………………………………………………………… …………………………………………………………………………………………… …………………………………………………………………………………………… …………………………………………………………………………………………… …………………………………………………………………………………………… Part 3. Essay writing (30 points) Even though globalization affects the world's economies in a very positive way, its negative side should not be forgotten. Present argumentation to highlight your opinion on this matter. Give reasons and specific examples to support your opinion(s). Express your view in an essay of about 300-350 words. …………………………………………………………………………………………… …………………………………………………………………………………………… …………………………………………………………………………………………… …………………………………………………………………………………………… …………………………………………………………………………………………… …………………………………………………………………………………………… …………………………………………………………………………………………… …………………………………………………………………………………………… …………………………………………………………………………………………… …………………………………………………………………………………………… Page 22


…………………………………………………………………………………………… …………………………………………………………………………………………… …………………………………………………………………………………………… …………………………………………………………………………………………… …………………………………………………………………………………………… …………………………………………………………………………………………… …………………………………………………………………………………………… …………………………………………………………………………………………… …………………………………………………………………………………………… …………………………………………………………………………………………… -------------- The end-------------Người ra đề: 1. Nguyen Thuy Nhung (0869237558) 2. Nguyen Phuong Ly Ly (0985397666)

Page 23


SỞ GD&ĐT BẮC NINH

ĐỀ THI CHỌN HỌC SINH GIỎI VÙNG

TRƯỜNG THPT CHUYÊN

DUYÊN HẢI - ĐỒNG BẰNG BẮC BỘ

BẮC NINH

NĂM HỌC 2018 - 2019 Môn: Tiếng Anh – Lớp 11

(ĐÁP ÁN)

(Thời gian: 180 phút – không kể thời gian giao đề)

SECTION A. LISTENING (50 points) Part 1. For questions 1-5, you will hear part of a interview with a British politician. Choose the answer (A, B, C or D) which fits best according to what you hear. Write your answers in the corresponding boxes provided. (10 points) – Proficiency Practice Test, page 22 1. B

2. A

3. D

4. C

5. C

Part 2. For question 6-10, listen to the recording and decide whether the following statements are true (T) or false (F). Write your answers in the corresponding boxes provided. (10 points) (food – cam 13) 6. T

7. T

8. T

9. F

10. F

Part 3. For question 11-15, you will hear part of a lecture on stone tools and pottery making in Ireland in the Neolithic period. Answer the following questions with NO MORE THAN FOUR WORDS. Write your answers in the space provided. (10 points) 11. people, (and) cattle 12. grinding, (and) polishing 13. outside 14. local sources 15. (a series of) stitches Cam 6 – Test 3 – Sec 3

Page 1


Part 4. For questions 16 – 25, listen to a piece of news about trade conflict and complete the summary using NO MORE THAN THREE WORDS OR NUMBER for each gap. Write your answers in the corresponding boxes provided. (20 points) 16. consumers

21. illegal, unjustified

17. fluctuations

22. bombshell

18. boost local industries

23. retaliatory tariffs

19. effectiveness

24. casualties

20. political risks

25. mutual de-escalation

SECTION B. LEXICO- GRAMMAR (30 points) Part 1. Choose the best option A, B, C, or D to complete the following sentences and write your answers in the corresponding numbered boxes. (10 points) 1. B

2. D

3. B

4. D

5. C

6. B

7. D

8. A

9. C

10. C

Part 2. Read the passage below which contains 5 mistakes. Identify the mistakes and write the corrections in the corresponding numbered boxes. (5 points) No

Line

Mistake

Correction

1

2

at

in

2

4

distributing

distributed

3

5

(information) to

(information) on/about

4

8

which

that

5

9

identity

identify

Part 3. Complete each sentence with one suitable particle or preposition. Write your answers in the corresponding numbered boxes. (5 points) 1. to

2. against

3. back

Page 2

4. over

5. off


Part 4. Write the correct form of the words given in the brackets. Write your answers in the corresponding numbered boxes. (10 points) 1. unworkable

2. discouraging

3. inconceivable 4. irrefutable

5. unprecedented

6. unpredictable

7. forewarned

8. invaluable

10. indispensable

9. ill-advised

C. READING (60 points) Part 1. Read the following passage and decide which answer (A, B, C, or D) best fits each gap. Write your answers in corresponding numbered boxes. (10 points) 1A

2. B

3.C

4.B

5.A

6.B

7. D

8.A

9.C

10.D

Part 2. Read the following text and fill in the blank with ONE suitable word. Write your answers in corresponding numbered boxes. (15 points) 1.at

2.stuff

3.despite

4.paying

5.only/just

6.whole

7.nor

8.way

9.more

10.either

Part 3. Read the following passage and choose the best answer to each of the following questions. Write your answers in corresponding numbered boxes. (10 points) 1C

2C

3D

4B

5C

6B

7A

8D

9B

10D

Part 4. For questions 1 -10, read the following passage and do the tasks that follow. (10 points) 1. A - vii 7. YES

2. B - v

3. C - viii

4. D - iii

5. E- vi

6. F - i

9. NOT GIVEN 10 10. YES

8. NO

Part 5: Answer questions 1-10 by referring to the magazine article below, in which four men are interviewed about their jobs. (15 points) 1. D

2. C

3. C

4. B

5. D

6. A

7. A

8. B

9. C

10. A

Page 3


D. WRITING (60 points) Part 1. Writing summary (15 pts) Contents (10 pts) - The summary MUST cover the following points: + Sound laws have always changed regularly on a large scale and there are three reasons for these changes. + 1st reason: fashion - when one person imitates another pronunciation (the most prestige’s). + 2nd: the imitation of children from adults’ language sometimes are imperfect. + 3rd: for random variations in pronunciation, the principle of ease or minimization of effort - The summary MUST NOT contain personal opinions. Language use (5 pts) The summary: - should show attempts to convey the main ideas of the original text by means of paraphrasing (structural and lexical use) - should demonstrate correct use of grammatical structures, vocabulary, and mechanics (spelling, punctuations, ...) - should maintain coherence, cohesion, and unity throughout (by means of linkers and transitional devices) Part 2: Describing Chart (15 pts) Contents (10 pts) - The report MUST cover the following points: * Introduce the charts (2 pts) and state the overall trends and striking features (2 pts) * Describe main features with relevant data from the charts and make relevant comparisons (6 pts) - The report MUST NOT contain personal opinions Language use (5 pts) The report: Page 4


- should demonstrate a wide variety of lexical and grammatical structures, - should have correct use of words (verb tenses, word forms, voice...) and mechanics (spelling, punctuations...) Part 3: Writing essay (30 pts) The mark given to part 3 is based on the following criteria: 1. Organization (5 pts) a. Ideas are well organized and presented with coherence, cohesion and unity. b. The essay is well-structured: * Introduction is presented with clear thesis statement. * Body paragraph are written with unity, coherence and cohesion. Each body paragraph must have a topic sentence and supporting details and examples when necessary. * Conclusion summarizes the main points and offers personal opinions (prediction, recommendation, consideration ...) on the issue. 2. Content (15 pts) a. All requirements of the task are sufficiently addressed. b. Ideas are adequately supported and elaborated with relevant and reliable explanations, examples, evidence.... 3. Language use (10 pts) a. Demonstration of a variety of topic-related vocabulary. b. Excellent use and control of grammatical structures (verb tenses, word forms, voice...) and mechanics (spelling, punctuations...).

-------------- The end-------------Người ra đề: 1. Nguyen Thuy Nhung (0869237558) 2. Nguyen Phuong Ly Ly (0985397666)

Page 5


SỞ GD-ĐT BẮC GIANG TRƯỜNG THPT CHUYÊN BẮC GIANG

ĐỀ THI ĐỀ XUẤT DUYÊN HẢI Năm học 2018 - 2019 Môn thi : Tiếng Anh- Khối 11 Thời gian làm bài: 180 phút (Bài thi gồm 18 trang. Tổng điểm: 200)

Ghi chú: Thí sinh làm bài trực tiếp vào đề thi này. Không viết bằng bút chì, bút mực đỏ, bút xóa PART I. LISTENING (50 points) I. You will hear part of a radio discussion with Ellen Harrington of the Meadow Lane Residents Group, andTim Barlow from CartonTown Planning Department. For questions 1-5, choose the correct answer (A, B, C or D) which fits best according to what you hear. ( 10 points) 1. What was Ellen's first reaction when the town centre was closed to traffic? A. She was terrified.

B. She was miserable.

C. She was delighted.

D. She was suspicious.

2. The mood of the Meadow Lane residents can best be described as _______. A. resigned

B. dissatisfied

C. furious

D. dejected

3. How doesTim feel about the changes in the town centre? A. He regrets they were made so quickly. B. He believes they were inevitable. C. He thinks the town council should have foreseen the problem. D. He is proud the town council went forward with them. 4. What doesTim think about the protest Ellen's group is planning? A. He doesn't think it will accomplish anything. B. He thinks it is not aimed at the right people. C. He doesn't think drivers will be affected. D. He thinks it will be dangerous. 1


5. What does Ellen think will make the protest effective? A. the amount of publicity it will generate B. the inconvenience it will cause to drivers C. the number of demonstrators who will take part D. the forthcoming election Your answers: 1.

2.

3.

4.

5.

II. For questions 6-10, listen to a complaint from a woman called Julie Gold and decide whether these statements are True (T) or False (F). Write your answers in the corresponding numbered boxes provided. (10 points) 6. Her complaint today was about a car booking in Baker Road. 7. The car she wanted to use was not in the correct location. 8. The advisor assumes that the previous car user was not able to park in the correct place. 9. Julie was shocked to discover that the car had been heavily damaged. 10. The advisor says that the caller will not be charged for today’s booking Your answers: 6.

7.

8.

9.

10.

III. Question 11-15. Answer the question below. Write NO MORE THAN FIVE WORDS for each answer. (10 points) 11. Who works in the Beehive building? ________________________________________________________________________ 12. What nationality was the architect of the building? ________________________________________________________________________ 13. In which year did construction of the building start? 2


________________________________________________________________________ 14. How long did it take to contruct? ________________________________________________________________________ 15. How tall is the building in metres? ___________________________________________________________________ IV. Listen to the news and fill in the missing information. Write NO MORE THAN THREE WORDS taken from the recording for each answer in the space provided. These are projected to be the (1)………………………………in the world in 2050, 35 years from now, determined by numerous factors, including current growth rates migration, fertility population age and (2)…………………………… Vietnam is currently (3)……………………………..and will fall five spots. Uganda is this list’s (4)……………………………….jumping 21 places by nearly tripling its population. Japan’s (5)…………………………… and low birth rate will see it lose 6 spots and decrease in size by 18 million people. Russia is hemorrhaging due to its (6)…………………………… 13th is Tanzania, continuimg the dominant trend of huge Sub-Saharan African projected population gains by (7) ………………………..…… and increasing its population by 174% Congo will more than double in size, gaining seven spots on this list. Slow development and a lack of access (8)…………………………………..are the main reasons why populations in Africa are booming. The 11th- ranked country will be Mexico, the same rank it held 35 years earlier by gaining people at a (9)…………………………..……… As Brazil becomes an (10)………………………….…….., its boom will slow. PART II. LEXICO – GRAMMAR (30 points) I. Choose the correct word or phrase to complete each sentence. (10 points) 3


1. I knew my mother would ________ a face the minute she saw my new hair cut. A. drag

B. lift

C. pull

D. race

2. When she started borrowing my clothes without asking, I had to put my ______ down. A. stamp

B. show

C. fish

D. foot

3. If you are going to town, keep your eyes ________ for that book I was telling you about. A. peeled

B. clean

C. wide

D. fresh

4. He was a tall, intimidating persson, with a firm tone of voice and a very short ______ . A. fuse

B. line

C. patience

D. temperament

5. Peter was ______ of revealing the company’s future plans to the reporter. A. circumspect

B. wary

C. prudent

D. cautious

6. I'd just met his parents for the first time so I was on my best ______ . A. manners

B. conduct

C. behaviour

D. demeanour

7. As his whole family were doctors, it was in his ______ to take up that profession A. blood

B. spirit

C. soul

D. heart

8. She tried to ________ Tom’s importance to the company in order to gain a promotion for herself. A. diminish

B. swindle

C. reduce

D. shrink

9. I hate the way Tony ________ around looking so self – important. A. struts

B. scampers

C. slithers

D. slinks

10. Most frequently, the earthquake lasts 30 to 60 seconds, so usually there is no time to avert the mortal ________ once the shaking starts. A. upkeep

B. upturn

C. upshot

D. upswing

II. There is ONE mistake in each line in the following passage. Find it and correct it .Write your answer in the numbered box provided below. (5 points) Line

4


1

We all know stories about people with exceptional memories who have the possibility to remember hundreds of numbers after hearing them only once. Now experts are saying that such feats can be taught. For example, almost people can repeat about nine numbers if they are read out slowly. In one experiment, ten hours’ training produced only a small improvement, but the result after practicing for a thousand hours was amazing: some people remembered 80 or even 100 numbers. Similarly, a number of adults can identify only about five out a set of 21 colours that are only slightly different. But after 80 training hours one person could recognize 18 of theirs. Studies of 76 major composers show that it took at least ten years of musical training before any of theirs wrote a major work. Psychologists are therefore rising the possibility that genius is the production of teaching.

2 3 4 5 6 7 8 9 10 Line

Mistake

Correction

Line

1

4

2

5

Mistake

Correction

3 III. Complete each sentence with one suitable particle or preposition.(5 points) 1. She’s rung________. I must have said that something to upset her. 2. A pay rise is not________ the realms of possibility, I’m afraid. 3. The murderer did________ all of his victims by poisoning them with cyanide. 4. All our household goods are insured________ accidental damage. 5. You shouldn’t have sent Mark that Valentine’s card. I think you’ve scared him________. IV. Write the correct form of the words given in the brackets. (10 points)

5


Over half a century ago, scientists found they could record the (1) ELECTRIC ________ signals of the brain at work. What at first appeared a random hotchpotch of activity became a pattern of elegent waves (2) ________determined. Ever since, scientists have wondered whether the

RHYTHM

secrets of our thoughts, (3) ________and even consciousness itself PERCEIVE might be hidden in the patterns of our brain waves. The question of why we have brain waves is,(4) ________, as hotly ARGUE debated today as it was when the patterns were discovered. But the meaning, and even the existence, of fast rythyms in the alert brain is highly (5) ________.

CONTROVERSY

What is problematic is that you can’t perceive these rhythms directly, they are so well hidden in the noise created by other brain activity, but many (6) ________ now hold the (7) ________ that the SEARCH-CONVINCE significance of these brain waves should not be (8) ________.

ESTIMATE

The lastest suggestion is that the rhythms could be (9) ________ in detecting processes going on in different regions of the brain. Some

DECIDE

believe that these rhythms might even interact, and in doing so help the brain to package information into (10) ________ thoughts. How we COHERE bring together these related signals in the brain is a puzzle as yet unsolved. Your answer: 1

6

2

7

3

8

4

9

5

10 6


PART III. READING (60 points) I. For questions 1–10, read the following passage and decide which answer (A, B, C, or D) best fits each gap. Write your answers in corresponding numbered boxes. (10 points) As he hacked his way through the (1) _____ undergrowth, Richard Miles wondered why his boss couldn’t have sent him on a simple (2) _____ holiday. This African exploration was more than he had bargained for. As he (3) _____ deep ravines, treacherous river crossings, and dark forests full of (4) _____ vegetation, he asked himself why he hadn’t gone to a holiday (5) _____ where he could have lounged around by a pool all day. The chartered flight that would rescue them from the living nightmare was not due for three more days and he wasn’t even sure the pilot would find them so far off the beaten (6) _____ . There were only animal pathways here, a (7) _____ cry from his (8) _____ life in the City of London. And as his hopes for returning back to civilisation (9) _____ , his mobile phone rang. It was his boss checking to see how the (10) _____ was working out. 1. A. crowded

B. barren

C. dense

D. mountainous

2. A. scheduled

B. programmed

C. home

D. package

3. A. negotiated

B. marched

C. strode

D. pursued

4. A. humid

B. quaint

C. bustling

D. lush

5. A. resort

B. attraction

C. route

D. site

6. A. path

B. track

C. road

D. trail

7. A. sharp

B. far

C. long

D. hard

8. A. confidential

B. resilient

C. privileged

D. desolate

9. A. diminished

B. diverged

C. diluted

D. dissented

10. A. excursion

B. travel

C. outing

D. trip

Your answers: 1.

2.

3.

4.

5.

6.

7.

8.

9.

10. 7


II. For questions 1-10, read the text below and think of the word which best fits each space. Use only one word in each space. There is an example at the beginning (0). (15 points) To all intents and (1) ___________ , Jill had left her successful job as a lawyer to have a sabbatical for six months and everyone expected her to return to her highly-paid job. But in reality she was on her way to East Africa to work as a volunteer in an orphanage, intent (2) ___________ staying there for ever. (3) ___________ since she was six, she had been fascinated by stories about Africa and then horrified by the conditions (4) ___________ which millions of children live. Two years ago she had gone on an expensive safari holiday with her friends to Uganda but she didn’t enjoy the holiday. (5) ___________ she had seen children washing in the filthy puddles outside their shacks, the situation hadn’t seemed real to her. As a result, (6) ___________ returning to her hotel, she made a decision that would change her life for ever. No (7) ___________ had she arrived home than she sold her house and got a job as a volunteer (8) ___________ the intention of making a difference to children’s lives. She deliberately didn’t tell anyone what she was doing in (9) ___________ they tried to talk her out of it. (10) ___________ all the secrecy. Your answers: 1.

2.

3.

4.

5.

6.

7.

8.

9.

10.

III. Read the following passage and mark the letter A, B, C or D to indicate the correct answer to each of the questions.(10 points) When one hears the expression “role models”, one’s mind naturally jumps to celebrities, especially as far as young people are concerned. Therefore, it would be more than natural to assume that teenagers, heavily influenced by the media, are dazzled by wellknown Hollywood stars, famous musicians and internationally renowned athletes. However, nothing could be further from the truth. In reality, according to a recent survey, over 75% of teens who filled out an online questionnaire claimed that the role model for whom they had the greatest respect was not a famous personality, but a family member. It seems that the qualities that make a good role model are more complex than researchers first assumed. For example, Nancy L, a teenage girl from Wisconsin, described her role model as a woman who had a clear sense of what was important to her, making the effort to create things that would make a real difference in 8


the world. The woman she was referring to was her favorite aunt, who was a painter and sculptor. Role models come into young people’s lives in various ways. They are family members, educators, peers and ordinary people encountered in their daily lives. Students emphasized that being a role model is not confined to those with international fame or unbelievable wealth. Instead, they said the greatest attribute of a role model is the ability to inspire others. Teachers were often mentioned as examples in this case, ones that are dedicated to encouraging students, helping them push their limits and strengthen their characters. Another quality high on the list was the ability to overcome obstacles. In addition to parents, peers often made up a large percentage of such role models. Young people are at a point in their lives when they are developing the skills of initiative and capability, so it is only natural that they admire people who show them that success in the face of difficulty is possible. A final and perhaps unexpected character trait that the youth of today admire is a clear set of values. Children admire people whose actions are consistent with their beliefs; in other words, who practice what they preach. Role models help them to understand the significance of honesty, motivation and the desire to do general good. For example, local politicians who clearly struggle to improve living conditions in their cities are high on their lists of role models. Perhaps what should be understood from what young people consider important in a role model is that each and every person around them affects them to a certain extent, perhaps much more than most parents think. This makes it crucial for adults to be aware of their influence on the young and set the best examples possible. (Adapted from ‘Reader Digest’) Question 1: Which of the following is closest in meaning to “dazzled”? A. impressed B. disappointed C. confused D. frightened Question 2: Which of the following is LEAST likely to be assumed as teens’ role model? A. A handsome actor B. A talented footballer C. A hot popstar D. A brilliant scientist Question 3: What is surprising about the findings of the survey? A. Celebrities are the most common role models to most teens. B. The role models of the respondents are not quite influential. C. The qualities that make up teens’ role models are not simple. D. Most celebrities have their family members as role models. Question 4: What does the passage tell us about Nancy L’s role model? A. She was not related to her. B. She was famous for her talent. C. She was a mysterious person. D. She had strong priorities. Question 5: Which of the following is closest in meaning to “confined to”? 9


A. assisted by B. restricted to C. similar to D. influenced by Question 6: According to the passage, what quality makes teachers good role models? A. their ambition to succeed B. their wide knowledge C. their ability as academic educators D. their positive effect on students Question 7: The ability to overcome obstacles is important to young people because ______. A. teens must have it to teach their peers B. it is not something that one can easily find C. obstacles make life more difficult D. it is relevant to the stage of life they are in Question 8: According to paragraph 5, children really look up to those who _______. A. are as active as possible B. do what they say they will do C. pay attention to the needs of the young D. are religious in their life Question 9: According to the passage, some politicians are considered admirable ______. A. because they are familiar to young people B. because of the strong power they have C. because of their concern for others D. because they believe in themselves Question 10: The passage suggests that adults should ______. A. try to avoid imposing their influence on younger people B. realize that they have a strong effect on young people C. be careful of the role models their children may have D. encourage children to reject celebrities as role models Your answers: 1.

2.

3.

4.

5.

6.

7.

8.

9.

10.

IV. Read the passage and do the tasks. (10 points) Complete Questions 1–5. Two examples are given. Questions 1–5: The reading passage has seven paragraphs, A–G. Choose the correct heading for paragraphs A–F from the list of headings below.(5 points) List of Headings i. Using hunting to stop a worse crime ii. Legal hunting has little financial benefit 10


iii. Trying to make a living iv. Start by learning about the problem v. Different agricultural styles lead to different outcomes vi. Emotional reactions may have negative consequences vii. The system is not perfect but can be beneficial viii. Motivation to take care of animals ix. Travelling to Africa by plane Example :

Paragraph G:

Answer iv

Paragraph A

___VI_________

1. Paragraph B

______________

2. Paragraph C

______________

3. Paragraph D

______________

4. Paragraph E

______________

5. Paragraph F

______________

Read the passage again and answer Questions 6–10. Questions 6–10. Do the following statements agree with the information given in the reading passage? (5 points) Write TRUE if the statement agrees with the information FALSE if the statement contradicts the information NOT GIVEN if there is no information on this 6. Money from trophy hunting is sometimes used for good causes. 7. During the 1970s and 1980s animals weren’t protected in Namibia. 8. In Namibia animals are able to move around in the same way as they are in Botswana. 9. Local communities only receive a small amount of the money from trophy hunting. 10. The Zambian Wildlife Authority (ZAWA) still receives some money from legal hunting. 11


A. When a famous Zimbabwean lion was hunted and killed by a foreign tourist, people on social media were furious. This resulted in an airline ban of the transportation of trophies killed by tourists and people repeatedly asking travellers to avoid countries that allow this kind of trophy hunting. Trophy hunting describes legal hunting where people pay to do it. It is permitted in countries including Namibia, South Africa, Tanzania and Zambia. While many people are disgusted by this, what they don’t often realise is that stopping this kind of hunting might actually do more harm than good. B. Let’s look at Namibia for example. The local Minister of Environment and Tourism, Pohamba Shifeta, said that if airlines stopped transporting wildlife trophies, this would prevent the Namibians from protecting wildlife in their country. This is because the money that people pay to trophy hunt is used to stop illegal hunting, which is a much bigger problem than legal hunting. This suggests that trophy hunting can have a positive impact on the protection of wildlife, in theory at least. C. Namibia is often described as trophy hunting’s biggest success story. It is indeed true that hunting played an important role in increasing the number of wild animals after wars in the 1970s and 1980s negatively affected herd sizes. Today there are still 80 animal protection organisations in Namibia that rely completely on money from legal hunting. As Namibian journalist John Grobler says, farmers look after their animals better if they sell them to hunters. Namibia is currently experiencing a lack of rain which means some farmers may not have enough food for their animals. If they can’t earn money from their animals because hunting is stopped, farmers may decide to let them die. If hunting is stopped altogether, farmers will let the whole herd die. D. In Botswana, hunting large animals is now illegal for everyone and they have not suffered from the problems that John Grobler suggests above. However, there is a big difference between Botswana and Namibia – in Botswana there are no fences between people’s land, which means animals are able to move around freely. If farmers stop feeding them, they just go somewhere else to find food. In Namibia there are fences so the same thing will not happen there. Botswana’s ban on hunting is not without its problems, however. Large, wild animals are regularly killed when human life, food crops or farm animals are put in danger. In fact, this kind of animal death is considered to be a bigger killer than controlled hunting. E. Interestingly, in a recent article, Botswanan villagers said they would protect local wildlife better if they could earn money from it through hunting. However, this opinion goes 12


against the results of a large study carried out by Economists at Large. They concluded that in nine African countries that allow trophy hunting, the ‘sport’ accounted for just 1.8 percent of total tourism revenue, while, more importantly, only 3 percent of the money actually reached the communities where hunting occurs. F. So what does all of this tell us? It tells us that whatever we might think about the hunters, hunting can have a positive effect – both for wildlife and for African people – when and where it is properly and ethically managed. However, too often the opposite occurs and the industry suffers from bad management and bad ethics. It also tells us that trophy hunting is far more complex than both those who love it and those who hate it often realise. So while the hunting industry might need some serious changes, it’s perhaps not time to stop it completely when African wildlife organisations have no other way of making money. It is interesting to look at Zambia in this regard. Before hunting was stopped in 2013, 60 percent of the Zambian Wildlife Authority’s (ZAWA) revenue came from legal hunting. Today, ZAWA has very little money and has had to receive some from the Zambian government more than once. G. So what can we do? Apart from supporting Africa’s national parks and wildlife areas as photographic tourists, there are no easy answers or quick solutions. But if we first try to understand the issue, it is a step in the right direction. And while this situation might make us angry, remember that shouting at our computer doesn’t really help anyone. V. Read the passage and do the tasks. (15 points) Do animals think? When an animal knows it is being chased and starts to run, is it obeying some ancient instinct, or does it 'know' to be afraid? A Mammals have brains so they can feel pain and fear and can react in disgust. If a wildebeest did not feel pain, it would continue grazing as lions slowly devoured it. If an antelope did not sense fear, it would not break into a sprint at the first hint of cheetah. If a canine were not disgusted, it would not vomit; it would not be, as the saying goes, sick as a dog. Pain, fear and disgust are part of a mammal's survival machinery developed over tens of millions of years of evolution. Homo sapiens have, however, only been around for about 200,000 years so all three emotional states owe something to mammal origins. If football hooligans can feel those emotions, then so too do deer, foxes and dogs. The argument is 13


about how 'aware' or 'conscious' non-human mammals might be during these emotional events. When an animal knows it is being chased and starts to run, is it obeying some instinct inherited from ancestors that knew when to flee a danger zone or does it actually 'know' to be afraid? B That might be the wrong question. A human startled by a strange shape in a darkened corridor experiences a pounding heart, lungs gasping for air and a body in recoil. This is the well-known flight or fight reaction. A human appreciates the full force of fear and has already started to counter the danger a fraction of a second before the brain has time to absorb and order the information presented by the menacing figure. This is because mental calculations are too slow to cope with surprise attack. Pain precedes logic. Touch something hot and you withdraw your hand even before you have time to think about doing so. Once again, the wisdom is after the event. C If humans can experience the universal emotions of fear, anger, disgust, happiness, sadness, and surprise, then so can mammals. But does an animal think about its state of fear? Does it have not just a mind but a theory of mind? Does it have a sense of its own identity and that of another being? Can it put itself in another animal's shoes, so to speak? All animals communicate, but only humans have language. The puzzle remains: do animals think? Can they think about abstractions, about the past or about other animals? Researchers have wrestled with a series of experiments to see whether animals are capable of behaving as if they had the capacity to learn, the will to improvise and the ability to guess what other animals are thinking. Dogs show a remarkable capacity to guess human intentions correctly. Dogs, however, have lived intimately with humans for 15,000 years, so are unlikely to make ideal test subjects. D Primates, humanity's closest relatives, show unexpected abilities. Researchers from St Andrews in 1999 counted 39 different ways in which chimpanzees deal with food. Since these differ according to group and geography, they have used the word 'culture' to describe these differing methods. One female chimpanzee in Kyoto, convinced researchers that she could place Arabic numerals in ascending order one to nine. Monkeys astonished a team at 14


Columbia University in New York in 1998 by distinguishing groups of objects numbering one to four. Chimpanzees in large captive colonies forge alliances, switch sides and doublecross each other. They have also been seen in the wild systematically searching for leaves that have a medicinal effect. From such observations, a new branch of research has been born. It is called zoopharmacognosy. E Chimpanzees and humans share a common ancestor, and 98% of their DNA. Do more distant mammal relatives share the capacity for cogitation? Several years ago, Keith Kendrick at the Babraham Institute in Cambridge astonished the world by revealing that sheep could recognise up to 50 other sheep and up to ten human faces for at least two years after first seeing them. If a sheep can tell the difference between its flock members from flash cards and screen pictures, it must surely have a sense of these other creatures even when they are not there. Perhaps this means it also has an idea of 'self'. F More disconcertingly, pigs have demonstrated their own theory of mind. Mike Mendl of Bristol University revealed astonishing evidence at the British Association science festival in 2002. A larger and stronger pig that did not know where food was hidden had learned to follow a weaker, but better informed pig, to the trough. At this point the weaker pig would start to use distracting behaviour to keep the bully pig guessing, and only lunge for the rations when not being watched. It seems the smaller pig could guess what the other was thinking and outsmart it. In a human, this is what we call 'intelligence'. G. One of the animal world's highest achievers, however, is not a mammal at all. Betty the crow ¡lives in an Oxford laboratory. She repeatedly picks up a straight piece of wire, bends it into a hook and uses the hook to lift an appetising treat from a tube too deep for her beak. Before achieving this feat for the first time, she had never previously seen a piece of wire. So an animal far removed from humankind could identify a challenge, contemplate a simple matter of physics, identify a tool shape, select a raw material, make a tool and retrieve the reward. Birds are cousins not of mammals but of the dinosaurs. Humans and birds 15


last shared a common ancestor 200 million years ago. Experiments like these confirm, over and over again, that other mammals are more like us than we thought. It becomes increasingly difficult to know just what it is that makes humans different. (Source: exam essential ielts - practice test 1- passage 03-p43) Questions 1-5 Reading passage has six sections, A-G. Which section contains the fallowing information? Write the correct letter A-Fin boxes 1-5 on your answer sheet. NB You may use any letter more than once. EXAMPLE: an investigation into the extent of animal intelligence and awareness - C 1. the suggestion that an animal less recognised for its intelligence has an impressive memory 2. a comparison of what different living creatures experience emotionally 3. an account of a supposedly simple creature that has learnt a clever trick 4. acknowledgment that inherited abilities should not be seen as a measure of intelligence. 5. an account of how one animal got the better of another. Questions 6-10 Answer the questions below. Write NO MORE THAN THREE WORDS ONLY from the text for each answer. 6 According to the text, which animal is hunted and eaten by lions? 7 What sort of people are given as an example of low intelligence humans? 8 Which phrase in section B means run away or stay and confront the danger? 9 According to the text, which two animals successfully completed numerical tasks? 10 What type of tool did Betty the crow make from a piece of wire? PART D: WRITING (60 points)

16


I. Read the following passage and use your own words to summarize it. You MUST NOT copy or re-write the original. Your summary should be about 120 words long. (15 points) The food industry is one of the largest manufacturing industries in the world. As the demand for economical, safe and convenient food and beverage increases, it continues to expand. The food industry involves the four stages of agriculture, manufacturing, marketing and distribution. Agriculture refers to the production of agricultural goods through the growing of plants and the raising of domesticated animals. It encompasses a wide variety of specialties. In modern agriculture, plant breeding, pesticides, fertilizers and technological improvements have sharply increased yields from cultivation. Selective breeding and modern practices in animal husbandry such as intensive poultry farming have similarly increased the output of meat. The major agricultural products can be broadly grouped into food, fibers, fuel and raw materials. Specific foods include cereals, vegetables, fruits and meat. Fibers include cotton, wool, hemp, silk and flax. Raw materials include lumber and bamboo. The second stage, manufacturing, is the use of tools and labor to make things for use or sale. Modern manufacturing includes all intermediate food processes required for the production and integration of a product's components. Food processing is the set of methods and techniques used to transform raw ingredients into food or to transform food into other forms for consumption by humans or animals either in the home or by the food processing industry. Food processing typically takes clean, harvested crops or slaughtered and butchered animal products and uses these to produce attractive, marketable and often long-life food products. The third stage, marketing, is an ongoing process of planning and executing the marketing mix for products, services or ideas to create exchange between individuals and organizations. It tends to be seen as a creative industry, which includes advertising, 17


distribution and selling. It is also concerned with anticipating the customers' future needs and wants, which are often discovered through market research. Essentially, marketing is the process of creating or directing an organization to be successful in selling a product or service that people are willing to buy. Good marketing must be able to create a "proposition" or set of benefits for the end customer that delivers value through products or services. Lastly, food distribution, a method of distributing or transporting food from one place to another, is a very important factor in public nutrition. There are three main components of food distribution. They involve transport infrastructure such as roads, vehicles, rail transport, airports and ports, food handling technology and regulation, storage, warehousing and adequate source and supply logistics, based on demand and need. Through distribution, food gets to the big and small outlets such as supermarkets and shops for retail. .............................................................................................................................................................................. .............................................................................................................................................................................. .............................................................................................................................................................................. .............................................................................................................................................................................. .............................................................................................................................................................................. .............................................................................................................................................................................. .............................................................................................................................................................................. .............................................................................................................................................................................. .............................................................................................................................................................................. .............................................................................................................................................................................. ............................................................................................................................................................................. .............................................................................................................................................................................. .............................................................................................................................................................................. .............................................................................................................................................................................. .............................................................................................................................................................................. .............................................................................................................................................................................. .............................................................................................................................................................................. 18


.............................................................................................................................................................................. .............................................................................................................................................................................. .............................................................................................................................................................................. .............................................................................................................................................................................. .............................................................................................................................................................................

II. The graph below gives information about international tourist arrivals in different parts of the world. Summarise the information by selecting and reporting the main features, and make comparisons where relevant. Write at least 150 words (15 points)

…………………………………………………………………………………….......................……………. ………………………………………………………………………………………….......………………….. …………………………………………………………………………………….......................……………. ……………………………………………………………………………….........................………………… ……………………………………………………………..........................…………………………………… ……………………………………………………………………………........................…………………… …………………………………………………………………………….........................…………………… ………………………………………………………………………………......…………………………….. 19


…………………………………………………………………………….....……………………………….. …………………………………………………………………………........………………………………….. ……………………………………………………………...............………………………………………….. …………………………………………………………........………………………………………………….. ………………………………………………………….......………………………………………………….. …………………………………………………………........………………………………………………….. ……………………………………………………………………….......…………………………………….. ……………………………………………………………………........……………………………………….. ……………………………………………………………………………………………………………….. ……………………………………………………………………………………………………………….. ……………………………………………………………………………………………………………….. ……………………………………………………………………………………………………………….. ……………………………………………………………………………………………………………….. …………………………………………………………………………………………………………….. …………………………………………………………………………………………………………….. …………………………………………………………………………………………………………….. …………………………………………………………………………………………………………….. …………………………………………………………………………………………………………….. ……………………………………………………………………………………………………………..

III. Writing an essay (30 points) You should spend about 40 minutes on this task. Write about the following topic: Tourism is an important industry which has developed the economies of countries in many parts of the world. What effect has tourism had on local communities? Give reasons for your answer and include any relevant examples from your own knowledge or experience. Write at least 250 words. …………………………………………………………………………………………………………….. …………………………………………………………………………………………………………….. …………………………………………………………………………………………………………….. 20


…………………………………………………………………………………………………………….. …………………………………………………………………………………………………………….. …………………………………………………………………………………………………………….. …………………………………………………………………………………………………………….. …………………………………………………………………………………………………………….. …………………………………………………………………………………………………………….. …………………………………………………………………………………………………………….. …………………………………………………………………………………………………………….. …………………………………………………………………………………………………………….. …………………………………………………………………………………………………………….. …………………………………………………………………………………………………………….. …………………………………………………………………………………………………………….. …………………………………………………………………………………………………………….. …………………………………………………………………………………………………………….. …………………………………………………………………………………………………………….. …………………………………………………………………………………………………………….. …………………………………………………………………………………………………………….. …………………………………………………………………………………………………………….. …………………………………………………………………………………………………………….. …………………………………………………………………………………………………………….. …………………………………………………………………………………………………………….. …………………………………………………………………………………………………………….. …………………………………………………………………………………………………………….. …………………………………………………………………………………………………………….. …………………………………………………………………………………………………………….. …………………………………………………………………………………………………………….. …………………………………………………………………………………………………………….. …………………………………………………………………………………………………………….. …………………………………………………………………………………………………………….. …………………………………………………………………………………………………………….. …………………………………………………………………………………………………………….. …………………………………………………………………………………………………………….. …………………………………………………………………………………………………………….. …………………………………………………………………………………………………………….. …………………………………………………………………………………………………………….. …………………………………………………………………………………………………………….. …………………………………………………………………………………………………………….. …………………………………………………………………………………………………………….. …………………………………………………………………………………………………………….. …………………………………………………………………………………………………………….. 21


…………………………………………………………………………………………………………….. …………………………………………………………………………………………………………….. …………………………………………………………………………………………………………….. …………………………………………………………………………………………………………….. …………………………………………………………………………………………………………….. …………………………………………………………………………………………………………….. …………………………………………………………………………………………………………….. …………………………………………………………………………………………………………….. ……………………………………………………………………………………………………………..

......................THE END...................

22


TRƯỜNG THPT CHUYÊN BẮC GIANG

ĐÁP ÁN ĐỀ XUẤT ĐỀ DUYÊN HẢI Năm học 2018- 2019

Môn: Tiếng Anh (khối 11) (Thời gian làm bài: 180 phút) PART I. LISTENING: 50/200 I. You will hear part of a radio discussion with Ellen Harrington of the Meadow Lane Residents Group, andTim Barlow from CartonTown Planning Department. For questions 1-5, choose the correct answer (A, B, C or D) which fits best according to what you hear. (10 points) 1. C 2. C 3. D 4. B 5. A Source: exam essential CAE - test 3- part 3 II. For questions 6-10, listen to a complaint from a woman called Julie Gold and decide whether these statements are True (T) or False (F). Write your answers in the corresponding numbered boxes provided. (10 points) 6. false 7. true 8. true 9. false 10. false Source: Exam Essentials Ielts Practice Test 5 III. Question 11-15. Answer the question below. (10 points) 11. Prime Minister/cabinet 12. Scottish 13. 1969 14. 10 years/ one decade 15. 72/ seventy two Source: Exam essential IELTS - test 6- section 2-p162 IV. Listen to the news and fill in the missing information. Write NO MORE THAN THREE WORDS taken from the recording for each answer in the space provided. (20 points) 1. most populated nations 1


2. mortality rates 4. near capacity 4. biggest riser 5. aging population 6. declining economic future 7. leaping 15 spots – 8. to birth control 9. relatively average rate 10. economic powerhouse https://www.youtube.com/watch?v=YJjz7LVVl8c&t=20s PART II. LEXICO – GRAMMAR (40 points) I. Choose the correct word or phrase to complete each sentence. (10 points) 1. C

6. C

2. D

7. A

3. A

8. A

4. A

9. A

5. B

10.C

II. There is ONE mistake in each line in the following passage. Find it and correct it .Write your answer in the numbered box provided below. (5 points) Line

Mistake

Correction

1

possibility

ability

2

almost

most

3

out

out of

4

rising

raising 2


5

production

product

III. Complete each sentence with one suitable particle or preposition.(5 points) 1. D. off 2. C. within 3. A. away with 4. B. against 5. A. off IV. Write the correct form of the words given in the brackets. (10 pts) 1 eletrical 6 researchers 2

Rhythmically

7

convintion

3

perception(s)

8

underestimated

4

(un)arguably

9

decisive

5

controversial

10

coherent

PART III. READING (60 points) I. For questions 1–10, read the following passage and decide which answer (A, B, C, or D) best fits each gap. Write your answers in corresponding numbered boxes. (10 points) 1 C 2 D 3 A 4 D 5 A 6 B 7 B 8 C 9 A 10 D II. For questions 1-10, read the text below and think of the word which best fits each space. Use only one word in each space. There is an example at the beginning (0). (15 points) 1 purposes 2 on 3 Ever 4 in 5 Until 6 on 7 sooner 8 with 9 case 10 Hence III. Read the following passage and mark the letter A, B, C or D to indicate the correct answer to each of the questions. (10 points) 1. A

3. C

5. B

7. D

9. C

2. D

4. D

6. D

8. B

10. B

IV. Read the passage and do the tasks (10 points) 1i

2 viii

3v

4 ii

5 vii

6 True (money that people pay for trophy hunt is used to stop illegal hunting) 7 Not given 8. False 3


9 True (only 3 percent of the money actually reached the communities where hunting occurs) 10. False V. Read the passage and do the tasks (15 points) 1. E 2. A 3. G 4. C 5. F 6. a wildebeest

7. football hooligans

8. flight or fight

9. chimpanzees and monkeys 10. a hook

PART IV. WRITING 60 I. Read the following passage and use your own words to summarize it. You MUST NOT copy or re-write the original. Your summary should about 100 words long. (15 points) Requirements: Good summary with enough content and clear, logical information. II. (15 points) 1. Completion: 2 pts

- The writing is complete. - The writing is neither too long nor too short.

2. Content: 5 pts

- Cover the main information of the table yet not go into too much detail. - Make general remarks and effective comparisons.

3. Organization: 3 pts

- The ideas are well-organized. - The description is sensibly divided into paragraphs.

4. Language: 3 pts

- Use a wide range of vocabulary and structures. - Good use and control of grammatical structures.

5. Handwriting,

- Intelligible handwriting. 4


punctuation and spelling:

- Good punctuation and no spelling mistakes.

2 pts

III. (30 points) Model writing

WRITING AN ESSAY Notes: The mark given to part 3 is based on the following criteria: 1. Content: (35% of total mark) a. Providing all main ideas and details as required 5


b. Communicating intentions sufficiently and effectively 2. Organization & Presentation: (30% of total mark) a. Ideas are well organized and presented with coherence, cohesion, and clarity b. The essay is well-structured 3. Language: (30% of total mark) a. Demonstration of a variety of vocabulary and structures appropriate to the level of English language gifted upper-secondary school students b. Good use and control of grammatical structures 4. Punctuation, and spelling and handwriting (5% of total mark) a. Good punctuation and no spelling mistakes b. Legible handwriting

6


Transcripts Part 1

7


8


9


iii.

10


11


12


SỞ GIÁO DỤC & ĐÀO TẠO PHÚ THỌ TRƯỜNG THPT CHUYÊN HÙNG VƯƠNG

KÌ THI CHỌN HỌC SINH GIỎI KHU VỰC DUYÊN HẢI BẮC BỘ NĂM HỌC 2019- 2020

ĐỀ ĐỀ NGHỊ

MÔN THI: Tiếng Anh

LỚP: 11

Ngày thi: … tháng … năm 2019 (Thời gian làm bài 180 phút không kể thời gian giao đề)

PART ONE. LISTENING (50p.) Question 1. For questions 1-5, listen to a discussion on the subject of identity and the labels that identify groups of people in society and choose the correct answer A, B, C, or D which fits best according to what your hear. Write your answers in the corresponding numbered boxes provided. (10p.) 1. What does Marc suggest about being labelled as ‘a philosopher’? A. It’s the label he uses most often. B. It’s not a label he could use in other contexts. C. It’s the label that best describes what he does. D. It’s only one of various labels he sometimes uses. 2. Elena suggests that the term ‘pigeonholed’ is used by people who A. prefer not to use labels. B. dislike the labels they have chosen. C. resent having labels applied to them. D. feel that they don’t fit under any labels. 3. Marc and Elena agree that a very strong sense of identity can A. leave people unprepared for changing circumstances. B. have adverse effects on relationships at work. C. make people too quick to categorise others. D. lead to tensions in aspects of family life. 4. When asked about labelling by the media, Elena reveals A. a determination to make it more sensitive. B. a concern about the social consequences. C. a feeling that it isn’t greatly significant. D. an acceptance that it has a part to play.


5. In his concluding comments, Marc suggests that labels of identity A. allow the individual to fit into society. B. help the individual to prioritise things in life. C. prevent the individual from being truly unique. D. stop the individual from becoming too self-obsessed Your answers: 1.

2.

3.

4.

5.

Questions 2. For questions 1-5, listen to a complaint from a woman called Julie Gold and decide whether these statements are True (T) or False (F). Write your answers in the corresponding numbered boxes provided. (10p.) 1. Her complaint today was about a car booking in Baker Road. 2. The car she wanted to use was not in the correct location. 3. The advisor assumes that the previous car user was not able to park in the correct place. 4. Julie was shocked to discover that the car had been heavily damaged. 5. The advisor says that the caller will not be charged for today’s booking. Your answers: 1.

2.

3.

4.

5.

Question 3: You will hear an interview with a man called Mark Phillips, who is talking about his work as a potter. Answer the questions (1-5) with NO MORE THAN FIVE WORDS. (10p.) 1. Why did pottery not appeal to Mark when he was younger? _________________________________________________________ 2. What was his mother’s advice when he decided to take up pottery? _________________________________________________________ 3. How does Mark describe the pots he makes? _________________________________________________________ 4. What has surprised Mark about the pottery community? _________________________________________________________ 5. In the future, what would Mark like to be able to? _________________________________________________________


Questions 4. For questions 1-10, listen to a report on Finland, the global education leader and supply the blanks with the missing information. Write NO MORE THAN FOUR WORDS taken from the recording for each answer in the space provided. (20p.) USA is considered to be (1)___________ in the world when it comes to education. Finland, the small Nordic country best known for Nokia phones, (2)___________________________ and heavy metal music is actually a leader in world education. The Finns becomes the global education leader without turning school into (3)___________________________. There are five reasons that set Finland apart: - No child gets left behind literally. Finland provides all families particularly low income families with a(n) (4)___________________________. - They are just like more chill man. Finnish children start school late, get large amount of (5)___________________________, rarely do assignments and almost never take standardise tests. - Teachers are respected. It is not easy to become a teacher in this country because there are only 8 universities offering the programmes required to earn a(n) (6)___________________________. - They believe that less is more. They give priority to patience, (7)___________________________ and problem solving skills instead of cramming knowledge. - They have fewer (8)___________________________ like income inequality or crime. To sum up, Finland do things differently but have a(n) (9)___________________________ of achieving better results. USA should be more open to what (10)_____________ are doing around the globe. Your answers: 1.

2.

3.

4.

5.

6.

7.

8.

9.

10.

PART TWO. LEXICO – GRAMMAR (30p.) Question 1. Choose the word or phrase which best completes each sentence. Write your answers (A, B, C, or D) in the space provided under this part. (10p.) 1. The Higg Boson become hot ____ talk following the release of a series of Hollywood movies based loosely around the discovery of a so-called “Good Particle”. A. common B. topic C. table D. subject 2. A few of older campers were sent home after a week as they were ____ A. lenient B.unruly C. erratic D. indulgent


3. The barriers suddenly collapsed and the crowd of supporters ____ forward on to the pitch. A. swirled B. gushed C. trickle D. surged 4. Archaeology is one is one of the most interesting scientific ____ A. divisions B. disciplines C. matters D. compartments 5. The projects has progressed in ___ and starts due to a constant change in funding. A. wits B. bits C. fits D. sits 6. It’s as if that silly argument we had has driven a ___ between us and we’ve lost all our old closeness. A. ledge B. beam C. plank D. wedge 7. The country is an economic ___ with chronic unemployment and rampant crime. A. lost cause B. false dawn C. dark horse D. basket case 8. I ___ and drew my foot out quickly. The bath water was so hot. A. tingled B. ached C. writhed D. pained 9. We managed to ___ the last economical depression by cutting down our workforce. A. surmount B. override C. float D. weather 10. The renewed in Elizabethan times is evident in the ___ of new Hollywood film set during that period. A. transience B. spate C. hypocrisy D. demise Your answers 1

2

3

4

5

6

7

8

9

10

Question 2. There are FIVE mistakes in this paragraph. Write them down & give the correction. Write your answers in the space provided. (5p.) Line 1 2 3 4 5 6 7 8 9 10 11

The role of the presenter is currently acting as a deadly weight on the advance of nature television. What’s more, the argument that personable presenters help draw viewers into shows is now weighed by the fashion for making them the focus on the programmes. This isn’t confined to macho croc wrestlers and brainless celebrities looking for a ‘green star’ on their CVs. One of the most awesome pieces of film ever made of British wildlife – the dusk roosting flights of a million starlings over the Somerset Levels – was all though ruined by the director’s insistence in interrupting the geometry of the performance every five seconds with a cut-off of the presenter waving his arms as if he were conducting them, or as if we were incapable of knowing how to respond without a presenter’s cue.


Your answers Line

Mistake

Correction

1. 2. 3. 4. 5.

Question 3. Fill in each gap with ONE preposition to finish the following sentences. Write your answers in the space provided. (5p.) 1. They chalked the poor sales ______ the lower number of tourists visiting this year. 2. When the cost was set ______ the benefits, the scheme looked good. 3. The students wrestled ______ the difficult mathematics problem. 4. There’s nothing to swot ______ as it’s a general knowledge quiz. 5. Don’t try to palm me ______ with your lies and excuses. Your answers 1.

2.

3.

4.

5.

Question 4. Use the correct form of each bracketed word in the numbered space. Write your answers in the space provided. (10p.) The standard of television programming _______ (1. product) in this country is in terminal decline. The _______ (2. shed) has become a meaningless term confined in its applicability to _______ (3. go) days when adult content felt the full force of ________ (4. censor) and was not allowed to appear on the box until after 9:00 p.m. Nowadays, however, it seems anything goes any time. And, truth told, whatever anything is, it seldom 'goes' for much longer than a half hour or so at any rate before it is interrupted by a ______ (5. commerce) break. And don't even get me started on those appalling _______ (6. inform) most of the networks run right the way through the night, one after another, for up to thirty minutes at a time. lt is _______ (7.true) painful. Terrestrial television is now, as far as I am concerned, a laughing stock. All the quality has been bought up by the satellite networks, with their big-money weight behind them, but even here _______ (8. pick) are slim. In protest at the dire state of things, I have become a converted _______ (9. net). I look to the web now to find good content. There, I can find just enough re-run of quality programmes to


prevent myself from falling into utter despair and pining for the good old days of _______ (10. year). Your answers 1.

6.

2.

7.

3.

8.

4.

9.

5.

10.

PART THREE: READING (60p.) Question 1. Read the text below and decide which answer (A, B, C or D) best fits each gap. Write your answers in the space provided. (10p.) Greenhouse gases are being released into the atmosphere 30 times faster than the time when the Earth experiences a (1) ______ episode of global warming. A study comparing the rate at which carbon dioxide and methane are being (2) ______ now, compared to 55 million years ago when global warming also occurred, has found dramatic differences in the speed of release. James Zachos, professor of earth sciences at the University of California, Santa Cruz, said the speed of present buildup of greenhouse gases is far greater than during the global warming after the (3) ______ of the dinosaurs. “The emissions that caused this past episode of global warming probably lasted 10,000 years,” Professor Zachos told the American Association for the Advancement of Science at a meeting in St. Louis. “By burning fossil fuels, we are likely to emit the same amount over the next three centuries.” He warned that studies of global warming events in the geological past (4) ______ the Earth’s climate passes a (5) ______ beyond which climate change accelerates with the help of positive feedbacks - vicious circles of warming. Professor Zachos is a leading (6) ______ on the episode of global warming known as the palaeoence-eocene thermal maximum, when average global temperatures increased by up to 50C due to a massive release of carbon dioxide and methane. His research into the deep ocean (7) ______ suggests at this time that about 4.5 billion tons of carbon entered the atmosphere over 10,000 years. “This will be the same amount of carbon released into the atmosphere from cars and industrial emissions over the next 300 years if present (8) ______ continue”, he said. Although carbon can be released suddenly and naturally into the atmosphere from volcanic activity, it takes many thousands of years for it to be removed permanently by natural processes. The ocean is capable of removing carbon, and quickly, but this natural (9) ______ can be easily (10) ______, which is probably what happened 55 million years ago. “It will take tens of thousands of years before


atmospheric carbon dioxide comes down to preindustrial levels,” the professor said. “Even after humans stop burning fossil fuels, the effects will be long-lasting.” 1. A. prearranged 2. A. emitted 3. A. dementia 4. A. comment 5. A. barricade 6. A. autocrat 7. A. dusts 8. A. trends 9. A. capacity 10. A. overcharged

B. premier B. exhaled B. demolition B. mark B. verge B. authority B. sediments B. gadgets B. competence B. overstated

C. previous C. incorporated C. detachment C. compliment C. threshold C. administrator C. dirt C. fads C. intelligence C. overshadowed

D. fundamental D. digested D. demise D. indicate D. perimeter D. proprietor D. powder D. crazes D. bulk D. overwhelmed

Your answers: 1.

2.

3.

4.

5.

6.

7.

8.

9.

10.

Question 2. Fill each of the numbered blanks in the following passage with one suitable word. Write your answers in the space provided. (15p.) The world-famous credit card company Mastercard is removing its name from its company logo. It will follow the likes of Apple and Nike to have a logo (1) _______ has a symbol only and no writing. Mastercard had traditionally (2) _______ a logo that had overlapping yellow and red circles with the word “Mastercard" written over the top. The company has decided to drop the word "Mastercard" (3) ______ use just the two intersecting circles as a wordless logo. The new logo will be used as the brand's (4)______ on credit cards and in stores, as well as on advertising at sports and (5) _______ events. The company's marketing officer said over 80 per cent of people spontaneously recognized the new symbol (6) ______ the word "Mastercard". Mastercard conducted extensive research for more than 20 months (7) _____ how recognizable the new logo would be without its name being on it. The new logo is part of a wider strategy to transition from (8) _______ a credit card company to a digital payments and financial services company. The world is now full of instantly recognizable logos that contain no words. Apple's bitten apple logo and Nike's swoosh (9) _______ prime examples. Most of the logos of apps on people's smartphones are just symbols. A design expert spoke about maximizing the effect of a small (10) _______ on a phone or watch. He said: "You're trying to optimize for a very small piece of real estate on a very small piece of glass. A 10letter name is kind of a monster."


Your answers: 1.

2.

3.

4.

5.

6.

7.

8.

9.

10.

Question 3. Read the following passage and choose the best answer A, B, C or D for each question. Write your answers in the space provided. (10p.) The stylistic innovation in paining known as Impressionism began in the 1870’s. The Impressionists wanted to depict what they saw in nature, but they were inspired to portray fragmentary moments by the increasingly fast pace of modern life. They concentrated on the play of light over objects, people, and nature, breaking up seemingly solid surfaces, stressing vivid contrast between colors in sunlight and shade, and depiction reflected light in all of its possibilities. Unlike earlier artists, they did not want to observe the world from indoors. They abandoned the studio, painting in the open air and recording spontaneous Impressions of their subjects instead of making outside sketches and then moving indoors to complete the work form memory. Some of the Impressionists’ painting methods were affected by technological advances. For example, the shift from the studio to the open air was made possible in part by the advent of cheap rail travel, which permitted easy and quick access to the countryside or seashore, as well as by newly developed chemical dyes and oils that ledto collapsible paint tubes, which enabled artists to finish their paintings on the spot. Impressionism acquired its name not from supporters but from angry art lovers who felt threatened by the new painting. The term “Impressionism” was born in 1874,when a group of artists who had been working together organized an exhibition of their paintings in order to draw public attention to their work. Reaction from the public and press was immediate, and derisive. Among the 165 paintings exhibited was one called Impression: Sunrise, by Claude Monet (1840-1926), Viewed through hostile eyes, Monet’s painting of a rising sun over a misty, watery scene seemed messy, slapdash, and an affront to good taste. Borrowing Monet’s title, art critics extended the term “Impressionism” to the entire exhibit. In response, Monet and his 29 fellow artists in the exhibit adopted the same name as a badge of their unity, despite individual differences. From then until 1886 Impressionism had all the zeal of a “church”, as the painter Renoir put it. Monet was faithful to the Impressionist creed until his death, although many of the others moved on to new styles. 1. What aspect of painting in the nineteenth century does the passage mainly discuss? A. The impact of some artists’ resistance to the fast pace of life B. The differences between two major styles of art C. A technological advance in the materials used by artists D. A group of artists with a new technique and approach to art


2. The word “depict” in line 2 is closest in meaning to A. reorganize B. deform C. represent D. justify 3. According to the passage, which of the following was one of the distinguishing characteristics of Impressionist painting? A. The emphasis on people rather than nature scenes B. The way the subjects were presented from multiple angles C. The focus on small solid objects D. The depiction of the effects of light and color 4. Which of the following is a significant way in which Impressionists were different from the artists that preceded them? A. They began by making sketches of their subjects B. They painted their subjects out-of-doors C. They preferred to paint from memory D. They used subjects drawn from modern life 5. The word “advent” in line 13 is closest in meaning to A. achievement B. acceptance C. arrival D. advantage 6. The exhibition of paintings organized in 1874 resulted in all of the following EXCEPT A. attracting attention from the public B. a negative reaction from the press C. an immediate demand for the paintings exhibited D. creating a name for a new style of painting 7. The word “affront” in line 23 is closest in meaning to A. insult B. encouragement C. return D. credit 8. The rejection of the Impressionist exhibition by critics was caused by which of the following? A. The small number of paintings on display B. Lack of interest in exhibitions by young artists C. The similarity between all the paintings exhibited D. Anger about seemingly poorly painted art 9. The author mentions Renoir in line 27 to give an example of an artist who A. became as famous as Monet B. was consistently praised by art critics C. described the enthusiasm of the Impressionists for their work D. was in favor of a traditional style of painting 10. The word “others” in line 28 refers to A. art critics B. fellow artists C. individual differences D. new styles


Your answers: 1.

2.

3.

4.

5.

6.

7.

8.

9.

10.

Question 4. You are going to read a newspaper article about the effect of having a dog in the classroom. For question 1-10 choose from the paragraphs (A-G). The paragraphs may be chosen more than once. (10p.)

Paws for thought Buying a dog for a school isn’t a barking mad idea, says Mary Braid. Man’s best friend is also a useful classroom assistant.

A Henry is the undisputed star of Dronfield school near Sheffield. Whatever the achievements of other members of the comprehensive school, it is Henry with his soulful eyes and glossy hair, who has hogged the limelight, appearing on television in Britain and abroad. Yet despite all the public adulation, Henry stirs up no envy or resentment among the 2,000 students at Dronfield High – in fact, they all adore him. The pupils say the Cavalier King Charles spaniel is simply a pupil’s best friend. Their teachers make even bigger assertions for Henry. They say the dog, who first arrived six months ago, is a super dog, who has improved pupil behaviour and encouraged more students to turn up regularly for their lessons and focus on their academic achievement.

B ‘It’s hard not to drift off in a large class sometimes’, explains Andrew Wainwright, 15, who like everyone ebe, is crazy about Henry. ‘So when I go to catch-up classes, Henry is always in the room where they’re held. He helps me focus and get on with it.’ Andrew says Henry is a calming influence although he is unsure of why this might be. But he knows that there’s something magical about being able to throw Henry a soft toy or have Henry lick his hand while he is studying. ‘If we fall behind, Miss Brown won’t let us look after him and everyone wants to walk Henry.’

C Wendy Brown is Andrew’s teacher. It was Brown and Julie Smart, the school counsellor, who first proposed buying a school dog. ‘Julie and I grew up with dogs and we were talking one day about how looking after dogs can affect children’s conduct,’ says Brown. ‘We did some research and discovered that the presence of pets has been shown to be therapeutic. A number of studies have found that animals improve recovery after surgery or illness and have a calming influence on people in lots of settings. Some of my kids can be a handful and some of the children Julie counsels have terrible problems.’

D The two teachers could have plucked a dog from a rescue centre but felt that those dogs were more likely to have difficulties. What they and what troubled children needed was a


stable, intelligent, people-loving animal. Step forward then puppy Henry, purchased from a local breeder, Julie looks after him after school hours – information that has calmed the animal lovers who complained to the school about Henry’s treatment. ‘They seemed to think we locked him in a school cupboard overnight,’ says Brown. ‘Also, the school budget was too tight to buy a dog and you can imagine that putting one before books might have stirred some people a bit. We wanted the least controversy possible so we settled on approaching local churches. They donated the funds to buy him and his favourite food.’

E Today Henry is on Dronfield’s front line when it comes to helping children struggling with everything from attention problems to a sudden death in the family. In the next few weeks, the dog will launch his own confidential counselling website, Ask Henry. Pupils will be encouraged to email and describe whatever is worrying them and Julie will answer on Henry’s behalf. Wouldn’t teenagers run horrified from such a scheme? Apparently not when Henry is involved! ‘Henry has been a massive success,’ insists Brown, explaining that even doubting staff have finally been won round. Perhaps that is because Henry, who lies on the floor during staff meetings, has also had a calming influence on teachers, ‘Not part of the plan,’ says Brown, ‘but a very welcome benefit.’

F Could the school dog become a craze? Brown has already been contacted by eight schools keen to get their own dog. Other schools such as the Mulberry Bush, a primary school for 36 children with emotional and behavioural problems, have stepped forward to point out they already have one. Rosie Johnston, a Mulberry staff member, first brought her golden retriever, Muskoka, into school when he was just nine weeks old. That was three years ago. Aside from being a calming influence, Muskoka even plays his part in literacy lessons. Children at the school can be too shy to read to adults so they read to Muskoka. ‘Their anxiety about mispronouncing something or getting the words in the wrong order is reduced when they read to him,’ says Johnston.

G Psychologist Dr Deborah Wells from Queen’s University Belfast specializes in animalhuman interaction. She believes the underlying key to the Henry’s effect is that dogs offer unconditional live and that cheers up adults and children and helps with self-esteem. But traditionalist Chris Woodhead, the former chief inspector of schools says, ‘I can see how children with behavioral difficulties might be helped but I’m skeptical about the use of dogs in mainstream education. I don’t see why a teacher cannot create a positive learning environment through the subject they teach and their personality. Dogs strike me as a bit of a publicity stunt. It’s the kind of sentimental story journalists love.’ But Henry remains as popular as ever. He’s just become the first animal to be made as an honorary member of the public services union Unison – in recognition of his services as a canine classroom assistant. a way that students can overcome their fear of making mistakes the criteria regarding the selection of an appropriate dog

1. _____ 2. _____


the claim that a dog has increased the students’ attendance at school a motivating reason for students to keep up with their school work evidence to back up the theory that dogs can improve physical well-being people eventually being persuaded that a dog at school is beneficial a decision which was taken to avoid provoking people the accusation that schools have dogs just to attract media attention a welcome positive effect on a group of people that Wendy Brown had not anticipated the fundamental reason why dogs can have a positive impact on people’s happiness

3. _____ 4. _____ 5. _____ 6. _____ 7. _____ 8. _____ 9. _____ 10. _____

Your answers 1.

2.

3.

4.

5.

6.

7.

8.

9.

10.

Question 5. You are going to read an extract from a magazine article. Five paragraphs have been removed from the extract. Choose from the paragraph A-F the one which fits each gap (1-5). There is one extra paragraph which you do not need to use. Write your answers in the space provided. (15p.)

The boy who broke every rule in the book Was Nicholas Culpeper a medical rebel who challenged the establishment, or simply a quack, asks Scarlett Thomas. Anyone who has ever used peppermint tea to ease indigestion or taken chamomile for a good night’s sleep has been using herbal medicine. However suspicious some of us may be of a complete system of ‘alternative’ healing, we all know that, for example, vinegar is good on wasp stings, and honey helps a sore throat. 1 These are questions which have persisted for centuries. Who has the right to medical knowledge? And how could you make sure you were in safe hands? It is to the 16th century, with its complex medical system of quacks, midwives, apothecaries and a few physicians, that Benjamin Wooley first takes us in his book The Herbalist. We learn of Henry VIII’s answer to the problem of national regulation: the creation of the College of Physicians, the members of which were given licensing and fining powers – but not the power to dispense medicines, which was instead held by the apothecaries, the pharmacists of the time.


Mutual distrust and rivalry between these groups seem to have defined the medical system of the next, 100 years. It wasn’t until the great plague that things were shaken up. London was left almost empty of doctors, with only apothecaries still providing medical care. Although they were supposed to practise only in accordance with the Pharmacopoeia Londinensis, a huge book of instructions and recipes created by the College of Physicians, most apothecaries did not actually read Latin. This inability meant that they could not in fact read the book. 2 Even without Latin, most apothecaries had some idea of what their medicines did. And despite not understanding the Latin slurs on their characters in the Pharmacopoeia, the apothecaries also knew that the College had it in for them. In 1634, Nicholas Culpeper, aged 18, arrived in London with £50 in his pocket, looking for an apprenticeship. He soon became an apprentice to an apothecary, becoming familiar with long lists of ‘simple’ ingredients set out in the Pharmacopoeia, including bizarre items like human blood and earthworms. 3 So eventually abandoning his apprenticeship and despite all the rules created by the College of Physicians, Culpeper set up on his own as an ‘independent’, trading out of a shop in London’s Threadneedle Street. His aim was to provide medical help for anyone who needed it and to treat people with simply prepared, locally sourced medicines. This career was interrupted by a stint as a soldier in the Civil War. It was shortly after it ended in 1649 that there was a widespread call for all legal matters to be conducted in English, so justice could be heard and understood by all. 4 When it appeared, it was twice as long as the original, bulging with additions and corrections. It also explained what the recipes were for. “In translating the book,” Woolley notes, “Nicholas broke every rule in it.” This was seen not just as a medical and but a deeply political one. The College of Physicians was outraged. 5 Was Culpeper a quack? No more so than the medical establishment of the time, argues Woolley. It was the College’s Pharmacopoeia after all that recommended the use of the treatments based on ground gall stones of Persian goats that surely led to King Charles II’s death. Yet Culpeper’s legacy – the idea that medicine is not something that should be controlled by the elite but something belonging to everybody – is as important now as it was in the 17th century.

A Perhaps this was fortunate, as the book warned of the deceit of those people who are allowed to sell the most filthy concoctions, and even mud, under the name and title of medicaments for the sake of profit. This was undoubtedly an attack on the capabilities and moral principles of the apothecaries.


B As odd as these may seem, many recipes would also call for the drug opium, which at the time, cost less than garden rhubarb. Culpeper did not have a good experience at this time, being assigned a new master on several occasions. Then again, this was probably not a good time for anyone to be in his position, when rules meant you could be summoned to a company ‘court’ for having ‘stubbornness and long hair’.

C The English Physician, Culpeper’s later book, better known as Culpeper’s Complete Herbal, did little to pacify them. It outlined not only the uses of healing plants but also Culpeper’s holistic view of medicine. Despite upsetting the establishment, it became one of the most popular and enduring books in British history.

D When things get more serious, of course, most people rush to the doctor. But what if the doctor gets it wrong? Or imagine a situation when, for whatever reasons, you wanted to find out how to use other plants to heal yourself.

E It reveals a profound insight into the trade practices of the time, and how the establishment view of who should be allowed to trade and under what conditions affected everything. This was especially true concerning the health of people denied control over their medical treatment.

F Impressed by this, Culpeper’s thoughts turned to a similar democratization of medical texts. These thoughts would be made reality when he was commissioned to produce an English edition of the Pharmacopoeia. Your answers 1.

2.

3.

4.

5.

PART IV: WRITING (60p.) Question 1. Read the following extract and use your own words to summarise it. Your summary should be about 140 words long. (15p.) Buying things today is so simple. Just enter a shop, say a book store, choose the desired book and pay for it. Long ago, before the invention of money, how did people trade? The most primitive way of exchange should be the barter trade. In this form of transaction, people used goods to exchange for the things that they had in mind. For instance, if person A wanted a book and he had a spare goat, he must look for someone who had the exact opposite, that is, that someone, say person B, must have a spare book of person A's choice and is also in need of a goat. Having found such a person, the problem does not end here.


A big goat may worth not only one book, hence person B may have to offer person A something else, say five chickens. However, he runs the risk of person A rejecting the offer as he may not need the chickens. The above example clearly illustrates the inefficiency of barter trading. Many years later, the cumbersome barter trade finally gave way to the monetary form of exchange when the idea of money was invented. In the early days, almost anything could qualify as money: beads, shells and even fishing hooks. Then in a region near Turkey, gold coins were used as money. In the beginning, each coin had a different denomination. It was only later, in about 700 BC, that Gyges, the king of Lydia, standardized the value of each coin and even printed his name on the coins. Monetary means of transaction at first beat the traditional barter trade. However, as time went by, the thought of carrying a ponderous pouch of coins for shopping appeared not only troublesome but thieves attracting. Hence, the Greek and Roman traders who bought goods from people faraway cities, invented checks to solve the problem. Not only are paper checks easy to carry around, they discouraged robbery as these checks can only be used by the person whose name is printed on the notes. Following this idea, banks later issued notes in exchange for gold deposited with them. These bank notes can then be used as cash. Finally, governments of today adopted the idea and began to print paper money, backed by gold for the country's use. _________________________________________________________________________ _________________________________________________________________________ _________________________________________________________________________ _________________________________________________________________________ _________________________________________________________________________ _________________________________________________________________________ _________________________________________________________________________ _________________________________________________________________________ _________________________________________________________________________ _________________________________________________________________________ _________________________________________________________________________ _________________________________________________________________________ _________________________________________________________________________ _________________________________________________________________________


Question 2. The table below shows the worldwide market share of the notebook computer market for manufacturers in the years 2006 and 2007. Summarise the information by selecting and reporting the main features, and make comparisons where relevant. You should write about 150 words. (15p.)

Company

2006 % Market Share

2007 % Market Share

HP

31.4

34

Dell

16.6

20.2

Acer

11.6

10.7

Toshiba

6.2

7.3

Lenovo

6.6

6.2

Fujitsu-Siemens

4.8

2.3

Others

22.8

19.3

_________________________________________________________________________ _________________________________________________________________________ _________________________________________________________________________ _________________________________________________________________________ _________________________________________________________________________ _________________________________________________________________________ _________________________________________________________________________ _________________________________________________________________________ _________________________________________________________________________ _________________________________________________________________________ _________________________________________________________________________ _________________________________________________________________________ _________________________________________________________________________ _________________________________________________________________________ _________________________________________________________________________


Question 3. Give your response to the following statement. You should write about 300350 words (30p.) In order for a child to develop his/her family life, mothers and fathers should take a course on parenting skills and child development each year. Do you agree or disagree? _________________________________________________________________________ _________________________________________________________________________ _________________________________________________________________________ _________________________________________________________________________ _________________________________________________________________________ _________________________________________________________________________ _________________________________________________________________________ _________________________________________________________________________ _________________________________________________________________________ _________________________________________________________________________ _________________________________________________________________________ _________________________________________________________________________ _________________________________________________________________________ _________________________________________________________________________ _________________________________________________________________________ _________________________________________________________________________ _________________________________________________________________________ _________________________________________________________________________ _________________________________________________________________________ _________________________________________________________________________ _________________________________________________________________________ _________________________________________________________________________ _________________________________________________________________________ _________________________________________________________________________ _________________________________________________________________________ -THE END-


SỞ GIÁO DỤC & ĐÀO TẠO PHÚ THỌ TRƯỜNG THPT CHUYÊN HÙNG VƯƠNG

KÌ THI CHỌN HỌC SINH GIỎI KHU VỰC DUYÊN HẢI BẮC BỘ NĂM HỌC 2019- 2020

ĐÁP ÁN ĐNHNGHỊ

PART ONE. LISTENING (50p.) Question 1. For questions 1-5, listen to a discussion on the subject of identity and the labels that identify groups of people in society and choose the correct answer A, B, C, or D which fits best according to what your hear. Write your answers in the corresponding numbered boxes provided. (10p.) 1. D

2. C

3.A

4. B

5. A

Questions 2.For questions 1-5, listen to a complaint from a woman called Julie Gold and decide whether these statements are True (T) or False (F). Write your answers in the corresponding numbered boxes provided. (10p.) 1. F

2. T

3. T

4. F

5. F

Question 3: You will hear an interview with a man called Mark Phillips, who is talking about his work as a potter. Answer the questions (1-5) with NO MORE THAN FIVE WORDS. (10p.) 1. He was more into music. 2. (She suggested) pottery evening classes. 3. They are practical things. 4. They are helpful. 5. (He will) get to grips with learning.

Questions 4.For questions 1-10, listen to a report on Finland, the global education leader and supply the blanks with the missing information. Write NO MORE THAN FOUR WORDS taken from the recording for each answer in the space provided. (20p.)


1.37th

2. angry birds

3. mindless drudgery

4. vast social safety net

5. recess time

6. teaching credential

7. hands-on learning

8. social ills

9. proven track record

10. educational innovators

PART TWO. LEXICO – GRAMMAR (30p.) Question 1. Choose the word or phrase which best completes each sentence. Write your answers (A, B, C, or D) in the space provided under this part.(10p.) 1.C

2.B

3.D

4.B

5.C

6.D

7.D

8.A

9.D

10.B

Question 2.There are FIVE mistakes in this paragraph. Write them down & give the correction. Write your answers in the space provided. (5p.) Line

Mistake

Correction

1.

1

deadly

Dead

2.

3

weighed

outweighed

3.

3

on

Of

4.

7

though

But

5.

8

on

in

Question 3. Fill in each gap with ONE preposition to finish the following sentences. Write your answers in the space provided. (5p.) 1. up to

2. against

3.with

4. up on

5. off

Question 4. Use the correct form of each bracketed word in the numbered space. Write your answers in the space provided. (10p.) 1.produced

6. infomercials

2. watershed

7. truly


3. bygone

8. pickings

4. censorship

9. netizen

5. commercial

10. yesteryear

PART THREE: READING (60p.) Question 1. Read the text below and decide which answer (A, B, C or D) best fits each gap. Write your answers in the space provided. (10p.)

1. C

2. A

3. D

4. D

5. C

6. B

7. B

8. A

9. A

10. D

Question 2. Fill each of the numbered blanks in the following passage with one suitable word. Write your answers in the space provided. (15p.) 1. which 6. without

2. used 7. on

3. and 8. being

4. symbol 9. are

5. other 10. logo

Question 3. Read the following passage and choose the best answer A, B, C or D for each question. Write your answers in the space provided. (10p.) 1. D 2. C 3. D 4. B 5. C 6. C

7. A

8. D

9. C

10. B

Question 4. You are going to read a newspaper article about the effect of having a dog in the classroom. For question 1-10 choose from the paragraphs (A-G). The paragraphs may be chosen more than once. (10p.) 1. F

2. D

3. A

4. B

5. C

6. E

7. D

8. G

9. E

10. G

Question 5. You are going to read an extract from a magazine article. Five paragraphs have been removed from the extract. Choose from the paragraph A-F the one which fits each gap (1-5). There is one extra paragraph which you do not need to use.Write your answers in the space provided. (15p.) 1. D 2. A 3. B 4. F 5. C


PART IV: WRITING (60p.) Question 1. Read the following extract and use your own words to summarise it. Your summary should be about 140 words long.(15p.) Contents (10 points): - The summary MUST cover the following points: + The topic sentence must cover: the development of methods of payment + The earliest type: goods exchanging>> its weakness lies in the inequality of the exchanged things and the users’ demand>>proved to be ineffective way of trading + The following form : the use of coins>> at first were more convenient than rational exchanging>> later became a burden for shopper and appeal thieves + Checks and notes used as cash were invented among the country - The summary MUST NOT contain personal opinions. Language use (5 points) The summary: - should show attempts to convey the main ideas of the original text by means of paraphrasing (structural and lexical use), - should demonstrate correct use of grammatical structures, vocabulary, and mechanics (spelling, punctuations,....), - should maintain coherence, cohesion, and unity throughout (by means of linkers and transitional devices).

Question 2. The table below shows the worldwide market share of the notebook computer market for manufacturers in the years 2006 and 2007. Summarise the information by selecting and reporting the main features, and make comparisons where relevant. You should write about 150 words. (15p.) The mark is based on the following marking scheme: 1. Task fulfillment (5p): + fully satisfies all the requirements of the task ƒ + clearly presents a fully developed response 2. Coherence and Cohesion (5p): + uses cohesion in such a way that it attracts no attention ƒ + skillfully manages paragraphing 3. Lexical Resources (5p):


+ uses a wide range of vocabulary with very natural and sophisticated control of lexical features; + rare minor errors occur only as ‘slips’ 4. Grammatical Range and Accuracy (5p): + uses a wide range of structures with full flexibility and accuracy; + rare minor errors occur only as ‘slips’ Cái này thừa điểm Question 3. Give your response to the following statement. You should write about 300-350 words (30p.) In order for a child to develop his/her family life, mothers and fathers should take a course on parenting skills and child development each year. Do you agree or disagree? The mark is based on the following scheme: 1. Content: (10p.) a provision of all main ideas and details as appropriate. 2. Language: (10p.) a variety of vocabulary and structures appropriate to the level of English language gifted upper-secondary school students 3. Presentation: (10p.) coherence, cohesion, and style appropriate to the level of English language gifted upper-secondary school students


KỲ THI HỌC SINH GIỎI CÁC TRƯỜNG THPT CHUYÊN KHU VỰC DUYÊN HẢI VÀ ĐỒNG BẰNG BẮC BỘ LẦN THỨ XII, NĂM 2019 ĐỀ THI MÔN: TIẾNG ANH LỚP 11 Thời gian: 180 phút (Không kể thời gian giao đề)

ĐỀ CHÍNH THỨC

Ngày thi:

(Đề thi gồm 18 trang)

(Thí sinh làm bài trực tiếp vào đề)

Điểm Bằng số

Giám khảo1

Bằng chữ

Giám khảo 2

Số phách

HƯỚNG DẪN PHẦN THI NGHE HIỂU •

Bài nghe gồm 4 phần, mỗi phần được nghe 2 lần, mỗi lần cách nhau 30 giây, mở đầu và kết thúc mỗi phần nghe có tín hiệu.

Mở đầu và kết thúc bài nghe có tín hiệu nhạc. Thí sinh có 3 phút để hoàn chỉnh bài trước tín hiệu nhạc kết thúc bài nghe.

Mọi hướng dẫn cho thí sinh (bằng tiếng Anh) đã có trong bài nghe.

A. LISTENING (50 pts) Part 1: You will hear a group of art history students going around an art gallery with their teacher. For questions 1-5, choose the correct answer (A, B, C or D) which fits best according to what you hear. (10pts) 1. Burne-Jones believed that a painting________. A. ought to be true to nature. B. must have a clear moral point. C. should play an instructive role in a modern industrial society. D. need not have any practical value. 2. It appears that the story of the King and the Beggar Maid was________. A. a well-known Victorian tale. B. popularized by a poet. C. brought to the artist’s attention by his wife. D. taken up by novelists at a later stage. 3. According to the student, how did the painter approach the work? Page 1 of 18


A. He wanted to portray the beggar very realistically. B. He copied parts of the painting from an Italian masterpiece. C. He had certain items in the painting made for him. D. He wanted to decorate the clothing with jewels. 4. The student thinks that in some way the painting depicts________. A. an uncharacteristically personal message B. the great sadness of the artist C. the artist’s inability to return the girl’s love D. the fulfillment of the artist’s hopes and dreams 5. What was people’s reaction to the painting? A. They recognized Frances Graham as the model for the Beggar Maid B. They realized how personal the painting was for the artist C. They interpreted the painting without difficulty D. They did not approve of the subject matter of the painting Your answers: 1.

2.

3.

4.

5.

Part 2: Listen to the audio about mobile devices and decide whether the following sentences are true (T) or false (F). (10 pts) 1.

Nowadays the smallest screens have the biggest potential.

2.

Mobile viewing is growing ten times as fast as desktop viewing.

3.

Mobile viewing means there will be less opportunity for companies to sell advertising.

4.

300 hours of video are uploaded every minute on YouTube

5.

Monetization is not a struggle on mobile devices at the moment.

Your answers: 1.

2.

3.

4.

5.

Part 3: You will hear an extract from a talk given to a group who are going to stay in the UK. Listen and give short answers to the questions. Write NO MORE THAN THREE WORDS AND/OR A NUMBER taken from the recording. (10 pts) What TWO factors can make social contact in a foreign country difficult? • 1 ............................... • 2 ............................... Page 2 of 18


Which types of community group does the speaker give examples of? • theatre • music • 3 .................................. In which TWO places can information about community activities be found? • 4 .................................. • 5 .................................. Your answers: 1.

2.

3.

4.

5.

Part 4: Listen to a Ted talk named “Try something new for 30 days” and complete the summary below. WRITE NO MORE THAN THREE WORDS OR A NUMBER in the corresponding numbered boxes. (20 pts) The speaker decided to follow in the footsteps of the (1)________________________, Morgan Spurlock, and try something new for 30 days. Think about something you've always wanted to add to your life and try it for the next 30 days. It turns out 30 days is just about the right amount of time to add a new or (2)_______________. There's a few things he has learnt while doing these 30-day challenges. Firstly, instead of the months flying by, forgotten, the time was (3)__________________. Also, he noticed that as he started to do more and harder 30-day challenges, his (4)_____________. Last year, he finished hiking up Mt. Kilimanjaro, the highest mountain in Africa, which he would never have been that (5)_______________ before he started my 30-day challenges. Moreover, if you really want something (6)______________, you can do anything for 30 days. For the writing a novel, the secret is not to go to sleep till you've written your words for the day. You might be (7)______________, but you'll finish your novel. Finally, when making (8)_________________, things he could keep doing, they were more likely to stick. There's nothing wrong with (9)________________. In fact, they're a ton of fun. But they're less (10)__________________. Your answers: 1.

6.

2.

7.

3.

8.

4.

9.

5.

10.

Page 3 of 18


B. LEXICO AND GRAMMAR (30 pts) Part 1: Choose the word/ phrase that best completes each of the following sentences. Write your answer in the corresponding numbered boxes. (10 pts) 1. We weren’t close friends but she ________ me well when I left college. A. hoped

B. boded

C. wished

D. bade

2. I don’t want to be too ________ on Alice, but I think I should tell her that her work isn’t good enough. A. stringent

B. stern

C. strict

D. hard

3. The management are making ________ to increase the company’s efficiency. A. measures

B. steps

C. moves

D. deeds

4. I was told yesterday that the car had been fixed but it’s broken down ________ again! A. up

B. still

C. yet

D. on

5. The anthology contains________ the work of modern poets, but it includes a few significant older works as well. A. mostly

B. most

C. at most

D. uppermost

6. Son: “This herb smells horrible!” Mom: “Be that as it may, it will do you a ________ of good” . A. power

B. wealth

C. strike

D. visit

7. As a matter of ________, we have six security guards on the premises at all times. A. wisdom

B. prudence

C. foresight

D. acumen

8. It was such a sad movie that we were all reduced ________ tears. A. for

B. with

C. into

D. to

9. She marched into the shops, as bold as ________, and demanded for her money back. A. bass

B. grass

C.brass

D. glass

10. Jane is out in the garden mulling________ a problem to do ________work. A. over/ with

B. away/ without

C. in/to

D. Into/for

Your answers: 1.

2.

3.

4.

5.

6.

7.

8.

9.

10.

Part 2: The text below contains 5 mistakes. Underline the mistakes and write the corrections in the space provided. (5 pts). There is an example at the beginning (0) Your answers No educational media better serves as a means of spatial

(0) Line1:media-> medium

communication than the atlas. Atlases deal with

Line 2:__________

such valueless information as population distribution and

Line 3:__________

density. One of the best, Pennycooke’s World Atlas, has been

Line 4:__________

Page 4 of 18


widely accepted as a standard owing to the quality of its maps

Line 5:__________

and photographs, which not only show various settlements but

Line 6:__________

also portray them in a variety of scales. In fact, the very first

Line 7:__________

map in the atlas is a clever designed population cartogram that

Line 8:__________

projects the size of each country if geographical size were

Line 9:__________

proportional to population. Followed the proportional layout, a

Line 10:_________

sequence of smaller maps shows the world’s population density,

Line 11:_________

each country’s birth and death rates, population increase and

Line 12:_________

decrease, industrialization, urbanization, gross national product

Line 13:_________

in terms of per capital income, the quality of medical care,

Line 14:_________

literacy, and language. To give readers a perspective on how

Line 15:_________

their own country fits in with the global view, additional

Line 16:_________

projections despite the world’s patterns in nutrition, calorie and

Line 17:_________

protein consumption, health care, number of physicians per unit

Line 18:_________

of population, and life expectancy by region. Population density

Line 19:_________

maps in a subcontinental scale, as well as political maps, convey

Line 20:_________

the diverse demographic phenomena of the world in a broad

Line 21:_________

array of scales.

Part 3. Complete each of the following sentences with a suitable preposition or particle. Write your answer in the corresponding numbered boxes. (5 pts)

1. I don't think we can run____________ a holiday abroad this year. 2. July is always sending ____________ the French teacher. 3. You will soon take ____________ your new boss, I’m sure. 4. The police tracked ____________ the killer and arrested him. 5. Paul talked me ____________ going skiing, against my better judgement. Your answers: 1.

2.

3.

4.

5.

Part 4: Write the correct form of each bracketed word in corresponding numbered boxes. (10 pts) Dickens and his world It was with the circulation of Pickwich Papers in 1836 that young Dickens began to enjoy a truly (1. PRECENDENT) _________ ascent into the favour of the Birtish reading public. He magnificently (2. PROVE) _________ a theory that his fame would disapear just as quickly as it had come. He ramained until his death 34 years later (3. DENY) _________ the most popular noverist the English- speaking world had ever known. Page 5 of 18


The public displayed a/an (4. SATIATE) _________ appetite for his works, and there was also a great diffusion of them through (5. NUMBER) _________ dramatic adaptaions (nearly all completely (6. AUTHORITY) _________ the copyright laws being much weaker in those days). His immense popularity was based on the widespread perception of him as a great champion of the poor and the (7. POSSESS) _________ against all forms of (8. JUST) _________ and abuse of power. In his personal life, however, he was (9. CAPACITY) _________ of achieving the level of fulfiment he enjoyed with the public, and all his close emotional relationships with women (10. VARY) _________ ended in failure. Yet, he created an extraordinary range and variety of female characters who live on in our minds and culture unlike any others created by Victorian novelists. Your answers: 1.

2.

3.

4.

5.

6.

7.

8.

9.

10.

C. READING (60pts) Part 1: For questions 1–10, read the following passages and decide which answer (A, B, C or D) best fits each gap. Write your answers (A, B, C, or D) in corresponding numbered boxes. (10pts) CELEBRITY ROLE-MODELS Research in the University of Leicester Department of Media and Communication examined interest in celebrities and gossip about them. It was carried out by Dr Charlotte De Backer who sought in her study to explain interest in celebrity culture. According to Dr De Backer: ‘Life is about learning and (1)_______ experience, and in that process we have a tendency to observe and mimic the actions of others. Ideally we mimic what makes others successful and (2)_______ unsuccessful actions others have trialled and paid for. In reality, humans seem to have the tendency to mimic the overall behaviour pattern of the higher status of those more successful than themselves. This explains why celebrities act as role models for broad (3) _______of behaviour they display whether good or bad.’ Dr De Backer also examined another theory for interest in celebrity, known as the Parasocial Hypothesis. In this (4) _______ , the bonds are parasocial, or one-way, because the celebrity reveals private information, often voluntarily. The audience members respond emotionally to this information, although there is hardly ever any feedback on the private life of the audience going to the celebrity, nor do celebrities (5) _______ emotions towards their audience. Her study of 800 respondents and over 100 interviews (6) _______ that younger participants showed greater interes in celebrity gossip, even if it was about celebrities who were much older than them and even when they did not know who the celebrities were. They showed greatest interest in internationally-known celebrities, because they considered those as more (7) _______.

Page 6 of 18


Her study also found that older people were interested in celebrity gossip not because they wanted to learn from the celebrities, but because it helped them to form social networks with other people. ‘We found in the interviews that older people do not gossip about celebrities becuase they want to learn from them or feel (8) _______ by them, but because they use celebrity gossip to (9) _______ with real-life friends and acquaintances. As we live in (10) _______ societies, celebrities can act as our mutual friends and acquaintances.’ 1. A. winning

B. gaining

C. achieving

D. capturing

2. A. escape

B. advoid

C. prevent

D. evade

3. A. reaches

B. domains

C. ranges

D. spheres

4. A. case

B. instance

C. state

D. position

5. A. exhibit

B. present

C. display

D. expose

6. A. reinforced

B. assured

C. validated

D. confirmed

7. A. reputable

B. honourable

C. prestigious

D. illustrious

8. A. befriended

B. sustained

C. patronized

D. upheld

9. A. tie

B. link

C. cement

D. bond

10. A. scattered

B. rare

C. mixed

D. idle

Your answers: 1.

2.

3.

4.

5.

6.

7.

8.

9.

10.

Part 2: Read the text below and think of one word which best fits each space. Use only ONE WORD for each space. Write your answers in the corresponding numbered boxes. (10 pts) Busy, busy, busy With obesity having shot up across the globe to dangerously high levels in recent years, it is litle wonded that people have started to ask why. True, (1)______ have changed; we all know that we live in a McWorld, hunting and gathering our food from fast-food outlets and supermarket aisles, but it can’t all be down to diet, can it? Technology has changed modern life to such an extent that few aspects of life today bear any (2)______ to lives only a couple of generations ago. Just taking fobs as an example, how many of us today spend twelve hours a day on our feet physically slogging ourselves (3)______ the ground? Or how many families could you imagine living without a car? Kids walking to school, parents going to half a (4)______ local shops, on foot, to buy the week’s food, family holidays by bus to the nearest seaside town. Take Tina Jameson, a mother of two (5)______ has to juggle home and a part-time job. She says ‘I have’t got time to walk annywhere. But I’d have even (6)______ time without a washing machine or dishwasher’. We now have so many (7)______ in our lives that

Page 7 of 18


allow us such drastically better lifestyle choices that at times it can be difficult to picture these in a negative way. Without doubt there are (8)______ to these changes. The number of people who suffer debilitaing injuries at work is miniscule in comparison to the past. Fewer hours working and more efficient transport are all (9)______ our benefit in allowing us a greater amount of leisure time. At what cost though? We may save a few hours day travelling and enjoy less physically demanding working conditons, but is this really (10)_____ it when the cost to our health and life expectancy is so high? Modern lifestyles have become shockingly sedentary and incombinaton with the deterioration in diet this is surely creating a ticking time bomb for modern humanity. Your answers: 1.

2.

3.

4.

5.

6.

7.

8.

9.

10.

Part 3: Read the following passage and choose the best answer (A, B, C or D) according to the text. Write your answers (A, B, C or D) in the corresponding numbered boxes (15pts). From the article "Against the Undertow: Language-Minority Education Policy and Politics in the 'Age of Accountability'" by Terrence G Wiley and Wayne E. Wright

Language diversity has always been part of the national demographic landscape of the United States. At the time of the first census in 1790, about 25% of the population spoke languages other than English (Lepore, 2002). Thus, there was a diverse pool of native speakers of other languages at the time of the founding of the republic. Today, nationwide, school districts have reported more than 400 languages spoken by language-minority students classified as limited English proficient (LEP) students (Kindler, 2002). Between 1991 and 2002, total K-12 student enrollment rose only 12%, whereas LEP student enrollment increased 95% during this same time period (National Clearinghouse for English Language Acquisition, 2002b). This rapid increase and changing demographics has intensified the long debate over the best way to educate language-minority students. Historically, many groups attempted to maintain their native languages even as they learned English, and for a time, some were able to do so with relatively little resistance until a wave of xenophobia swept the country during World War 1 (Kloss, 1977/1998). Other groups, Africans, and Native Americans encountered repressive politics much earlier. During the 1960s, a more tolerant policy climate emerged. However, for the past two decades there has been a steady undertow of resistance to bilingualism and bilingual education. This article provides historical background and analyzes contemporary trends in language-minority education within the context of the recent national push for accountability, which typically takes the form of high-stakes testing. Page 8 of 18


The origins of persistent themes regarding the popular antagonisms toward bilingual education and the prescribed panaceas of "English immersion" and high-stakes testing in English need to be scrutinized. As background to the contemporary context, we briefly discuss the history of language politics in the United States and the ideological underpinnings of the dominant monolingual English ideology. We analyze the recent attacks on bilingual education for what this attack represents for educational policy within a multilingual society such as the United States. We emphasize multilingual because most discussions of language policy are framed as if monolingualism were part of our heritage from which we are now drifting. Framing the language policy issues in this way masks both the historical and contemporary reality and positions non-English language diversity as an abnormality that must be cured. Contrary to the steady flow of disinformation, we begin with the premise that even as English has historically been the dominant language in the United States since the colonial era, language diversity has always been a fact of life. Thus, efforts to deny that reality represent a "malady of mind" (Blaut, 1993) that has resulted in either restrictionist or repressive language policies for minorities. As more states ponder imposing restrictions on languages of instruction other than English-as California, Arizona, and Massachusetts have recently done-it is useful to highlight several questions related to the history of language politics and language planning in the United States. Educational language planning is frequently portrayed as an attempt to solve the language problems of the minority. Nevertheless, the historical record indicates that schools have generally failed to meet the needs of language-minority students (Deschenes, Cuban, & Tyack, 2001) and that the endeavor to plan language behavior by forcing a rapid shift to English has often been a source of language problems that has resulted in the denial of language rights and hindered linguistic access to educational, social, economic, and political benefits even as the promoters of English immersion claim the opposite. The dominance of English was established under the British during the colonial period, not by official decree but through language status achievement, that is, through "the legitimization of a government's decisions regarding acceptable language for those who are to carry out the political, economic, and social affairs of the political process" (Heath, 1976, p.51). English achieved dominance as a result of the political and socioeconomic trade between England and colonial administrators, colonists, and traders. Other languages coexisted with English in the colonies with notable exceptions. Enslaved Africans were prohibited from using their native tongues for fear that it would facilitate resistance or rebellion. From the 1740s forward, southern colonies simultaneously institutionalized "compulsory ignorance" laws that prohibited those enslaved from acquiring English literacy for similar reasons. These restrictive slave codes were carried forward as the former southern colonies became states of the newly United States and remained in force until

Page 9 of 18


the end of the Civil War in 1865 (Weinberg, 1977/1995). Thus, the very first formal language policies were restrictive with the explicit purpose of promoting social control. 1. What is the primary purpose of including the statistic from the 1790 census in the introductory paragraph? To explain how colonizing the US eradicated language diversity To show concrete evidence that language diversity in the US is not a new phenomenon To note that before that time, there was no measure of language diversity in the US To demonstrate that census data can be inaccurate 2. The article compares two sets of statistics from the years 1991-2002, increases in K-12 enrollment and increases in LEP students, to highlight. That the two numbers, while often cited in research, are insignificant That while many people with school-age children immigrated to the US during this time, an equal amount left the country as well C) That language diversity had no impact on US student enrollment during this time D) That while the total amount of students enrolled in US schools may have grown slowly, the amount of those students who were LEP increased dramatically 3. According to the second paragraph, many groups maintained their native languages without resistance into the 20th century EXCEPT__________. A) Native Americans and African Americans B) Irish Americans and African Americans C) Mexican Americans and Native Americas D) Native Americans and Dutch Americans 4. Why is the word "undertow" emphasized in the second paragraph? A) To explain how certain groups continued to carry their native languages with them despite the opposition from those against language diversity B) To show the secretive and sneaky nature of those opposed to language diversity C) To call attention to the ebb and flow of language resistance during the 20th century, experiencing periods of both rest and extremism D) To explain that, while many groups tried to maintain their native languages, many gave in to social and political pressure to use only English 5. What is the best way to describe the function of the third paragraph in this excerpt?. A) The paragraph provides its primary thesis as well an outline of the article's main points B) The paragraph is an unnecessary and irrelevant inclusion C) The paragraph serves to reveal the conclusions of the article before detailing the data D) The paragraph firmly establishes the article's stance against language diversity

Page 10 of 18


6. What is the best summary of why the phrase "multilingualism" is emphasized in the third paragraph? A) Language repression stems from the US's unwillingness to recognize the languages of its foreign allies B) Because language is constantly changing and often goes through multiple phases over time C) The authors firmly believe that speaking more than one language gives students a substantial benefit in higher education. D) Language policy discussions often assumes that the US has a monolinguistic history, which is untrue and poses language diversity as threatening 7. Phrases such as "prescribed panaceas" and "malady of the mind" are used in the third paragraph to__________. A) Defend the point that the US must standardize its language education or there will be severe results B) Point out that language is as much a physical process as an intellectual one C) Illustrate how certain opponents of language diversity equate multilingual education with a kind of national disease D) Demonstrate how the stress of learning multiple languages can make students ill 8. According to the fourth paragraph, all of the following are potential negatives of rapid English immersion EXCEPT__________. A) It can lead to a denial of language rights for particular groups B) Students become more familiar with conversational expressions and dialect C) It can prevent access to certain benefits that are always available to fluent speakers D) It can promote feelings of alienation among groups that are already in a minority status 9. The best alternate definition of "language status achievement" is __________. A) When enough scholarly work has been produced in a language, it is officially recognized B) Those who are in power socially and economically determine the status of a language C) Languages fall into a hierarchy depending upon the numbers of populations that speak them D) The position of a language in which no others may coexist with it 10. From the context of the final paragraph, what does "compulsory ignorance" mean? A) Populations at the time were required only to obtain a certain low level of education B) Slave populations were compelled to only speak in their native languages and not learn English C) That slaves were forcibly prevented from developing their native language skills out of fear that they would gain power D) Slave owners would not punish slaves who did not wish to learn and speak only English Your answers: Page 11 of 18


1.

2.

3.

4.

5.

6.

7.

8.

9.

10.

Part 4 : Read the following passage and do the tasks that follows. Write your answer in the space provided. (15 pts) Ford – driving innovation 1. In 1913 an American industrialist named Henry Ford employed an innovative system in his factory that changed the nature of American industry forever – the production line. Instead of a group of workers constructing a complete product, Ford’s production-line techniques relied on machine parts being moved around the factory on a conveyor belt, passing each employee who had a single task to perform before the component moved down the line. This saved time in that employees were not required to move around, collect materials or change tools; they simply stood in one place and repeated the same procedure over and over again until the end of their shift. In this way, Ford was able to mass produce the now famous Model-T car for only 10% of traditional labour costs. 2. Working on a production line was monotonous work, undoubtedly, but it was not in the production line alone that Ford was something of a pioneer. In 1913 the average hourly rate for unskilled labour was under $2.50 and for such low wages and repetitive work, the labour turnover in Ford’s factory was high, with many employees lasting less than a month. In order to combat this problem, he took a step that was condemned by other industrialists of the time, fearful that they would lose their own workforce – he raised wages to $5 an hour. The benefits were twofold. Not only did Ford now have a stable and eager workforce, he also had potential customers. It was his intention ‘to build a motorcar for the great multitude’, and the Model-T car was one of the cheapest cars on the market at the time. At $5 an hour, many of his employees now found themselves in a position to feasibly afford a car of their own. Ford’s production practices meant that production time was reduced from 14 hours to a mere 93 minutes. In 1914 company profits were $30 million, yet just two years later this figure had doubled. Until 1927 when the last Model-T rolled off the production line, the company produced and sold about 15 million cars. 3. Although Ford was without doubt successful, times changed and the company began losing its edge. One problem came from the labour force. Ford was a demanding employer who insisted that the majority of his staff remained on their feet during their shift. One error meant that the whole production line was often kept waiting, and Ford felt that workers were more attentive standing than sitting. Yet the 1930s saw some radical changes in the relationships between employer and employee, as an increasing number of industries were forming Labour Unions. Ford flatly refused to get involved, employing spies in the workplace to sabotage any plans for a union within his factories. Eventually a strike in the Page 12 of 18


early 1940s forced Ford to deal with unions. Another example of Ford being unable to adapt came from his unwillingness to branch out. Ford’s competitors began operating the same systems and practices, but also introduced the variety Ford was lacking. The Model-T had remained essentially the same, even down to the colour, and by the time he realised his error, he had already lost his pre-eminence in the industry. Subsequent involvement in aeroplane manufacturing, politics and publishing was a failure. Leaving the company to his grandson in 1945, he died two years later leaving an inheritance estimated at $700 million. 4. Yet the legacy of Fordism lives on. The development of mass production transformed the organisation of work in a number of important ways. Tasks were minutely subdivided and performed by unskilled workers, or at least semiskilled workers, since much of the skill was built into the machine. Second, manufacturing concerns grew to such a size that a large hierarchy of supervisors and managers became necessary. Third, the increasing complexity of operations required employment of a large management staff of accountants, engineers, chemists, and, later, social psychologists, in addition to a large distribution and sales force. Mass production also heightened the trend towards an international division of labour. The huge new factories often needed raw materials from abroad, while saturation of national markets led to a search for customers overseas. Thus, some countries became exporters of raw materials and importers of finished goods, while others did the reverse. 5. In the 1970s and ’80s some countries, particularly in Asia and South America, that had hitherto been largely agricultural and that had imported manufactured goods, began industrialising. The skills needed by workers on assembly-line tasks required little training, and standards of living in these developing countries were so low that wages could be kept below those of the already industrialised nations. Many large manufacturers in the United States and elsewhere therefore began ‘outsourcing’ – that is, having parts made or whole products assembled in developing nations. Consequently, those countries are rapidly becoming integrated into the world economic community Choose the correct heading for paragraphs B-F from the list of headings below. Write the correct number, i-ix, next to Questions 1-5. List of Headings i. Effect on modern industry ii. New payment procedures iii. Labour problems iv. The Model-T v. Creating a market vi. Revolutionary production techniques vii. The Ford family today Page 13 of 18


viii.

Impact on the global economy

ix. Overseas competition

1. Paragraph A 2. Paragraph B 3. Paragraph C 4. Paragraph D 5. Paragraph E Your answers: 1.

2.

3.

4.

5.

Questions 6-10: Complete the summary below. Choose NO MORE THAN THREE WORDS from the passage for each answer. Write your answer in the space provided. The application of (6)______________ and pay rise scheme enabled Ford to generate mass production. However, the issues related to the labor force and (7) the______________ to start doing something new drove Ford to the verge of crisis. One of the long-term effects of Ford’s business practices was that many developing countries became industrialised as a result of (8) ______________ some work to other countries. For those working in the factory, the skills for (9) ______________ were easily acquired and (10) ______________ was minimal.

Part 5: Read the text below and answer questions (10pts). Consultant A A university degree is no guarantee of a job, and job hunting in itself requires a whole set of skills. If you find you are not getting past the first interview, ask yourself what is happening. Is it a failure to communicate or are there some skills you lack? Once you see patterns emerging it will help you decide whether the gaps you have identified can be filled relatively easily. If you cannot work out what the mismatch is, get back to the selection panel with more probing questions, and find out what you need to do to bring yourself up to the level of qualification that would make you more attractive to them: but be careful to make this sound like a genuine request rather than a challenge or complaint. Consultant B Do not be too dispirited if you are turned down for a job, but think about the reasons the employers give. They often say it is because others are 'better qualified', but they use the term loosely. Those who made the second interview might have been studying the same subject as you and be of similar ability level, but they had something which made them a closer match to the selector's ideal. That could be experience gained through projects or vacation work, or it might be that they were better at Page 14 of 18


communicating what they could offer. Do not take the comments at face value: think back to the interviews that generated them and make a list of where you think the shortfall in your performance lies. With this sort of analytical approach you will eventually get your foot in the door. Consultant C Deciding how long you should stay in your first job is a tough call. Stay too long and future employers may question your drive and ambition. Of course, it depends where you are aiming. There can be advantages in moving sideways rather than up, if you want to gain real depth of knowledge. If you are a graduate, spending five or six years in the same job is not too long provided that you take full advantage of the experience. However, do not use this as an excuse for apathy. Graduates sometimes fail to take ownership of their careers and take the initiative. It is up to you to make the most of what's available within a company, and to monitor your progress in case you need to move on. This applies particularly if you are still not sure where your career path lies. Consultant D It is helpful to think through what kind of experience you need to get your dream job and it is not a problem to move around to a certain extent. But in the early stages of your career you need a definite strategy for reaching your goal, so think about that carefully before deciding to move on from your first job. You must cultivate patience to master any role. There is no guarantee that you will get adequate training, and research has shown that if you do not receive proper help in a new role, it can take 18 months to master it. Consultant E A prospective employer does not want to see that you have changed jobs every six months with no thread running between them. You need to be able to demonstrate the quality of your experience to a future employer, and too many moves too quickly can be a bad thing. In any company it takes three to six months for a new employee to get up to speed with the structure and the culture of the company. From the company's perspective, they will not receive any return on the investment in your salary until you have been there for 18 months. This is when they begin to get most value from you - you are still fired up and enthusiastic. If you leave after six months it has not been a good investment - and may make other employers wary. (Source: CAE Handbook. Reproduced with permission from Cambridge English)

For questions 1–10, identify which section A–E each of the following is mentioned. Write ONE letter A–E in the space provided. Each letter may be used more than once. According to the text, which section(s) mention the following?

Your answers

Keep your final objective in mind when you are planning to change jobs Page 15 of 18

1.


It takes time to become familiar with the characteristics of a company you have

2.

joined. You should demonstrate determination to improve your job prospects.

3.

Make sure your approach for information is positive in tone.

4.

It is not certain that you will be given very much support in your job initially.

5.

Stay optimistic in spite of setbacks.

6.

Promotion isn't the only way to increase your expertise.

7.

Ask for information about your shortcomings.

8.

Some information you are given may not give a complete picture.

9.

It will be some time before you start giving your employers their money's worth.

10.

D. WRITING (60 pts) Part :For questions 1-3, complete the second sentence so that it has a similar meaning to the first sentence, using the word given. Do not change the word given. You must use between three and eight words, including the word given. There is an example at the beginning (0).(5pts) 0.‘Why don’t we have a picnic this weekend?’ said Andy

(HAVING)

Andy suggested ………having a picnic that……….. weekend. 1.If something’s worrying you, you should tell me about it now.

(CHEST)

-> If you have a problem, why don’t you _____________________________________ now? 2. I think you should confront your boss right now with how you feel about it. (OUT) -> Why don’t you _____________________________________ your boss right now? 3. Anna had decided Tom was not telling the truth about what happened. (TEETH) -> Anna was sure Tom _____________________________________ about what happened. For questions 4 – 5, use the word(s) given in brackets and make any necessary additions to write a new sentence in such a way that it is as similar as possible in meaning to the original sentence. Do NOT change the form of the given word(s). 4. When they broke the news, she stayed perfectly calm and controlled. (HAIR) _____________________________________________________________. 5. She wasn’t speaking seriously. (TONGUE) _____________________________________________________________.

Part 2. Chart description.(20pts) The table below shows the weight of people in a particular country from 1999 to 2009. Summarize the information by selecting and reporting the main features, and make comparisons where relevant. Write at least 150 words. Page 16 of 18


…………………………………………………………………………………………………………………………………………………………… ……………………………………………………………………………………………………………………………………………………………… ……………………………………………………………………………………………………………………………………………………………… ……………………………………………………………………………………………………………………………………………………………… ……………………………………………………………………………………………………………………………………………………………… ……………………………………………………………………………………………………………………………………………………………… ……………………………………………………………………………………………………………………………………………………………… ……………………………………………………………………………………………………………………………………………………………… ……………………………………………………………………………………………………………………………………………………………… ……………………………………………………………………………………………………………………………………………………………… ……………………………………………………………………………………………………………………………………………………………… ……………………………………………………………………………………………………………………………………………………………… ……………………………………………………………………………………………………………………………………………………………… ……………………………………………………………………………………………………………………………………………………………… ……………………………………………………………………………………………………………………………………………………………… ……………………………………………………………………………………………………………………………………………………………… ……………………………………………………………………………………………………………………………………………………………… ……………………………………………………………………………………………………………………………………………………………… ……………………………………………………………………………………………………………………………………………………………… ……………………………………………………………………………………………………………………………………………………………… ………………………………………………………………………………………………………………………………………………………………

Part 3: Write an essay of about 350 words to express your opinion on the following issue (35 pts) Discipline is an ever-increasing problem in many schools in Vietnam. Some people think that discipline should be the responsibility of teachers, while others think that this is the role of parents. Discuss both sides and give our opinion. Page 17 of 18


Give reasons for your answer, and include any relevant examples from your knowledge or experience. You may continue your writing on the back page if you need more space ……………………………………………………………………………………………………………………………………………………………… ……………………………………………………………………………………………………………………………………………………………… ……………………………………………………………………………………………………………………………………………………………… ……………………………………………………………………………………………………………………………………………………………… ……………………………………………………………………………………………………………………………………………………………… ……………………………………………………………………………………………………………………………………………………………… ……………………………………………………………………………………………………………………………………………………………… ……………………………………………………………………………………………………………………………………………………………… ……………………………………………………………………………………………………………………………………………………………… ……………………………………………………………………………………………………………………………………………………………… ……………………………………………………………………………………………………………………………………………………………… ……………………………………………………………………………………………………………………………………………………………… ……………………………………………………………………………………………………………………………………………………………… ……………………………………………………………………………………………………………………………………………………………… ……………………………………………………………………………………………………………………………………………………………… ……………………………………………………………………………………………………………………………………………………………… ……………………………………………………………………………………………………………………………………………………………… ……………………………………………………………………………………………………………………………………………………………… ……………………………………………………………………………………………………………………………………………………………… ……………………………………………………………………………………………………………………………………………………………… ……………………………………………………………………………………………………………………………………………………………… ……………………………………………………………………………………………………………………………………………………………… ……………………………………………………………………………………………………………………………………………………………… ……………………………………………………………………………………………………………………………………………………………… ……………………………………………………………………………………………………………………………………………………………… ……………………………………………………………………………………………………………………………………………………………… ……………………………………………………………………………………………………………………………………………………………… ……………………………………………………………………………………………………………………………………………………………… ……………………………………………………………………………………………………………………………………………………………… ……………………………………………………………………………………………………………………………………………………………… ……………………………………………………………………………………………………………………………………………………………… ………………………………………………………………………………………………………………………………………………………………

-The end-

Page 18 of 18


KỲ THI HỌC SINH GIỎI CÁC TRƯỜNG THPT CHUYÊN KHU VỰC DUYÊN HẢI VÀ ĐỒNG BẰNG BẮC BỘ LẦN THỨ XII, NĂM 2019 ĐÁP ÁN MÔN: TIẾNG ANH LỚP:11

ĐÁP ÁN (Đáp án gồm 11 trang) A. LISTENING (50 pts) Part 1: Choose the correct answer (A, B, C or D) which fits best according to what you hear. (10pts) 1. D

2. B

3. C

4. B

5. C

Part 2: You will hear a radio interview, decide whether the following sentences are true (T) or false (F). (10 pts) 1. T

2.F

3.F

4.T

5.F

Part 3: You will hear a guide speaking to tourists who are visiting some Romans remains. Listen and give short answers to the questions. Write NO MORE THAN THREE WORDS AND/OR A NUMBER taken from the recording. (10 pts) 1& 2 in either oder, 4&5 in either order 1. language 2. customs 3. local history (groups) 4. (the) (public) library/libraries 5. (the) town hall Part 4: Listen to a Ted talk named “Try something new for 30 days” and complete the summary below. WRITE NO MORE THAN THREE WORDS OR A NUMBER. (20pts) 1. great American philosopher

6. badly enough

2. substract a habit

7. sleep-deprived

3. much more memorable

8. small, sustainable changes

4. self confidence grew

9. big, crazy challenges

5. adventurous

10. likely to stick

1


B. LEXICO AND GRAMMAR (30 pts) Part 1.Choose the word/ phrase that best completes each of the following sentences. Write your answer in the corresponding numbered boxes. (10 pts) 1. C

2.D

3. C

4. C

5. A

6.A

7. B

8. D

9. C

10. A

Part 2. The text below contains 5 mistakes. Underline the mistakes and write the corrections in the space provided. (5 pts). There is an example at the beginning (0) No

Line

Mistake

Correction

1

3

valueless

invaluable/ valuable

2

8

clever

cleverly

3

10

Followed

Following

4

14

capital

capita

5

20

in

on

Part 3: Complete each of the following sentences with a suitable preposition or particle. Write your answer in the corresponding numbered boxes. (5 pts) 1. to

2. up

3. to

4. down

5. into

Part 4: Write the correct form of each bracketed word in the following sentences. (10 pts) 1.

2. disproved

3. undeniably

4. insatiable

unprecendent 6.

5.

numberless/

innumerable 7. dispossessed

8. injustice

9. incapable

10. invariably

unauthorized C. READING (60pts) Part 1: For questions 1–10, read the following passages and decide which answer (A, B, C or D) best fits each gap. Write your answers (A, B, C, or D) in corresponding numbered boxes. -(10 pts) 1.B

2.B

3.C

4.A

5.C

6.D

7.C

8.A

9.D

10.A

Part 2. Read the text below and think of one word, which best fits each space. Use only ONE WORD for each space. Write your answers in the corresponding numbered boxes. (10 pts) 1. diet

2. resemblance

3. into

4. dozen

5. who

6. less

7. conveniences

8. positives

9. to

10. worth

2


Part 3. Read the following passage and choose the best answer (A, B, C or D) according to the text. Write your answers (A, B, C or D) in the corresponding numbered boxes (15pts). 1.B

2.D

3.A

4.C

5.A

6.D

7.C

8.B

9.B

10.C

Part 4. Read the following passage and do the tasks that follows. Write your answer in the space provided. (15 pts)

1. vi

2. v

6.production- 7.unwillingness

3.iii

4.i

8. outsourcing

9. assembly line 10. training

line

5.viii

tasks

techniques Part 5. Read the text below and answer questions (10pts). 1.D

2.E

3.C

4.A

5.D

6.B

7.C

8.A

D. WRITING (60 points) Part 1.Sentene transformation (5pts) 1. ………get it of your chest…… 2. ……have it out with….. 3. ….. was lying through his teeth……. 4. When they broke the news, she didn’t turn a hair. 5. She was speaking with her tongue in her cheek/ with tongue in cheek. Part 2: Graph description (20 pts) 1. Completion: 2 pts 2. Content: 7 pts - Cover the main information in the chart yet not go into too many details. - Make general remarks and effective comparisons. 3. Organisation: 3 pts - The ideas are well organized - The description is sensibly divided into paragraphs 4. Language: 7 pts - Use a wide range of vocabulary and structure - Good grammar 5. Punctuation and spelling: 1 pt

3

9.B

10.E


Part 3: Essay writing (35 points) 1. Completion: (3 pts.) 2. Content: (10 pts) Provide relevant and convincing ideas about the topic, supported by specific example and/or reasonable justification. 3. Organization: (10 pts) - Ideas are well organized and presented with unity, cohesion and coherence. 4. Language: (10 pts) - Demonstrate of a wide range of vocabulary and structures. - Good use of grammatical structures. - Present the ideas with clarity. 5. Punctuations and spelling. (2 pts) - The end-

TAPESCRIPT:

4


PART 1: Exam narrator You will hear a group of art history students going round an art gallery with their teacher. For questions 1–5, choose the answer (A, B, C, or D) which best fits according to what you hear. Teacher Now, let’s move along to the next gallery … whose turn is it to tell us about the next painting? Amanda, is it you? Amanda Yes, this is the one I’ve prepared. Teacher Good … now I’ve got one or two questions for Amanda to guide us through this painting, so if you could all pay attention, we can get started … Brian … thank you. Now, as you can see it’s a pre-Raphaelite painting, so we’re talking 1880, 1890 … and what can you tell us about this – and other pre-Raphaelite paintings for that matter – compared to what came before? Amanda Well, there was very definitely a reaction against some of the earlier concerns – for example the preRaphaelites didn’t believe in the idea that it was important to be true to nature or realistic … This is a good example – it’s by the painter Burne-Jones, completed in 1884, and it shows a lot about his philosophy of painting … Teacher OK. And what was it exactly? Amanda Well in his own words, … is it OK if I use my notes? Teacher Yes of course. Amanda He said that a painting should be ‘a beautiful romantic dream of something that never was, never will be, in a land that no-one can define or remember, only desire.’ Teacher So in other words the very opposite of realism – no practical lessons for modern industrial societies or whatever. Amanda Yes, exactly, and this painting is in many ways very typical of Burne-Jones – in fact his wife later said it was his most distinctive work, the one that really summed up what he thought. Teacher OK, tell us about the story it tells. Amanda It’s called King Cophetua and the Beggar Maid, and it’s based on an old legend from early medieval times about a king who falls in love with a beggar girl, and finds that his love for her is greater than all his wealth and power. Teacher Was it a well-known story? Amanda Yes – most people knew it well, but only through reading Tennyson’s poetry, in which he wrote about it, rather than from the original story. Teacher So it’s another example of what we were talking about earlier – the link between the romantic movement in literature and the movements in art … do go on. Amanda In the painting, the artist imagines the King sitting at the girl’s feet, gazing at her in adoration. BurneJones said he was determined that the King should look like a king and the beggar should look like a Queen, and

5


he had certain details such as the crown and the maid’s dress specially made for him so that he could capture the detail. The setting has echoes of 15th century Italian art, particularly Mantegna and Crivelli, and it’s all elaborately decorated with highly wrought textures and jewel-like colours. If you look at the clothing you can see what I mean. The two characters in the background have got these rich flowing clothes, and there’s the same richness in the King’s flowing cloak. Teacher So what is he trying to tell us about here … what about these anemones … do they have any particular significance do you think? Amanda Yes, the maid is holding a bunch of anemones, and if you look closely you can see that some of them have fallen on the steps by the King. The flowers are a symbol of unrequited love, and there’s a lot of personal feeling in this painting, as there is in much of his work. At the time he was doing this, Burne-Jones had met and fallen in love with a girl called Frances Graham, but she then married someone else. So it’s likely that the King represents BurneJones and the Queen represents Frances Graham, and the painting shows his feelings about losing the woman he loved. Teacher Are there any other themes that the audience in 1884 would have recognized apart from on this personal level? Amanda Yes, to the general public it would have had a completely different meaning, which they would have recognized quite easily – they would interpret the painting as being about the rejection of worldly wealth and the elevation of love above everything else. Teacher Yes, absolutely … and that was a message that was very close to Burne-Jones’s heart and was very relevant for late Victorian Britain … Well thank you Amanda, and now we’ll move on to the next artist … Part 2: (CNN)

6


7


8


Part 3: You will hear an extract from a talk given to a group who are going to stay in the UK Listen and give short answers to the questions. Good evening, and welcome to the British Council. My name is John Parker and I’ve been asked to talk to you briefly about certain aspects of life in the UK before you actually go there. So I'm going to talk first about the best ways of making social contacts there. Now you might be wondering why it should be necessary. After all, we meet people all the time. But when you’re living in a foreign country it can be more difficult, not just because of the language, but because customs may be different. If you’re going to work in the UK you will probably be living in private accommodation, so it won’t be quite so easy to meet people. But there are still things that you can do to help yourself. First of all, you can get involved in activities in your local community, join a group of some kind. For example, you’ll probably find that there are theatre groups who might be looking for actors, set designers and so on, or if you play an instrument you could join music groups in your area. Or if you like the idea of finding out about local history there’ll be a group for that too. These are just examples. And the best places to get information about things like this are either the town hall or the public library. Libraries in the UK perform quite a broad range of functions nowadays – they’re not just confined to lending books, although that’s their main role of course.

9


Part 4: Ted talks : Try something new for 30 days Link: https://www.ted.com/talks/matt_cutts_try_something_new_for_30_days/transcript?referrer=pla ylist-ted_in_3_minutes#t-25670 A few years ago, I felt like I was stuck in a rut, so I decided to follow in the footsteps of the great American philosopher, Morgan Spurlock, and try something new for 30 days. The idea is actually pretty simple. Think about something you've always wanted to add to your life and try it for the next 30 days. It turns out 30 days is just about the right amount of time to add a new habit or subtract a habit -- like watching the news -- from your life. 00:43 There's a few things I learned while doing these 30-day challenges. The first was, instead of the months flying by, forgotten, the time was much more memorable. This was part of a challenge I did to take a picture every day for a month. And I remember exactly where I was and what I was doing that day. I also noticed that as I started to do more and harder 30-day challenges, my self-confidence grew. I went from desk-dwelling computer nerd to the kind of guy who bikes to work. For fun! 01:15 (Laughter) 01:17 Even last year, I ended up hiking up Mt. Kilimanjaro, the highest mountain in Africa. I would never have been that adventurous before I started my 30-day challenges. 01:28 I also figured out that if you really want something badly enough, you can do anything for 30 days. Have you ever wanted to write a novel? Every November, tens of thousands of people try to write their own 50,000-word novel, from scratch, in 30 days. It turns out, all you have to do is write 1,667 words a day for a month. So I did. By the way, the secret is not to go to sleep until you've written your words for the day. You might be sleep-deprived, but you'll finish your novel. Now is my book the next great American novel? No. I wrote it in a month. It's awful. 02:12 (Laughter) 02:14 But for the rest of my life, if I meet John Hodgman at a TED party, I don't have to say, "I'm a computer scientist." No, no, if I want to, I can say, "I'm a novelist." 02:26 (Laughter) 02:29 So here's one last thing I'd like to mention. I learned that when I made small, sustainable changes, things I could keep doing, they were more likely to stick. There's nothing wrong with big, crazy challenges. In fact, they're a ton of fun. But they're less likely to stick. When I gave up sugar for 30 days, day 31 looked like this. 02:51 (Laughter)

10


02:54 So here's my question to you: What are you waiting for? I guarantee you the next 30 days are going to pass whether you like it or not, so why not think about something you have always wanted to try and give it a shot! For the next 30 days.

- The end-

11


HỘI CÁC TRƯỜNG CHUYÊN

KỲ THI HỌC SINH GIỎI CÁC TRƯỜNG THPT CHUYÊN

KHU VỰC DH & ĐBBB

KHU VỰC DUYÊN HẢI VÀ ĐỒNG BẰNG BẮC BỘ

TRƯỜNG THPT CHUYÊN THÁI NGUYÊN

LẦN THỨ XII, NĂM 2019

ĐỀ ĐỀ XUẤT

ĐỀ THI MÔN: TIẾNG ANH 11

(Đề thi gồm 16 trang)

Thời gian: 180 phút (Không kể thời gian giao đề) Ngày thi: 15/4/2017 (Thí sinh viết câu trả lời vào bảng cho sẵn trong đề)

Điểm Bằng số

Bằng chữ

Giám khảo 1

Giám khảo 2

Số phách

A. LISTENING (50pts) Part 1. You will hear a radio interview with a road safety expert on the topic of road rage. For questions 1-5, choose the answer (A, B, C or D) which fits best according to what you hear. (10pts) 1. James says that driver become angry if: A. they think they will be delayed. B. other drivers threaten them. C. other people don’t drive as well as they do. D. they lose control of their car.

2. Revenge rage can lead motorists to A. chase after dangerous drivers. B. become distracted whilst driving. C. deliberately damage another car. D. take unnecessary risks.

3. James say that passengers become angry when buses are A. slow B. expensive C. crowded D. uncomfortable 4. According to James, what does the experiment with grass show? 1


A. People living in country areas are better drivers. B. Strong smells help us drive more safely. C. Our surroundings can affect the way we drive. D. Regular breaks on a journey keep drivers calm.

5. James thinks the hi-tech car A. sounds less irritating than a passenger. B. is not very reliable. C. could cause further danger. D. would be difficult to control. Your answers 1.

2.

3.

4.

5.

Part 2: You will hear the historian, George Davies, talking about society and the theatre in England in the time of William Shakespeare. Decide whether the following statements are true (T) or false (F). Write your answers in the corresponding numbered boxes. (10pts) 6. According to Professor Davies, the level of literacy in sixteen-century England matched his expectations. 7. In Professor Davies’ opinion, the advantage of the usual method of communication in the sixteenth century was that people absorbed more of what they heard. 8. Professor Davies believes that Shakespeare’s company developed their basic acting skills by attending special voice classes. 9. In Professor Davies’ view, the advantage of sixteen-century theatres was that the performances were complemented by everyday life. 10. Professor Davies thinks that sixteen-century plays were expected to deal with personal confessions. Your answers 6.

7.

8.

9.

10.

Part 3. (10 pts) You will hear a talk about an investigation into obesity. For questions 11-15, listen and answer the following questions with NO MORE THAN FIVE WORDS. Write your answer in the space provided. 11. What are less common within the family? ____________________________________ 12. What can make people eat more than their need? ____________________________________ 13. What kind of emotions can affect people’s eating habits? ____________________________________ 14. Who are more likely to eat more due to negative emotions? 2


____________________________________ 15. What will the researchers continue to investigate in the coming week? ____________________________________

Part 4: Listen to six pieces of BBC News. For questions 16–25, use NO MORE THAN THREE WORDS tocomplete eachgap. Write your answers in the corresponding numbered space. The oil cartel OPEC’s decision to (16) ________________ has led to a period of abnormally low prices that has(17) ____________________ of manufacturing countries. Stephen O’Brien, a UN envoy, made a plea to help residents in the (18) ____________________. The US president elect undertook to leave his business to avoid any (19) ____________________. The plane crash might have been caused by a(n) (20) ____________________, not electricity blackout asreported, prior to the pilots’ (21) _____________________ with the air traffic control. Ukrainian missile tests by Russian (22) ________________ would be conducted as planned in the peninsula (23)____________________ two years ago. Researchers believe the possibility that many families (24) ____________________ the birth of girls, which may have (25) ________________________ of the gender gap in China. B. LEXICO-GRAMMAR (30 pts) Part 1: For questions 1- 10, choose the correct answer A, B, C, or D to each of the following questions. Write your answers in the corresponding numbered boxes provided. 1. He was absolutely _______ with anger when he found that I had scratched his car. A. burned

B. carmine

C. fickle

D. livid

2. I don’t think Paul will ever get married — he’s the stereotypical _______ bachelor. A. settled

B. confirmed

C. fixed

D. determined

3. I cannot think who had_______ the gaff, but it seems everyone knows that Nicoleand I are planning to get married. A. burst

B. blown

C. split

D. banged

4. As we were in an urgent need of syringes and other medical equipment,the aid organization promised to deliver them _______ the double. A. at

B. in

C. with

D. round

5. She was so ill that it was ________ whether she would live or not. A. win or lose

B. come and go C. touch and go D. on and off

6. “Have you got a copy of Gone with the Wind?” “You’re ________ luck. We’ve just one copy left” A. by

B. with

C. in

D. on

7. When I got stuck in the elevator, I was scared out of my ________. A. brains

B. head

C. wits

D. nerves

8. All traffic is being _________ because of the military parades. A. diverted

B. converted

C. changed

D.altered 3


9. As he was running for a charity which was _____ to his heart, he felt even moredetermined to complete the race. A. warm

B. near

C. next

D. close

10. On completing her fifth Atlantic race, she decided to _____ herself a new challenge. A. set

B. make

C. fix

D. accept

Your Answers 1.

2.

3.

4.

5.

6.

7.

8.

9.

10.

Part 2. The passage below contains 10 mistakes. Underline the mistakes and correct them in the space provided in the column on right. (1) has been done as an example. (10 pts) Example: 1 2 3 4 5 6 7 8 9 10 11 12

1. Use Using

Vietnamese generally shake hands when getting and parting. Use both hands shows respect as a slightly bow of the head. In rural areas, elderly people who do not extend their hand are greeted with bow. Women are more certainly to bow than the head than to shake hands. Vietnamese names begin with the family game and are chased by a given name. People address one another by their given names but add a title that indicates their received relationship on the other person. These titles are family relate rather than professional. Among colleagues, for example, the young of the two might combine the given name with the given name and title is Xin chao (hello). Classifiers for gender and familiarity are also combined with the greeting. In formal meetings, business cards are sometime changed on greeting. Vietnamese people have a strong sense of hospitality and feel embarrassing if they cannot show their guests full respect by preparing for their arrival. Therefore, it is appropriate to visit someone without having been invited. Gifts are not required, so are appreciated. Flowers, incense, or teat may be proper gifts for the hosts. Hosts also appreciate a small gift for their children or elderly parents. Your answers Line

Mistake

Correction

Line

Mistake

Correction

Part 3. Fill in the gaps of the following sentences with suitable particles or prepositions.Write the answers in the correspondent numbered boxes. (10pts) There is an example at the beginning (0). Example: 0. up with 0. He finds it hard to put _________ the noise of the nearby factory. 1. The station is _____________ walking distance of the hotel. 4


2. I’m so tired after work that I often drop _____________ in front of the TV. 3. The decision was deferred _____________ a later meeting. 4. That awful new office block is a real blot _____________ the landscape. 5. I don’t know how Nicole survives, living all away _____________ the sticks. 6. Carl and I just spent the whole day lazing _____________ listening to music. 7. Unfortunately, most of the photos Terry took were ______________ focus. 8. Why don’t we meet _____________ here again on the 12th, when I get back fromGermany? 9. The prime minister has come _____________ fire during this election campaign forbeing slow to respond to events. 10. Keep plodding _____________ and you’ll finish your novel eventually.

Your answers: 1.

2.

3.

4.

5.

6.

7.

8.

9.

10. Part 4:Write the correct form of each bracketed word. Slave Narratives In the wake of the bloody Nat Turner (1. rebel)__________ in Southampton County, Virginia, in 1831, an increasingly fervent (2. slave)__________ movement in the United States sponsored (3. hand)__________autobiographical accounts of slavery by fugitives from the South in order to make (4. abolish)__________of a largely (5. difference)__________ white Northern readership. From 1830 to the end of the slavery era, the fugitive slave narrative dominated the (6. literature)__________ landscape of antebellum black America. The Narrative of the Life of Frederick Douglass, an American Slave, Written by Himself (1845) gained the most attention, (7. establishment)__________ Frederick Douglass as the leading African American man of letters of his time. By predicating his struggle for freedom on his (8. sole)__________ pursuit of literacy, education, and (9. depend)__________, Douglass portrayed himself as a (10. make)__________ man, which appealed strongly to middle-class white Americans. Your answers: 1.

2.

3.

4.

5.

6.

7.

8.

9.

10.

C. READING COMPREHENSION (60 pts) Part 1. From the words listed below, choose the one which best fits the space, A, B, C or D. LONDON’S BLACK CABS 5


Black cabs, officially known as Hackney Carriages, are (1) _______ London and are special for a number of reasons. For a start, they are the only taxis in the city that can be hailed from the kerb with a raised hand signal to get the driver’s attention. Currently, it is estimated that there are 20,000 black cabs (2) _______ on the capital’s streets. Their origin, in fact, can be (3) _______ the name ‘Hackney Carriage’ said to derive from the French word haquenée referring to the type of horse used to pull the carriages in the days of horse-drawn carriages. The first horse-drawn Hackney coaches appeared on London’s streets in the 17th century during the reign of Queen Elizabeth I. As transport developed and motor cars were (4) _______, motor cabs replaced the horse-drawn carriages. Since the end of the 19th century, various car manufacturers’ vehicles have been used as motor cabs but it was not until the mid20th century that the cabs we have been (5) _______ over the last decades first appeared. It is such a(n) (6) _______ of becoming a black cab driver in London and it is (7) _______. If you want to gain this honour you will need to have passed the infamous test known as ‘the Knowledge’, which was first introduced in 1851 following (8) _______ of complaints by passengers whose cab drivers got lost. This incredibly difficult test can take around three or four years to prepare for and you can often catch a glimpse of those drivers who are doing just this zipping around London on their mopeds, with a map (9) _______ to a clipboard on their handlebars. These people are essentially trying not only to master the 25,000 or so streets within a six-mile radius of Charing Cross, but also to work out the most direct routes from place to place. They must know thousands of ‘points of interest’ such as hotels, hospitals, places of worship, theatres, stations, sports and leisure facilities, to name but a few. Practically everywhere and anywhere that a potential passenger would wish to be taken to or from must be known, so a nodding acquaintance, for a black cab driver, is (10) _______, and perhaps this is the most difficult part, knowing the quickest way to get from one place to another. Little wonder so few people are successful. 1. A. commensurate with

B. equivalent to

C. synonymous with

D. tantamount to

2. A. hereabouts

B. hither and thither

C. or thereabouts

D. there and then

3. A. ferreted out from

B. hunted down from C. mapped out to

D. traced back to

4. A. all the rage

B. of high standing

C. of repute

D. in vogue

5. A. clued in on

B. gunned up on

C. in the know about

D. no stranger to

6. A. handiwork

B. procurement

C. realisation

D. undertaking

7. A. beyond you

B. no brainer

C. no mean feat

D. over your head

8. A. droves

B. hordes

C. packs

D. swarms

9. A. chained

B. fastened

C. linked

D. sealed

10. A. beyond measure

B. beyond redemption C. beyond the pale

D. beyond the veil

Your answers: 1.

2.

3.

4.

5.

6.

7.

8.

9.

10.

Part 2.Read the text below and think of the word which best fits each gap. Use only oneword in each gap. Write your answer in correspondent numbered boxes. (10pts) Despite the continued resilience of those early town perks, it wasn’t until the Depressionthat modern Hershey started to take (1) …………. Perhaps the only town in the countryactually to prosper during the 1930s, it thrived because Hershey vowed his Utopia wouldnever be on the (2) …………... Instead he funded a massive building boom that gave (3)…………. to the most visited buildings in today’s Hershey and delivered wages to morethan 600 workers. He admitted that his (4) …………. were partly selfish: 6


“If I don’tprovide work for them, I’ll have to feed them. And since building materials are now attheir lowest cost levels, I’m going to build and give them jobs.” He seems to have sparedno (5) ………….; most of the new buildings were strikingly opulent. The first to befinished was the three-million-dollar limestone Community Centre, home to the 1,904-seat Venetian-style Hershey Community Theatre, which has played (6) …………. since1933 to touring Broadway shows and to music, dance, and opera performances. It offersjust as much to look at when the lights are on and the curtains closed. The floors in theaptly (7) …………. Grand Lobby are polished Italian lava rock, surrounded by marblewalls and capped with a bas-relief ceiling showing (8) …………. of wheat, beehives,swans, and scenes from Roman mythology. With dazzling inner foyer, Hershey (9)…………. his nose even harder at the ravages of the Depression: The arched ceiling istiled in gold, the fire curtain bears a painting of Venice, and the ceiling is studded with 88tiny lightbulbs to re-create a star-(10) …………. night. Your answers: 1.

2.

3.

4.

5.

6.

7.

8.

9.

10. Part 3: Read the following passage and choose the best answer (A, B, C or D)according to the text. Write your answers (A, B, C or D) in the correspondingnumbered boxes (15pts). 1 2 3 4 5 6 7 8 9 10

Until recently, most American entrepreneurs were men. Discrimination against women inbusiness, the demands of caring for families, and lack of business training had kept thenumber of women entrepreneurs small. Now, however, businesses owned by womenaccount for more than $40 billion in annual revenues, and this figure is likely to continuerising throughout the 1990s. As Carolyn Doppelt Gray, an official of the Small BusinessAdministration, has noted, “The 1970s was the decade of women entering management,and the 1980s turned out to be the decade of the woman entrepreneur”. What are some ofthe factors behind this trend? For one thing, as more women earn advanced degrees inbusiness and enter the corporate world, they are finding obstacles. Women are stillexcluded from most executive suites. Charlotte Taylor, a management consultant, hadnoted, “In the 1970s women believed if they got an MBA and worked hard, they couldbecome chairman of the board. Now they’ve found out that isn’t going to happen, so theygo out on their own”.

11 12 13 14 15 16 17 18

In the past, most women entrepreneurs worked in “women’s” fields: cosmetics andclothing, for example. But this is changing. Consider ASK Computer Systems, a $22-million-a-year computer software business. It was founded in 1973 by Sandra Kurtzig,who was then a housewife with degrees in math and engineering. When Kurtzig foundedthe business, her first product was software that let weekly newspapers keep tabs ontheir newspaper carriers-and her office was a bedroom at home, with a shoebox underthe bed to hold the company’s cash. After she succeeded with the newspaper softwaresystem, she hired several bright computerscience graduates to develop additionalprograms. When these were marketed and sold, ASK began to grow. It now has 200employees, and Sandra Kurtzig owns $66.9 million of stock.

19 20 21 22

Of course, many women who start their own businesses fail, just as men often do. Theystill face hurdles in the business world, especially problems in raising money; thebanking and finance world is still dominated by men, and old attitudes die hard. Mostbusinesses owned by women are still quite small. But the situation is changing; there arelikely to be many more Sandra Kurtzigs in the years ahead.

1. What is the main idea of this passage? A. Women today are better educated than in the past, making them more attractive to the business world. B. The computer is especially lucrative for women today. C. Women are better at small businesses than men are. D. Women today are opening more businesses of their own. 7


2. The word “excluded” in line 8 (Women are still excluded from most executive suites) is closest in meaning to A. not permitted in B. often invited to C. decorators of D. charged admission to 3. All of the following were mentioned in the passage as detriments to women in thebusiness world EXCEPT A. women were required to stay at home with their families B. women lacked ability to work in business C. women faced discrimination in business D. women were not trained in business 4. In line 10, “that”(Now they’ve found out that isn’t going to happen)refers to A. a woman becomes chairman of the board B. women working hard C. women achieving advanced degrees D. women believing that business is a place for them 5. According to the passage, Charlotte Taylor believes that women in the 1970s A. were unrealistic about their opportunities in business management B. were still more interested in education than business opportunities C. had fewer obstacles in business than they do today D. were unable to work hard enough to succeed in business 6. The author mentions the “shoebox under the bed” in the third paragraph in order to A. show the frugality of women in business B. show the resourcefulness of Sandra Kurtzig C. point out that initially the financial resources of Sandra Kurtzig’s business werelimited D. suggest that the company needed to expand 7. The expression “keep tabs on” in line 15 is closest in meaning to A. recognize the appearance of B. keep records of C. provide transportation for D. pay the salaries of 8


8. The word “hurdles” in line 20 can be best replaced by A. fences B. obstacles C. questions D. small groups 9. It can be inferred from the passage that the author believes that businesses operated by women are small because A. women prefer a small intimate setting B. women can’t deal with money C. women are not able to borrow money easily D. many women fail at large businesses 10. The author’s attitude about the future of women in business is A. sceptical B. optimistic C. frustrated D. negative Your answers: 1.

2.

3.

4.

5.

6.

7.

8.

9.

10.

Part 4. Read the following passage and choose the most suitable from A to G on the list and write it in each gap from 1 to 5 and from the gap 6 to 10 complete the notes. Write your answers in the correspondent numbered boxes. (20 pts) DOES GLOBALIZATION HELP THE POOR? A. In recent years, we have heard steady proclamations emanating from the advocates of economic globalization and leaders of the world’s leading financial institutions – the World Bank, the International Monetary Fund (IMF), the World Trade Organization (WTO), etc. - that the ultimate purpose in pushing economic globalization is to help the world’s poor. More specifically, they contend that removing barriers to corporate trade and financial investments is the best path to growth, which they claim offers the best chance of rescuing the poor from poverty. They also assert that the millions of people who oppose the economic globalization model are harming the interests of the poor. They should back off and leave it to corporations, bankers and global bureaucracies to do the planning and solve the world’s problems. Such claims are routinely replayed in the media. One prominent national columnist writes, ‘Protesters are choking the only route out of poverty for the world’s poor.’ In other words, if the protesting stopped, the financial institutions would save the day. Is this believable? Is it the salvation of the poor that really drives global corporations or are their primary motives quite different? B. Almost all the evidence from the most robust period of economic globalization – 1970 to the end of the 20th century — shows that its outcome is the exact opposite of what its supporters claim. Interestingly, this evidence now comes as much from the proponents of globalization as from its 9


opponents. Clearly, poverty and inequality are rapidly accelerating everywhere on Earth. A 1999 report by the United Nations Development Program found that inequalities between rich and poor within and among countries are quickly expanding, and that the global trading and finance system is a primary cause. Even the US Central Intelligence Agency (CIA) confirms the United Nations’ (UN) conclusions, agreeing that globalization brings massive inequalities. “The benefits of globalization do not reach the poor,” says the CIA, ‘and the process inevitably results in increased unrest and protest.’ C. The ideologies and rules of economic globalization - including free trade, deregulation and privatization - have destroyed the livelihoods of millions of people, often leaving them homeless, landless and hungry, while removing their access to the most basic public services like health and medical care, education, sanitation, fresh water and public transport.The records show that economic globalization makes things worse for the poor, not better. Economic globalization has actually succeeded in making global corporations and a few elites wildly wealthy. Of the largest 100 economies m the world, 52 are corporations. This is what the UN describes as the ‘staggering concentration of wealth among the ultra-wealthy’. D. Contrary to claims, wealth generated by globalization does not trickle down. Rather, the wealth is locked at the top, removing from governments and communities the very tools necessary to redistribute it, and in doing so protect domestic industries, social services, the environment and sustainable livelihoods. There may be isolated instances where temporary improvement has been achieved in Third World countries, and, of course, the financial institutions love to trumpet these. The truth, however, is that benefit has been very short-lived and the majority of it has gone to the elites in these countries and to the chief executives of the global corporations at the hub of the process. E. People may point to the ‘Asian Tiger’ economies like Taiwan, South Korea and Singapore as examples of the success of globalization, but the truth is that improvement has not been achieved by assiduously adhering to the dictates of international financial advisors. These economies initially resisted the prescribed economic model and managed to stay free of the volatility of export markets. When they did finally succumb to pressure from the IMF and the World Bank, they found their glory days quickly disappearing into the infamous Asian financial crisis of the late 90s. F. The majority of poor countries have not enjoyed much benefit from globalization, and a growing number of people understand that the system is selling a false promise. The policies of the financial institutions are not designed to benefit them, but to benefit rich industrial countries and their global corporations. The question then is: “Do these globalizing institutions know what they’re doing or do they just mindlessly follow a failed ideological model?”. Many commentators fiercely opposed to globalization now firmlybelieve that the institutions do indeed know exactly what they’re doing and that theyalways have. They have an assignment to remove all obstacles to the free flow of capitalas they seek to pry open the world’s last natural resource pools, markets and cheaplabour.To suggest they do all this to help the poor is high cynicism. G. Perhaps the most traumatic impact of globalization has resulted from local economiesbeing forced to shift from a small-scale diversified agricultural model towards theindustrial export model. Half the world’s population still lives directly on the land,growing food for their communities.They grow staples and a mix of diverse crops, andthey replant with indigenous seed varieties developed over centuries. They have perfectedtheir own fertilization and pesticide management. Such systems have sustained hundredsof millions of people for millennia.Global corporations must resist local selfsufficiency. Profit is generated byincreased processing activity and global trading. We have seen companies spend millionsof dollars on publicity professing that small farmers are not productive enough to feed thehungry world. This publicity runs in tandem with the investment and trade strategies ofthe same companies - strategies that aim to replace local, diverse farming for selfreliancewith monocultures. The people who once grew their crops are driven off their land.People who once fed themselves become landless, homeless, jobless and hungry.Dependency and starvation replace self-sufficient livelihoods and self-reliant nations,while global corporations maintain their wealth by shipping luxury items thousands ofmiles to already overfed markets. Clearly, these 10


corporations are not concerned aboutfeeding the hungry. They are concerned about feeding themselves. The passage has seven sections labelled, A-G. For questions 1-5, choose the correctheading for each section from the list of headings in the box. You do not need to use allthe headings. I. Initial gains - ultimate disaster II. A stark contrast between the poor and the super-rich. III. The obliteration of traditional practices IV. The poor must take some responsibility V. Boasts about small victories are misplaced VI. We know best. Don’t try to stop us. VII. Markets yet to be exploited VIII. Incompetence or a well-planned strategy? IX Parties on both sides can see the short-comings X. Asia leads the way Example: Paragraph B: IX Paragraph G: III 1. Paragraph A : 2. Paragraph C : 3. Paragraph D : 4. Paragraph E : 5. Paragraph F :

For questions 6-10, complete the notes. Use NO MORE THAN TWO WORDS for eachanswer. Globalization – two sides of the coin Advocates say: Growth realized only by (6) …………… to business Hope of salvation thwarted by (7) …………… Opponents say: (8) …………… of the system equally sceptical Millions destitute without essential (9) …………… (10) …………… now wealthier than some countries Your answers: 1.

2.

3.

4.

5.

6.

7.

8.

9. 11


10. Part 5: Answer questions 1-10, by referring to the magazine article in which four successful career women talk about emigrating to New Zealand. A) Nicky Meiring

B) Jenny Orr

C) Sarah Hodgetts

D) Lucy Kramer

Which woman... 1. mentions a negative point about a job she has had? 2. explains an advantage of choosing to pursue her career in New Zealand? 3. appreciates the approach to achieving goals in New Zealand? 4. expresses a sense of regret about leaving her country? 5. appreciates the honesty she feels exists in New Zealand? 6. denies conforming to a certain stereotype? 7. appreciates New Zealand for its sense of calm and normality? 8. mentions her move to a different area in the same field? 9. states that her original nationality puts her in an advantageous position? 10. recommends that New Zealanders take more pride in their country?

The Brain Gain With New Zealand becoming renowned as a great place to live, it was the first-choice destination for a new generation of talented migrants looking for a better life. Sharon Stephenson talks to four of them. A) Nicky Meiring, Architect Listen to Nicky Meiring talk about South Africa and it soon becomes evident that she’s mourning for a country she once called home. ‘The current economic situation has made South Africa quite a hard place to live in,’ she says, ‘but I do miss it.’ Nicky first arrived in Auckland in 1994 and got a job in an architectural practice in Auckland where she soon settled in. She says ‘New Zealand often feels like utopia. I just love the tranquillity and the fact you can lead a safe and ordinary life.’ She lives and works from a renovated factory where her mantelpiece is littered with awards for the design of her summer house on Great Barrier Island. ‘Although the design of buildings is fairly universal, houses here are generally constructed of timber as opposed to brick and when it comes to the engineering of buildings, I have to take great heed of earthquakes which isn’t an issue in South Africa,’ she says. “But the very fact that my training and points of reference are different means I have something to offer. And I’m so glad I have the opportunity to leave my stamp on my new country.” B) Jenny Orr, Art Director American Jenny Orr’s southern accent seems more at home in the movies than in New Zealand’s capital, Wellington. ‘I’m from Alabama, but no, we didn’t run around barefoot and my father didn’t play the banjo!’ she jokes, in anticipation of my preconceptions. Having worked in corporate design for ten years in the USA, she was after a change and thought of relocating to New Zealand. It didn’t take long for her to land a job with an Auckland design firm, where she was able to gain experience in an unfamiliar but challenging area of design -packaging -and before long, she was headhunted to a 12


direct marketing agency which recently transferred her to Wellington. While she admits she could have the same salary and level of responsibility at home, ‘it would probably have been harder to break into this kind of field. I’m not saying I couldn’t have done it, but it may have taken longer in the US because of the sheer number of people paying their dues ahead of me.’ Ask Jenny how she’s contributing to this country’s ‘brain gain’ and she laughs. ‘I don’t see myself as being more talented or intelligent but opposing views are what make strategies, concepts and designs better and I hope that’s what I bring.’ C) Sarah Hodgetts, Creative Planner What happens when all your dreams come true? Just ask Sarah Hodgetts. Sarah says that she had always dreamed of a career in advertising. ‘But I was from the wrong class and went to the wrong university. In the UK, if you’re working class you grow up not expecting greatness in your life. You resign yourself to working at the local factory and knowing your place.’ New Zealand, on the other hand, allowed her to break free of those shackles. ‘It’s a land of opportunity. I quickly learned that if you want to do something here, you just go for it, which is an attitude I admire beyond belief.’ Within a month of arriving, she’d landed a job in customer servicing with an advertising agency. Then, when an opening in research came up, she jumped at the chance. ‘My job is to conduct research with New Zealanders,’ she explains. ‘So, I get to meet people from across the social spectrum which is incredibly rewarding.’ Being a foreigner certainly works in her favour, says Sarah. ‘Because a lot of my research is quite personal, respondents tend to see me as’ impartial and openminded and are therefore more willing to share their lives with me.’ She certainly sees New Zealand in a good light. ‘I wish New Zealanders could see their country as I do. That’s why it saddens me that they don’t think they’re good enough on the global stage.’ D) Lucy Kramer, School Director Born in Sydney, Australia, Lucy Kramer left for London when she was 23 to further her career as a stockbroker. ‘London certainly lived up to my expectations and I had a very exciting, very hectic lifestyle,’ Lucy explains. But after four years she felt burnt out and was becoming increasingly disillusioned with her job. ‘People at work were far too competitive for my liking,’ she says. It was at this time she made two life-changing decisions. ‘I signed up for a teacher- training course and shortly after that met my partner, Graeme. He asked me to come back to New Zealand with him and I didn’t hesitate.’ It wasn’t long before she found work in a large Auckland school and, since then, she has rapidly worked her way up to a management position. ‘It’s fair to say I’m not earning what I used to but my New Zealand colleagues are much more easy- going. A good atmosphere more than makes up for the drop-in salary. Another thing that impresses me is that you can leave your stuff on a seat in a cafe and it’ll still be there half an hour later. People are pretty trustworthy here. Sometimes it bothers me that we’re so remote -you can feel a bit cut off from what’s going on in the rest of the world, but on the whole, I’d say it’s one of the best moves I ever made.’ Your answers 1.

2.

3.

4.

5.

6.

7.

8.

9.

10.

D. WRITING (60 pts) Part 1.Read the following passage and use your own words to summarise it. Your summary should beabout 140 words long. (10pts) A patent is an exclusive right to use an invention for a certain period of time, which is given to an inventor as compensation for disclosure of an invention. Although it would be beneficial for the world economy to have uniform patent laws, each country has its own laws designed to protect domestic inventions and safeguard technology. Despite widespread variation, patent laws generally fall under one of two principles: the first-to-file and first-to invent. The first-to-file principle awards 13


a patent to the person or institution that applies for a patent first, while the first-to invent principle grants the patent to the person or institution that was first to invent – and can prove it. Most countries have adopted the first-to-file system. However, the United States maintains a first-to-invent system, despite obvious shortcomings. A result of countries employing different patent law principles is inconsistency of patent ownership. Patent ownership is not recognized globally. On the contrary, ownership may change depending on the country. It is not uncommon for an invention to have two patent owners – one in the United States and one in the rest of the world. This unclear ownership often has economic consequences. If a company is interested in using a patented invention, it may be unable to receive permission from both patent owners, which in turn may prevent manufacture of a particular product. Even if permission is received from both owners, pay royalties to both may be quite costly. In this case, if the invention is useful enough, a company may proceed and pass on the added cost to consumers. International economic tension has also been increasing as a result of differing policies. Many foreign individuals and companies believe that they are at a serious disadvantage in the United States with regard to patent ownership because of the logistical difficulties in establishing first-to invent status. Further, failure of the United States to recognize patent ownership in other countries is in violation of the Paris Conventions on Industrial Properties, which requires all member nations to treat all patents equally. The conflict surrounding patents has prompted the World Intellectual Properties Organization (WIPO) to lobby for universality in patent laws. WIPO maintains that the first necessary step involves compelling the United States to re-examine its patent principle, taking into account the reality of a global economy. This push may indeed result in more global economic cooperation. ………………………………………………………………………………………………………… ………………………………………………………………………………………………………… ………………………………………………………………………………………………………… ………………………………………………………………………………………………………… ………………………………………………………………………………………………………… ………………………………………………………………………………………………………… ………………………………………………………………………………………………………… ………………………………………………………………………………………………………… ………………………………………………………………………………………………………… ………………………………………………………………………………………………………… ………………………………………………………………………………………………………… ………………………………………………………………………………………………………… ………………………………………………………………………………………………………… ………………………………………………………………………………………………………… ………………………………………………………………………………………………………… ………………………………………………………………………………………………………… ………………………………………………………………………………………………………… ………………………………………………………………………………………………………… …………………………………………………………………………………………………………

14


Part 2:The chart below shows how frequently people in the USA ate in fast food restaurants between 2003 and 2013.

Frequency of eating at fast food restaurants among people in the USA (2003-2013) 35 30

% of people

25 20 15 10 5 0

Everyday

Several times a week

Once a week

2003

2006

Once or twice a A few times a year month

Never

2013

………………………………………………………………………………………………………… ………………………………………………………………………………………………………… ………………………………………………………………………………………………………… ………………………………………………………………………………………………………… ………………………………………………………………………………………………………… ………………………………………………………………………………………………………… ………………………………………………………………………………………………………… ………………………………………………………………………………………………………… ………………………………………………………………………………………………………… ………………………………………………………………………………………………………… ………………………………………………………………………………………………………… ………………………………………………………………………………………………………… ………………………………………………………………………………………………………… ………………………………………………………………………………………………………… ………………………………………………………………………………………………………… …………………………………………………………………………………………………………

…………………………………………………………………………………………………………


………………………………………………………………………………………………………… …………………………………………………………………………………………………………

Part 3:Essay writing ( around 250 words) The future of work in the age of industrial and technological revolution, as seen by many people, is seemingly grim for the labour force, as more and more jobs are being replaced by autonomous machines that are enormously capable and productive. In your opinion, how can workers adapt to this change? What is needed of a student as a futureworking adult? ………………………………………………………………………………………………………… ………………………………………………………………………………………………………… ………………………………………………………………………………………………………… ………………………………………………………………………………………………………… ………………………………………………………………………………………………………… ………………………………………………………………………………………………………… ………………………………………………………………………………………………………… ………………………………………………………………………………………………………… ………………………………………………………………………………………………………… ………………………………………………………………………………………………………… ………………………………………………………………………………………………………… ………………………………………………………………………………………………………… ………………………………………………………………………………………………………… ………………………………………………………………………………………………………… ………………………………………………………………………………………………………… ………………………………………………………………………………………………………… ………………………………………………………………………………………………………… ………………………………………………………………………………………………………… ………………………………………………………………………………………………………… ………………………………………………………………………………………………………… ………………………………………………………………………………………………………… ………………………………………………………………………………………………………… ………………………………………………………………………………………………………… ………………………………………………………………………………………………………… ………………………………………………………………………………………………………… ………………………………………………………………………………………………………… ………………………………………………………………………………………………………… ………………………………………………………………………………………………………… 16


………………………………………………………………………………………………………… ………………………………………………………………………………………………………… ………………………………………………………………………………………………………… THE END

17


KỲ THI HỌC SINH GIỎI CÁC TRƯỜNG THPT CHUYÊN

HỘI CÁC TRƯỜNG CHUYÊN KHU VỰC DH & ĐBBB TRƯỜNG THPT CHUYÊN THÁI

KHU VỰC DUYÊN HẢI VÀ ĐỒNG BẰNG BẮC BỘ

NGUYÊN

LẦN THỨ XII, NĂM 2019 ĐỀ THI MÔN: TIẾNG ANH 11

HƯỚNG DẪN CHẤM

Thời gian: 180 phút (Không kể thời gian giao đề) Ngày thi: 15/4/2017 (Thí sinh viết câu trả lời vào bảng cho sẵn trong đề) A. LISTENING (50pts) Part 1: You will hear a radio interview with a road safety expert on the topic of road rage. For questions 1-5, choose the answer (A, B, C or D) which fits best according to what you hear. (10pts) 1. A

2. B

3. A

4. C

5. C

Part 2: You will hear the historian, George Davies, talking about society and the theatre in England in the time of William Shakespeare. Decide whether the following statements are true (T) or false Write your answers in the corresponding numbered boxes. (10pts) 6. T

7. T

8. F

9. T

10. F

Part 3 (10 pts) 2 pts/correct answer. 11. formal eating times 12. emotional state 13. negative, (and) positive emotions 14. people who are overweight 15. serious eating disorders Part 4: 16. cut production 17. strained finances 18. besieged rebel-held neighbourhoods 19. conflict of interest 20. low on fuel 21. losing contact 22. air defence forces 23. annexed 1


24. concealed 25. distorted official figures B. LEXICO-GRAMMAR (30 pts) Part 1: For questions 1- 10, choose the correct answer A, B, C, or D to each of the following questions. Write your answers in the corresponding numbered boxes provided. 1.D 2.B 3.B 4.A 5. C. touch and go 6. C. in 7. C 8. A 9. D 10. A Part 2. The passage below contains 10 mistakes. Underline the mistakes and correct them in the space provided in the column on right. (1) has been done as an example. (10 pts)

Your answers Line

Mistake

Correction

Line

Mistake

Correction

2

Slightly

slight

7

Young

Younger

3

Certainly

likely

9

Changed

exchanged

4

Chased

followed

10

Embarrassing embarrassed

5

On

to

11

Appropriate

inappropriate

6

Relate

related

12

So

but

Part 3. Fill in the gaps of the following sentences with suitable particles or prepositions. Write the answers in the correspondent numbered boxes. (10pts) 0. up with 1. within 2. off 3. to 4. on 5. in 6. around 7. out of 8. up 9. under 10. away

2


Part 4: Write the correct form of each bracketed word. 1. rebellion

4. abolitionists 7. establishing

2. antislavery

5. indifferent

8. solitary

3. first-hand

6. literary

9. self-made

10. independence

C. READING COMPREHENSION (60 pts) Part 1. From the words listed below, choose the one which best fits the space, A, B, C or D. 1. B

2. D

3. C

4. C

5. D

6. B

7. C

8. D

9. B

10. A

Part 2. Read the text below and think of the word which best fits each gap. Use only one word in each gap. Write your answer in correspondent numbered boxes. (10pts) 1. shape

2. breadline

3. rise

4. intentions/intents/aims/purposes/objectives/goals/plans/schemes/aspirations/ambitions/desires 5. expense

6. host

7. named

8. sheaves

9. thumbed

10. studded/lit

Part 3: Read the following passage and choose the best answer (A, B, C or D) according to the text. Write your answers (A, B, C or D) in the corresponding numbered boxes (15pts). 1. D

2. A

3. B

4. A

5. A

6. C

7. B

8. B

9. C

10. B

Part 4. Read the following passage and choose the most suitable from A to G on the listand write it in each gap from 1 to 5 and from the gap 6 to 10 complete the notes. Write your answers in the correspondent numbered boxes. (10 pts) 1. VI

2. II

3. V

4. I

6.removingbarriers

7. protesters

9. publicservices

10.Corporations

5. VII 8. proponents/advocates

Part 5: Answer questions 1-10, by referring to the magazine article in which four successful career women talk about emigrating to New Zealand. 1. D

2. B

3. C

4. A

5. D

6. B

7. A

8. B

9. C

10. C 3


D. WRITING (60 pts) Part 1: Summary In his paper “Global Implications of Patent Law Variation,” Koji Suzuki (1991) states that lack of consistency in the world’s patent laws is a serious problem. In most of the world, patent ownership is given to the inventor that is first to file for a patent. However, the United States maintains a first-to-invent policy. In view of this, patent ownership can change depending on the country. Multiple patent ownership can result in economic problems; however, most striking is the international tension it causes. The fact that the United States does not recognize patent ownership in other countries, in violation of the Paris Convention on Industrial Properties, has prompted the World Intellectual Properties Organization (WIPO) to push the United States to review its existing patent law principles.

Part 2:The chart below shows how frequently people in the USA ate in fast food restaurants between 2003 and 2013.

Frequency of eating at fast food restaurants among people in the USA (2003-2013) 35 30

% of people

25 20 15 10 5 0

Everyday

Several times a week

Once a week

2003

2006

Once or twice a A few times a year month

Never

2013

Part 2: Chart The bar chart illustrates the frequency with which Americans ate in fast food establishments from 2003 to 2013. It is clear that the majority of Americans ate in fast food restaurants between once a week and once a month in all three years. We can also see a shift towards eating in these restaurants less frequently by the end of the 10-year period. In each of the years shown on the chart, close to 60% of people in the US ate in fast food restaurants between once and four times a month. Roughly 15 to 20% of Americans reported 4


eating in fast food outlets several times per week, while 3 to 4% of people ate in these outlets daily. At the other end of the scale, around 4% of people avoided fast food restaurants completely. Between 2006 and 2013, the total proportion of Americans who ate in fast food establishments either once a week or several times a week fell by almost 10%. At the same time, there was an increase of around 8% in the ‘once or twice a month’ category. In other words, the weekly fast food habit that was common in 2003 and 2006 became a monthly or twice monthly habit in 2013.

Part 3:Essay writing Marking guide -

-

-

This is an open question, as opposed to people’s first impression. Therefore, any answer should be appreciated and not written off just because it doesn’t answer the question in the title. Look for fallacies in students’ answer. This is an argumentative essay. Abide by the standard IELTS marking guide. Any idea that falls into the following suggested ones should be given full mark: 1. Against the “new technology will replace human”: • Automation has happened in the past (steam machines…) and many jobs were lost, but we didn’t run out of work to do. • For every new technology, there are jobs created directly for people who design and maintain the technology, and sometimes a whole new industry is built on this technology. • But we tend to forget the indirect effect of the labour-saving inventions. When companies can do more with less, they can expand or even add new products or open new locations, and they can lower prices to compete -> consumers buy more, or they can use the savings to buy other things (more haircut, more of other products, more travelling…) And this is how we have improved our standard of living since the beginning of time, and it always require new workers. 2. For the “technology will replace human” • It is easy to underestimate technology these days (AI has beaten human in chess, Go, and live TV shows, voice recognition…) • And since automation can replace human workers en masse, it is easy to employ 100% automation to do much more efficiently. • Human workers’ productivity across the board for nations around the world has been stagnant, and seems to be slowing down. • Unemployment has been on the rise around the world. If new jobs are being created then why don’t we see unemployment going down? Any idea that falls outside the ones above should be marked in accordance, again, with the IELTS standard.

5


TRƯỜNG THPT CHUYÊN

ĐỀ ĐỀ XUẤT THI DHBB - NĂM HỌC 2018-2019

VĨNH PHÚC

MÔN TIẾNG ANH 11 Thời gian làm bài: 180 phút;

I.

LISTENING

PART 1: You will hear two people speaking about their fondness for trains. Each given question has four options A, B, C and D. Choose the best option for each question. You will hear the audio twice. 1) What point does Philip make about the people who are involved in the locomotive side of trains? A) Some avoid doing it because its too dirty. B) Some worry about the dangers involved. C) Some find it to be an instinctive activity. D) Some only do it for the financial rewards it brings. 2) When Mike discusses why he enjoys the world of trains, what contrast does he identify between the different groups of people he interacts with? A) A difference in social status. B) A difference in ages. C) A difference in technical ability. D) A difference in motivation. 3) What view is stated by Mike about how the railway has helped him in his role as a parent? A) It has helped him develop a closer relationship with his daughter. B) It has taught him patience. C) It has helped him focus on his daughters career possibilities. D) It has given him stability in his life. 4) When discussing different aspects of the railway, both speakers agree that A) Age does not need to be a handicap. B) Both men and women are equally welcome. 1


C) It can be quite demanding. D) Cleanliness is not the first adjective that springs to mind. 5) What final conclusion do both speakers reach about volunteering on the railway? A) The time they have spent has been difficult but worth it. B) They have both learnt a lot. C) They are better people for having been involved with it. D) It has helped them grow while at the same time giving them a direction for the future. PART 2: You will hear a lecture about study. Decide whether the statements are TRUE or FALSE.You will hear the audio twice. 1. In the past, time management meant you needed to set goals and try to achieve these goals. 2. In this college, students are assigned team projectsat the end of each semester. 3. The kind of suggestion the lecturer gives to the students is a regular one-hour session in their personal timetables. 4. If you want to set an overview of your time, you should need at least half a week. 5. According to the lecturer, the benefit that wise time management may have is having more time to spend on relaxation and other activities. PART 3: You will hear Tim Cole talking about guidebooks. Answer the following questions with short answers of NO MORE THAN FIVE WORDS.You will hear the audio twice. 1. What place did Tim want to find that turned out to be a comedy club when he used a guidebook in Australia? _________________________________________ 2. When must the publication date be for a guidebook to be bought? _________________________________________ 3. What does Tim particularly dislike about guidebooks? _________________________________________ 2


4. What are other things that should be included in guidebooks? _________________________________________ 5. What way of searching travel advice does Tim suggest to Hawaii visitors? _________________________________________ PART 4: You will hear a piece of news about Hurricane Florence ready to Hit MidAtlantic as a Category 4 hurricane. Fill in each blank with NO MORE THAN THREE WORDS from the recording.You will hear the audio twice.

1. It is advised to get away from the coast as ____________________, Hurricane Florence, brews in the Atlantic Ocean. 2. ____________________for more than 1 million people in the states of North Carolina, South Carolina and Virginia have already been triggered. 3. The wind speeds hovered around 140 miles per hour Tuesday, which makes Florence a ____________________. 4. The area it hits could be ____________________ for months. 5. Its catastrophic storm surge is the ____________________in sea levels as a hurricane blows water ashore. 6. Anything more than a 12 foot storm surge is life threatening according to the ____________________. 7. It is predicted to slow down after making ____________________ by scientists tracking the storm. 8. This hurricane`s wind speeds ____________________ Tuesday. 9. There is that outside shot of this ____________________further up as it approaches land. 10. That’s what really becomes a danger in scenario here with potential rainfall amounts as much as 20 or more inches on some of these ____________________. 3


II.

LEXICO-GRAMMAR

Exercise 1: Choose the best option to complete the sentences. 1. The spacecraft _________ into space and then spent three days getting to the Moon. a. hurled

b. threw

c. blasted

d. orbited

2. Did you ever realise you have an uncanny _________ of coming down to the kitchen just as I am about to serve dinner? Very strange! a. habit

b. knack

c. trick

d. routine

3. __________ on by my husband, I applied for the job I had always dreamt of having. a. begged

b. obliged

c. urged

d. pleaded

4. __________ by both financial and emotional problems, the father decided he had had enough and slipped away from the house quietly at dawn. a. pressured

b. beset

c. filled

d. drowned

5. I felt let down by a very unsupportive boardroom, ___________ by the very chairman himself, Wilson Gray. I had no choice but to resign. a. at most

b. not least

c. notwithstanding d. henceforth

6. Out upon the cloud covered hills, we only caught the occasional glimpse of the town lights ___________ far below in the valley. a. sparkling

b. glimmering

c. flashing

d. beaming

7. After a month, I will ___________ the ropes and won't keep bothering you for help. a. show

b. learn

c. get

d. finish

8. The young rookie scored over 20 goals in his first year, taking the whole league by __________. a. force

b. example

c. storm

d. assault

9. Hubert ___________ remembered locking the door and couldn't understand how it was now standing wide open. a. distinctly

b. sharply

c. totally

d. utterly

4


10. If you are looking for Sheila, she is over in the corner of the library with her head ___________ in her books. a. covered

b. entombed

c. buried

d. drowned

Exercise 2: There are FIVE mistakes in the passage. Identify them and correct them as in the example provided. Where do emotions come from? For many centuries, the question of how our minds work has been left to theologians and philosophers. But at the beginning of the twentieth century, a new science, experimental psychologically emerged, in which the speculative theories of the past were confirmed or disproved by the scientific method. At the forefront of this research was J. B. Watson. His area of interest was the origin of human emotions. Do we learn them, or do we have them when we are born? In particular, Watson wanted to study fear, and was prepared to go to however lengths to study his theory. Watson’s subject was a 9-month-old infant, Albert. During the experiment, Watson presented the child with things what are often considered frightening – a rat, fire, a clown mask. At first, Albert was unafraid of these things. But then Watson tormented the child with loud, expected noises as he was playing with them. Sure enough, Albert learnt to associate these things with the unpleasant experience. Even when the noises were stopped, Albert withdrew his body and puckered his face when presented once more with the rat and mask. Example: Line 1: has been => was Exercise 3: Fill in each blank with one most suitable preposition or particle. 1. The room fell silent when Johnson, unaware of the couple's history, brought___________ the subject of divorce. 2. I suppose I always felt a certain amount of jealousy towards Elvin who was always such a well thought___________ young man. 3. If they ask you about me, don't let___________ that you know where I am, OK? 5


4. I hope you won't feel too put ___________ if I turn up at the last minute. The hotel was fully booked! 5. A lesser know symptom of this condition is that you will suddenly have panic attacks and break ___________ cold sweats with little or no warning. Exercise 4: Put the words in capitals into the correct forms. Read the text below. Use the word given in capitals at the end of some of the lines to form a word that fits in the gap in the same line. Bristleworms - a hobbyist's guide Historically, Bristleworms have had a bad reputation among 1. WATER

aquarium

aficionados. These marine worms usually enter the hobbyist's aquarium by hitching a ride on a piece of coral. Once 2. ESTABLISHMENT , they become part of the tank's ecosystem. Bristleworms range greatly in size. The smallest ones are about an inch long, and the large ones can grow to over 20 inches, although, being 3. SEGMENTAL , their bodies are often retracted and so not usually seen at their greatest extent. Literature has frequently contented that bristleworms are harmful, 4. ASSERTIVE that they eat clams, anemones and even coral fish. However, most 5. ENTHUSIASM now conclude that small bristleworms pose no threat, and are merely 6. SCAVENGE , clearing the tank from detritus and carcasses of animals that are already dead. However, larger worms, particularly those of the species known as fireworms, are 7.VORACITY eaters and can do 8. REPAIR damage. These worms are better removed, although this is a challenge in itself, as the worms are 9. NOCTURNE , sensitive to light and will go into hiding at the slightest 10. DISTURB . III.

READING

Exercise 1: Choose the best option to complete each blank in the passage. The Vacuum Cleaner Until about 250 years ago, households did not take dirt as seriously as they do now - it was a fact of life, and that was that. Cleaning often consisted of an annual ..(1)... called 6


'spring cleaning' when the furniture was moved aside, and all the linen products in the house were cleaned. Carpets and rugs were taken outside, hung on ropes and had the dust ..(2).. out of them - an exhausting and messy process. The industrial revolution brought about a major change - as new products became available to make homes cleaner, a corresponding interest in 'domestic hygiene' appeared in households. This in turn led to the ..(3).. of further products, one of which was the vacuum cleaner. ..(4).. has it that when one of the first vacuum cleaners was demonstrated, a kindly scientist took the proud inventor..(5).. , and offered a bit of advice that was to become crucial to the future evolution of the product - 'make it suck, not blow'. The first vacuum cleaners appeared in the 1860s in the United States. They were operated by hand pumps and were almost as ..(6).. as spring cleaning. It was only when electric motors had become sufficiently ..(7)... to become portable that vacuum cleaners became common household items. Most of today's major ..(8).. - including Electrolux and Hoover - were born in the 1920s. The household ..(9).. that vacuum cleaners suck up is mostly dead skin cells - humans ..(10).. millions of cells every day. A much smaller proportion comes from dust and soil carried into the house from outside . 1. a. ritual

b. result

c. resolution

d. scrub

2. a. cleaned

b. taken

c. beaten

d. sucked

3. a. fabrication

b. appearing c. recreation

4. a. Story

b. Epic

c. Legend

d. Tale

5. a. away

b. aside

c. aback

d. along

6. a. laborious

b. hard

c. nefarious

d. straining

7. a. scientific

b. forward

c. technological

d. advanced

8. a. brands

b. marks

c. make

d. trademarks

9. a. grit

b. rubbish

c. refuse

d. dirt

10. a. lose

b. outgrow

c. omit

d. shed

d. development

7


Exercise 2: Complete the passage with ONE WORD for each blank. An Unlikely Muse A new wave of music and arts projects has emerged, focusing on someone who may seem for some a dubious (1)___________ of inspiration. Imelda Marcos, former (2)___________

lady of the Philippines, is currently becoming the subject of

musicals, song cycles and shows on a worldwide arena. When the Marcos regime collapsed in 1986, and Imelda and her husband Ferdinand were exiled in Hawaii, they carried with (3)___________

allegations of

embezzlement, corruption and human rights abuses. Imelda had spent the last twenty years living off a seemingly endless supply of funds, living an exotic and glamorous lifestyle and rubbing (4)___________ with powerful figures worldwide. In 1972, when the superstar couple’s popularity was fading and they were at risk of (5)___________ their power, Ferdinand Marcos instated martial, leading to an era of chaos and plunder, and (6)___________ is described by some as the second most corrupt regime of the twentieth century. Ferdinand and Imelda fled in 1986 to escape the People’s Power Revolution, Imelda leaving (7)___________ some 2000 pairs of shoes. After her husband died in Hawaii due to ill (8)___________ , Imelda stood trial in the United States on (9)___________ of her husband. Following that, she returned to the Philippines to face seventy more counts of corruption and tax (10)___________ . She has now returned to congress in the Philippines, her make-up and gowns as flawless as ever. Exercise 3: Read the text and choose the best options to answer the questions. This passage is from Charlotte Brontë, The Professor, originally published in 1857. No man likes to acknowledge that he has made a mistake in the choice of his profession, and every man, worthy of the name, will row long against wind and tide before he allows himself to cry out, “I am baffled!” and submits to be floated passively back to land. From the first week of my residence in X— felt my occupation irksome. 8


The thing itself—the work of copying and translating business-letters— was a dry and tedious task enough, but had that been all, I should long have borne with the nuisance; I am not of an impatient nature, and influenced by the double desire of getting my living and justifying to myself and others the resolution I had taken to become a tradesman, I should have endured in silence the rust and cramp of my best faculties; I should not have whispered, even inwardly, that Ilonged for liberty; I should have pent in every sigh bywhich my heart might have ventured to intimate itsdistress under the closeness, smoke, monotony, andjoyless tumult ofBigben Close, and its panting desirefor freer and fresher scenes; I should have set up theimage of Duty, the fetish of Perseverance, in mysmall bedroom at Mrs. King’s lodgings, and they twoshould have been my household gods, from which my darling, my cherished-in-secret, magination, thetender and the mighty, should never, either bysoftness or strength, have severed me. But this wasnot all; the antipathy which had sprung up betweenmyself and my employer striking deeper root andspreading denser shade daily, excluded me from every glimpse of the sunshine of life; and I began tofeel like a plant growing in humid darkness out of theslimy walls of a well. Antipathy is the only word which can express thefeeling Edward Crimsworth had for me—a feeling, ina great measure, involuntary, and which was liable tobe excited by every, the most trifling movement,look, or word of mine. My southern accent annoyedhim; the degree of education evinced in my languageirritated him; my punctuality, industry, andaccuracy, fixed his dislike, and gave it the highflavour and poignant relish of envy; he feared that Itoo should one day make a successful tradesman.Had I been in anything inferior to him, he would nothave hated me so thoroughly, but I knew all that heknew, and, what was worse, he suspected that I keptthe padlock of silence on mental wealth in which hewas no sharer. If he could have once placed me in aridiculous or mortifying position, he would haveforgiven me much, but I was guarded by threefaculties—Caution, Tact, Observation; and prowlingand prying as was Edward’s malignity, it could neverbaffle the lynx-eyes of 9


these, my natural sentinels.Day by day did his malice watch my tact, hoping itwould sleep, and prepared to steal snake-like on itsslumber; but tact, if it be genuine, never sleeps. I had received my first quarter’s wages, and wasreturning to my lodgings, possessed heart and soulwith the pleasant feeling that the master who hadpaid me grudged every penny of that hard‑earnedpittance—(I had long ceased to regardMr. Crimsworth as my brother—he was a hard,grinding master; he wished to be an inexorabletyrant: that was all). Thoughts, not varied but strong,occupied my mind; two voices spoke within me;again and again they uttered the same monotonousphrases. One said: “William, your life is intolerable.”The other: “What can you do to alter it?” I walkedfast, for it was a cold, frosty night in January; as Iapproached my lodgings, I turned from a generalview of my affairs to the particular speculation as towhether my fire would be out; looking towards thewindow of my sitting-room, I saw no cheering redgleam. 1. Which choice best summarizes the passage? A. A character describes his dislike for his new joband considers the reasons why. B. Two characters employed in the same officebecome increasingly competitive. C. A young man regrets privately a choice that hedefends publicly. D. A new employee experiences optimism, thenfrustration, and finally despair. 2. The main purpose of the opening sentence of thepassage is to __________ A. establish the narrator’s perspective on acontroversy. B. provide context useful in understanding thenarrator’s emotional state. C. offer a symbolic representation ofEdward Crimsworth’s plight. D. contrast the narrator’s good intentions with hismalicious conduct. 3. During the course of the first paragraph, thenarrator’s focus shifts from _________ A. recollection of past confidence toacknowledgment of present self-doubt. B. reflection on his expectations of life as atradesman to his desire for another job. C. generalization about job dissatisfaction to thespecifics of his own situation. 10


D. evaluation of factors making him unhappy toidentification of alternatives. 4. The references to “shade” and “darkness” at the endof the first paragraph mainly have which effect? A. They evoke the narrator’s sense of dismay. B. They reflect the narrator’s sinister thoughts. C. They capture the narrator’s fear of confinement. D. They reveal the narrator’s longing for rest. 5. The passage indicates that Edward Crimsworth’sbehavior was mainly caused by his __________ A. impatience with the narrator’s high spirits. B. scorn of the narrator’s humble background. C. indignation at the narrator’s rash actions. D. jealousy of the narrator’s apparent superiority. 6. The passage indicates that when the narrator beganworking for Edward Crimsworth, he viewedCrimsworth as a __________ A. harmless rival.

B. sympathetic ally.

C. perceptive judge.

D. demanding mentor.

7. Which choice provides the best evidence for theanswer to the previous question? A.Paragraph 1 (“the antipathy... life”) B.Paragraph 2 (“My southern... irritated him”) C.Paragraph 2 (“Day... slumber”) D.Paragraph 3 (“I had... brother”) 8. At the end of the second paragraph, the comparisonsof abstract qualities to a lynx and a snake mainlyhave the effect of __________ A. contrasting two hypothetical courses of action. B. conveying the ferocity of a resolution. C. suggesting the likelihood of an altercation. D. illustrating the nature of an adversarialrelationship. 11


9. The passage indicates that, after a long day ofwork, the narrator sometimes found his livingquarters to be __________ A. treacherous.

B. dreary.

C. predictable.

D. intolerable.

10. Which choice provides the best evidence for theanswer to the previous question? A.Paragraph 1 (“I should... scenes”) B.Paragraph 1 (“I should... lodgings”) C.Paragraph 3 (“Thoughts... phrases”) D.Paragraph 3 (“I walked... gleam”) Exercise 4: Read the text and answer the questions that follow. Question 1- 6 The reading passage has seven paragraphs, A-G. Choose the correct heading for paragraph A -G from the list below. There is one example that has been done for you. Write the correct number i-ix, in boxes 1- 6 on your answer sheet. List of Headings: i

Disobeying FAA Regulations

ii

Aviation disaster prompts action

iii Two coincidental developments iv Setting Altitude Zones v

An oversimplified view

vi Controlling pilots' licence vii Defining airspace categories viii Setting rules to weather conditions ix Taking of Safety x

First step towards ATC

Example Paragraph B

Answer X 12


1. Paragraph A 2. Paragraph C 3. Paragraph D 4. Paragraph E 5. Paragraph F 6. Paragraph G AIR TRAFFIC CONTROL IN THE USA A An accident that occurred in the skies over the Grand Canyon in 1956 resulted in the establishment of the Federal Aviation Administration (FAA) to regulate and oversee the operation of aircraft in the skies over the United States, which were becoming quite congested. The resulting structure of air traffic control has greatly increased the safety of flight in the United States, and similar air traffic control procedures are also in place over much of the rest of the world. B

Rudimentary air traffic control (АТС) existed well before the Grand Canyon

disaster. As early as the 1920s, the earliest air traffic controllers manually guided aircraft in the vicinity of the airports, using lights and flags, while beacons and flashing lights were placed along cross-country routes to establish the earliest airways. However, this purely visual system was useless in bad weather, and, by the 1930s, radio communication was coming into use for АТС. The first region to have something approximating today's АТС was New York City, with other major metropolitan areas following soon after. C In the 1940s, АТС centres could and did take advantage of the newly developed radar and improved radio communication brought about by the Second World War, but the system remained rudimentary. It was only after the creation of the FAA that full-scale regulation of America's airspace took place, and this was fortuitous, for the advent of the jet engine suddenly resulted in a large number of very fast planes, reducing pilots' margin of error and practically demanding some set of rules to keep everyone well separated and operating safely in the air. 13


D Many people think that АТС consists of a row of controllers sitting in front of their radar screens at the nation's airports, telling arriving and departing traffic what to do. This is a very incomplete part of the picture. The FAA realised that the airspace over the United States would at any time have many different kinds of planes, flying for many different purposes, in a variety of weather conditions, and the same kind of structure was needed to accommodate all of them. E To meet this challenge, the following elements were put into effect. First, АТС extends over virtually the entire United States. In general, from 365m above the ground and higher, the entire country is blanketed by controlled airspace. In certain areas, mainly near airports, controlled airspace extends down to 215m above the ground, and, in the immediate vicinity of an airport, all the way down to the surface. Controlled airspace is that airspace in which FAA regulations apply. Elsewhere, in uncontrolled airspace, pilots are bound by fewer regulations. In this way, the recreational pilot who simply wishes to go flying for a while without all the restrictions imposed by the FAA has only to stay in uncontrolled airspace, below 365m, while the pilot who does want the protection afforded by АТС can easily enter the controlled airspace. F The FAA then recognised two types of operating environments. In good meteorological conditions, flying would be permitted under Visual Flight Rules (VFR), which suggests a strong reliance on visual cues to maintain an acceptable level of safety. Poor visibility necessitated a set of Instrumental Flight Rules (IFR), under which the pilot relied on altitude and navigational information provided by the plane's instrument panel to fly safely. On a clear day, a pilot in controlled airspace can choose a VFR or IFR flight plan, and the FAA regulations were devised in a way which accommodates both VFR and IFR operations in the same airspace. However, a pilot can only choose to fly IFR if they possess an instrument rating which is above and beyond the basic pilot's license that must also be held. 14


G Controlled airspace is divided into several different types, designated by letters of the alphabet. Uncontrolled airspace is designated Class F, while controlled airspace below 5,490m above sea level and not in the vicinity of an airport is Class E. All airspace above 5,490m is designated Class A. The reason for the division of Class E and Class A airspace stems from the type of planes operating in them. Generally, Class E airspace is where one finds general aviation aircraft (few of which can climb above 5,490m anyway), and commercial turboprop aircraft. Above 5,490m is the realm of the heavy jets, since jet engines operate more efficiently at higher altitudes. The difference between Class E and A airspace is that in Class A, all operations are IFR, and pilots must be instrument-rated, that is, skilled and licensed in aircraft instrumentation. This is because АТС control of the entire space is essential. Three other types of airspace, Classes D, С and B, govern the vicinity of airports. These correspond roughly to small municipal, medium-sized metropolitan and major metropolitan airports respectively, and encompass an increasingly rigorous set of regulations. For example, all a VFR pilot has to do to enter Class С airspace is establish two-way radio contact with АТС. No explicit permission from АТС to enter is needed, although the pilot must continue to obey all regulations governing VFR flight. To enter Class В airspace, such as on approach to a major metropolitan airport, an explicit АТС clearance is required. The private pilot who cruises without permission into this airspace risks losing their license. Question 7-10 Do the following statements agrees with the given information of the reading passage? In boxes 7-10 on your answer sheet, write: TRUE if the statement agrees with the information FALSE if the statement contradicts the information NOT GIVEN if there is no information on this 7. The FAA was created as a result of the introduction of the jet engine. 8. Air traffic control started after the Grand Canyon crash in 19 56. 15


9. Beacons and flashing lights are still used by the ATC today. 10. Some improvements were made in radio communication during World War II.

Exercise 5: You are going to read an extract from an article about modern art and whether it can be called 'art'. For questions 1-10, choose from the people (A, B, C or D). The people may be chosen more than once. Is it Art? Corinne Art is the result of an artist using her or his skill or creative imagination for a creative purpose, to give pleasure to the viewer through its aesthetic qualities, or to get a reaction from the audience to a wider more significant issue outside of the work of art itself. That work of art might be a painting, a sculpture, an installation of some kind or an example from the performing arts like dance or mime. I think we sometimes get bogged down by the notion of 'skill'. For many in the anti modern art camp, there needs to be evidence of the artist's craft on show before the work is taken seriously and can merit the term 'art', be it intricate drawing skills, expert use of form or an artist's eye for colour. I'm not suggesting that an artist need not have these credentials but hand in hand with craft is, as I said earlier, creative imagination, the ability to see the value or beauty of something unremarkable which would often go unnoticed by the untrained eye. Much of modern art I think possesses this second quality which is why I often leave an exhibition of modern art feeling that I've had the chance to reflect on something that I wouldn't normally have given the time of day to. The art has engaged me, has had an impact, made me think about something in a way that I wouldn't have thought about before. Michael I would certainly call myself an art enthusiast and have been for many years and in my opinion the modern art world is full of second-rate junk which most of us, if we were being totally honest, would agree a 4-year-old child could do. The idea that a slept-in 16


bed such as that 'produced' by Tracy Emin or many of the pieces by Damien Hirst and his ilk are works of art is hard to justify as is the huge price tag that accompanies their work. I find it particularly galling when extremely talented people out there who have spent years honing their skills and learning the craft of drawing or painting are completely ignored. What's more, one of the dangers of this kind of 'art' is that it serves to alienate the mass of the population from the visual arts. The man in the street viewing one of these pieces is left thinking the world of modern art has no value; worse still, that he lacks the intellectual ability to understand the meaning of the piece when in fact there is little to interpret. Thankfully, one or two great artists make it through, but I'm afraid many are lost amongst the deluge of dross the art-world deems 'art'. For me, the first measure of the worth of an artist must be the degree of skill exhibited in the work or at the very least a pedigree of fine art preceeding any more abstract pieces produced by the artist such as was the case with Picasso. Robert The idea that modern art is some kind of mass deception and that all modern artists are talentless fraudsters just doesn't hold water. And I'm not talking here about the painters who for centuries have made a living out of copying works of art and selling them on as originals. I'm talking about abstract art and the idea that the great art collectors such as the Saatchis or Rockefellas and the great museums of art around the world, would somehow allow themselves to be duped into paying a fortune for an abstract painting or sculpture. Are these artists really tricking these people into paying huge sums of money for something worthless? Of course not. Though some of these works may not appear to the layman as having any artistic merit, neither did the great impressionists or the more abstract works of Picasso or Rothko when they were first exhibited. In the same way that great poetry can speak to us in a way that prose never can, abstract art can engage with the audience in more subtle and effective ways than is the case with art of a more realistic nature. So, they may get their fingers burnt now and again but I don't think the Saatchis will be cursing the day they spent huge sums on works of 17


abstract art. Quite the opposite in fact and in the process of making a canny investment they have helped further raise the profile of some of our great modern artists. Janet Here we go again: the media are once more up in arms about the latest 'is it art' shockhorror editorials following the latest Turner Prize shortlisting. When will they learn? For decades art in many forms has moved away from realism and towards abstraction. Ever since the invention and popularisation of photography, art has had to reinvent itself. Patrons who wanted a perfect representation of themselves no longer needed to turn to the artist. Artists started to struggle with the challenge of catching the essence of the thing depicted rather than simply its external appearance. Abstract artists try to convey a pure idea, not the exact replica of the subject concerned. It's true that some works of art are so obscure that you may need to read up on the theory behind the creation, which is usually helpfully supplied in art galleries. But this isn't always necessary. Take Guernica by Picasso. To get a full understanding of this painting it could be argued the audience needs to appreciate the historical context, the bombing of the Basque city during the Spanish Civil War. It would also probably help to have a good understanding of the techniques of abstraction that Picasso had used to create the effect. However, I think most people viewing this masterpiece would be struck by the horror it depicts even without this background knowledge. And I would argue it is the effect of this abstraction that adds to the impact on us compared to a realistic portrayal of such a scene. Which person gives each of these opinions about modern art? A. Corinne

B. Michael

C. Robert

D. Janet

1) Some practices have been going on for hundreds of years. 2) Some people may not have the knowledge to understand a work of art fully. 3) Certain aesthetic qualities can be invisible until brought to our attention by the artist. 4) Picasso is an example of an artist who proved his craftsmanship. 18


5) Appreciation of the work itself is not always the artist's aim. 6) The purpose of Art has undergone change. 7) People don't always appreciate the works of great artists initially. 8) Abstract art is generally overpriced. 9) Abstract art isn't always a good investment. 10) We can be touched by a work of art without knowing the context. IV.

WRITING

Exercise 1: Write a summary of the passage within 140 words. Super-recognisers 1

Hundreds of angry people took to the streets in London and at least ten other English

cities in 2011. Petrol bombs were thrown, buildings and vehicles destroyed and shops looted. Many of the rioters were caught on CCTV cameras, but most of the images were poor and the perpetrators had covered their faces. 2

Gary Collins, an off-duty policeman, was watching the London riots on TV. He

immediately recognised several people and cut his holiday short to help with identification. He ended up spending six months going through the CCTV film and managed to identify 190 people, many from their eyes alone. His help was of decisive importance in the investigation. Even with 200,000 hours of footage, facial recognition software managed to identify just one person. 3

Collins is no ordinary police officer. Soon after joining the police force, he realised he

had a special gift: after seeing a face briefly, he could remember it in detail years later. He is what is known as a super-recogniser. This term was first used in 2009 when a study estimated that 1–2 per cent of the population have severe problems recognising faces – called prosopagnosia or ‘face-blindness’ – and another 1–2 per cent are exceptionally good at it. They can recall up to 95 per cent of faces they see, whereas an average person remembers about 20 per cent. 4

London’s Metropolitan Police set up a unit of super-recognisers in 2015 after Collins’

success proved it could be useful. It is the first of its kind in the world. Detective Chief 19


Inspector Mick Neville of Scotland Yard, one of its founders, had realised that there was a problem with CCTV back in 2008. CCTV film was not used efficiently in the courts and did not seem to work as an effective deterrent. If criminals were captured on film, they knew they were unlikely to be recognised. 5

Neville’s unit has changed all that. Since it started, the tiny team of six officers has

made nearly a quarter of all identifications in London, mainly by spending hours scanning film and photos. This is impressive, considering that there are 32,000 police officers in the city. The team’s success relies on the ubiquity of CCTV; there are thought to be more than four million cameras throughout Britain. 6

This winning combination of human skill and technology has helped convict

criminals from shoplifters and pickpockets to sex offenders and murderers. Offenders are very seldom convicted solely on the evidence of a super-recogniser, but it is used to direct investigations. Many defendants plead guilty when they realise they have been caught red-handed.Super-recognisers could be useful in many jobs such as security and passport control. Exercise 2: The graph below shows the population figures for different types of turtles in India from 1980 to 2012. Summarize the information by selecting and reporting the main features, and make comparisons where relevant.

20


Exercise 3: In recent years, more and more people are choosing to read e-books rather than paper books. Do the advantages outweigh the disadvantages? Use your own knowledge and experience and support your arguments with examples and relevant evidence. Give reasons for your viewpoint.

21


KEY I. LISTENING PART 1: 1. C

2. D

3. A

4. D

5. D

2. FALSE

3. TRUE

4. FALSE

5. TRUE

PART 2: 1. FALSE PART 3: 1. A budget hotel. 2. Within the last twelve months. 3. The restaurant suggestions 4. Detailed maps (and/or background information) (of the area) 5. Twitter tourism PART 4: 1. a monster storm

6. National Hurricane Center.

2. Mandatory evacuation orders

7. landfall

3. Category 4 hurricane

8. fluctuated

4. uninhabitable

9. potentially even strengthening

5. abnormal rise

10. costal communities

II. LEXICO-GRAMMAR Exercise 1: (Total: 10pts, 1pt/each correct answer) 1. c

2. b

3. c

4. b

5. b

6. b

7. b

8. c

9. a

10. c

Exercise 2: (Total: 5pts, 0.5pt/each correct identification + 0.5pt/each correction) 1. Line 2: in => at 2. Line 3: psychologically =>psychology 3. Line 7: however => whatever 4. Line 9: what => which/that 5. Line 11: expected => unexpected 22


Exercise 3: (Total: 5pts, 1pt/each correct answer) 1. up

2. of

3. on

4. out

5. into

Exercise 4: (Total: 10pts, 1pt/each correct answer) 1. saltwater

6. scavengers

2. established

7. voracious

3. segmented

8. irreparable

4. asserting

9. nocturnal

5. enthusiasts

10. disturbance

III. READING Exercise 1: (Total: 10pts, 1pt/each correct answer) 1. a

2. c

3. d

4. c

5. b

6. a

7. d

8. a

9. d

10. d

Exercise 2: (Total: 15pts, 1.5pts/each correct answer) 1. source

6. what

2. first

7. behind

3. them

8. health

4. shoulders

9. behalf

5. losing

10. evasion

Exercise 3: (Total: 10pts, 1pt/each correct answer) 1. A 2. B 3. C 4. A 5. D 6. B 7. D 8. D 9. B 10. D Exercise 4: (Total: 10pts, 1pt/each correct answer) 1. ii 2. iii 3. v

4. iv 5. viii 6. vii

7. FALSE 8. FALSE 9. NOT GIVEN 10. TRUE Exercise 5: (Total: 15pts, 1.5pts/each correct answer) 1. C 2. D 3. A 4. B 5. A 6. D 7. C 8. B 9. C 10. D IV. WRITING Notes: 23


The mark given to parts 1, 2 and 3 is based on the following scheme: 1. Content: (35% of total mark) a. Providing all main ideas and details as required b. Communicating intentions sufficiently and effectively 2. Organization & Presentation: (30% of total mark) a. Ideas are well organized and presented with coherence, cohesion, and clarity b. The essay is well-structured 3. Language: (30% of total mark) a. Demonstration of a variety of vocabulary and structures appropriate to the level of English language gifted upper-secondary school students b. Good use and control of grammatical structures 4. Handwriting, punctuation, and spelling (5% of total mark) a. Intelligible handwriting b. Good punctuation and no spelling mistakes Markers should discuss the suggested answers and the marking scale thoroughly before marking the papers.

24


Section 3 Tim: I'm Tim Cole, and as an experienced travel writer, I'm here to tell you not to believe everything you read in guidebooks because following some of the recommendations they give can result in the most bizarre situations. I'll never forget the night I arrived in Sydney, for example. I'd turned up at the address of what I thought was a budget hotel given in the guidebook at 1 a.m., exhausted and looking forward to a few hours' rest, but instead found myself at a comedy club, which at the time I didn't find at all funny. The problem is that too many travellers are too trusting of their guidebooks and don't bother to research even the most basic facts before they set off. Some guidebooks are only updated every couple of years, so it's no wonder many things have moved on by the time you get there. The most important thing when choosing a guidebook is to check the publication date; if it's not within the last twelve months, don't buy it. Then the other thing to think about is who the guidebook is aimed at. If you're into the history and culture of a place, don't buy a guidebook full of information on the alternative nightlife scene. But my pet hate, and something I'm always extremely wary of, are the restaurant suggestions. So often I've turned up somewhere and the menu, price and décor bear no relation to the place I've been reading about – if they haven't already gone out of business and shut down, that is. Other things to look out for in a guidebook are the maps. These need to be detailed but not so small you can't read them. You don't want to have to carry a magnifying glass around with you. Books that include unnecessary information are another thing I find annoying – like photos of famous places, for example. We already know what the Eiffel tower looks like! Why not include more background information instead? Of course, most guidebooks are also now available in a digital format and many travellers prefer using these because they're obviously not heavy to carry, so you can download as many as you like. But I don't find them easy to use at all because navigation is much harder than flicking through the index at the back of a book. Life's 25


just too short and you can never guarantee you'll have wifi access anyway. Until I can get a digital travel guide which is tailor-made for my individual trip, I'm happy to stick with the traditional form of guidebook. However, on my trip to Hawaii last summer I experimented with a new way of getting good travel advice: Twitter tourism. Instead of using a guidebook, I decided to rely on the advice of locals and visitors alike – and let them choose what I should visit, where I should stay and what I should eat. I didn’t mind as long as their advice was based on a recent experience. It actually worked out really well and it felt like a real adventure. Without the Twitter travel tips I'd never have visited the Ukulele Festival or eaten spam sushi. One thing I'd never imagined doing – and I'm so grateful for the advice – was a ten-kilometre kayak expedition along the coast for a night time swim with manta rays in a huge cave. A truly magnificent sight. And my top tip for anyone visiting Hawaii! Section 4 Hurricane Florence ready to Hit Mid-Atlantic as a Category 4 hurricane; September 11th Memorial Services; Should Pluto be Reinstated as a Planet; Global Medical Relief Fund helping Children from Around the World with Prosthetics; Giant Rideable Mantis Aired September 12, 2018 - 04:00:00 ET THIS IS A RUSH TRANSCRIPT. THIS COPY MAY NOT BE IN ITS FINAL FORM AND MAY BE UPDATED. CARL AZUZ, CNN 10 ANCHOR: Thank you for downloading, streaming or just plain watching CNN 10. I`m Carl Azuz explaining world news from the CNN Center. First today, get away from the coast. That`s what a Americans in the U.S. Southeast are being told as a monster storm brews in the Atlantic Ocean. It`s name is Hurricane Florence. It`s already triggered mandatory evacuation orders for more than 1 million people in the states of North Carolina, South Carolina and Virginia. And Florence is 26


uniquely dangerous. For one thing it`s powerful. Its wind speeds hovered around 140 miles per hour Tuesday. That makes Florence a Category 4 hurricane. That makes Florence strong enough to blow the roofs off houses, knock down walls, snap most trees, take out power. The area it hits could be uninhabitable for months. For another it`s storm surge could be catastrophic. This is the abnormal rise in sea levels as a hurricane blows water ashore. A CNN meteorologist says Hurricane Florence could bring a 20 foot storm surge. That would make the tide 20 feet higher than it normally is as Florence blows in. The National Hurricane Center says anything more than a 12 foot storm surge is life threatening. Third, scientists tracking this storm predict it will slow down after it makes landfall. That`s a major problem as far as flooding is concerned. Last years Hurricane Harvey was a slow moving storm. It poured rain on Houston, Texas for more than a week and that caused scenes like this. Predicting exactly what storms like Florence will do is still like predicting the weather. There`s uncertainty about it. This hurricane`s wind speeds fluctuated Tuesday. Forecasters didn`t agree on whether it would still be a Category 4 storm at landfall, if it would get stronger or weaker before it blows ashore. But here`s how things looked yesterday afternoon. (BEGIN VIDEO CLIP) UNIDENTIFIED MALE: We now have Hurricane Watches along the coastal region of South Carolina all the way up to the border of Virginia and same goes for Storm Surge Watches that have been issued across the region but everything else remains the same. Still a Category 4, still 140 mile per hour sustained winds and the storm still pushes off to north and west northwest that are 15 miles per hour. But here we go with 27


this and we know water temperatures from here forward will just continue to get warmer. You need water temps of 82 degrees Fahrenheit to maintain a tropical system. We`ll go to about 85 eventually to about 88 before it makes landfall. That`s why there is that outside shot of this potentially even strengthening further up to a Category 5 as it approaches land. But, you take a look. That rapid intensification in place, 36 hour period from a Cat 1 to Category 4, remarkable system. And of course, we`ve looked very carefully at where it`s going to end up and consistency has been all the name of the game with this as far as pinpointing South Carolina on into North Carolina even including portions of Virginia. At this point, the latest models bring this in sometime into the overnight hours now of Thursday and potentially early Friday morning coming in as a Category 4, somewhere around the costal regions of North Carolina from Wilmington up towards Cape Hatteras. The spaghetti model guidance kind of shows the concentration of which right around areas around Wilmington as the best likelihood for landfall and Hatteras certainly in line as well. And when you take a look at the comparison of the most reliable models we typically go to the American and European, American in red. European coming in in blue. Very similar as far as where they`re lined up here going into the overnight hours of Thursday. But notice, once they make landfall both models kind of want to have it hover around for maybe a day or potentially more. That`s what really becomes a danger in scenario here with potential rainfall amounts as much as 20 or more inches on some of these costal communities.

28


Turn static files into dynamic content formats.

Create a flipbook
Issuu converts static files into: digital portfolios, online yearbooks, online catalogs, digital photo albums and more. Sign up and create your flipbook.